Você está na página 1de 150

Preface

God created the natural number, and all the rest is the work of man. ~ Leopold Kronecker

This is a numberful world. Everyone is counting something or counting on something. India faces a
number of problems. You are reading this book because you want numbers from your performance
in CAT. Even serial killers kill for numbers. If you cant express yourself even vaguely in numbers or
understand others numbers, you arent really human. If you understand the numbers completely,
you still arent human you are divine!

Complete mastery over numbers is the primal attribute of any creature of abnormal intelligence.
TestCrackers Book of Numbers is an attempt to make you achieve the maximum level of
competence in Number Theory. The Book discusses all the theories, presents their proof,
demonstrates their applications through solved examples and offers you sufficient number of
unsolved problems to refine your understanding.

The numbers are there. Everywhere. But you need not go in all direction to search for them. All that
has ever been asked and all that can ever be asked in Number Theory is presented and perfected in
this Book. The pacing is smooth and the presentation is lucid. Most importantly, the Book is written
to be liked. In the pages that follow, you will move from the foundation to the pinnacle of Number
Theory.

There is a magical finality to this book, like it is the end of your search

Wherever there is number, there is beauty. Wherever there is number, there is magic. As a true
devotee of numbers, I assure you, I have put my heart and soul and a little bit of magic into this
Book. The book will play its part in taking you to the magical number of 99.XX percentile.

Numberfully yours,

Ashank Dubey
1 | C A T C L A S S E S B Y A s h a n k D u b e y

TYPES OF NUMBERS

Lets try to understand the hierarchy of numbers.

The Number Line

Number line is a line on which all the positive and negative numbers (along with zero) can be
marked in a sequence. It stretches from negative infinity to positive infinity.

All the numbers which can be represented on the number line are called real numbers.

Natural Numbers

Natural numbers are used for counting. Thats why these numbers are also called counting
numbers. The group of natural numbers starts from 1 and includes 1, 2, 3, 4, 5,.. and so on. Zero,
negative numbers, and decimals are not included this group. Natural numbers are also known as
positive integers.

To be able to locate the inequalities on the number line is very helpful in solving problems.


2 | C A T C L A S S E S B Y A s h a n k D u b e y

Illustration: If there are two


inequalities represented by a (x 0)
& b (x > -2) here, by plotting them on
the number line we know that the
solution set will be (-2, 0].

What is the solution set for c & d?

Solved Problems

1. How many times the digits of a computer keyboard need to be pressed in typing the first 146 natural
numbers?
(a) 328 (b) 331 (c) 329 (d) 330
Solution: From number 1 to 9, we will use 1 digit in each number digits used = 9.From number 10 to
number 99, we will use 2 digits in each number digits used = 2 90 = 180. From number 100 to
number 146, we will use 3 digits in each number digits used = 3 47 = 141. Therefore, total number
of digits used = 9 + 180 + 141= 330.

2. How many times do you write the digit 4 while writing numbers from 5 to 500?
(a) 200 (b) 199 (c) 100 (d) 99
Solution: From 5 to 104, digit 4 comes 20 times (10 times at the unit place & 10 times at the 10th place).
Similarly, from 105 to 204, 205 to 304, 305 to 404 & 405 to 504, the digit 4 comes 20 times. But from
400 to 499, digit 4 comes 100 times at 100th place. Therefore, total number of times that we write the
digit 4 from 5 to 504= 20 + 20 + 20 + 20 + 20 + 100 = 200. But, we are looking for number of 4s from 5 to
500. So, the required answer = 200 1 = 199. (Excluding 504)

3. If you write the first 150 natural numbers in a straight line, how many times do you write the digit 0?
(a) 24 (b) 25 (b) 34 (d) 30
Solution: The occurrence of 0 in first 100 natural numbers will be in the following: 10, 20, 30, 140,
150. So, digit 0 comes 15 times at unit place. But from 100 to 109, digit 0 comes 10 times at 10th place.
Therefore, total number of times that we write the digit 0 = 15 + 10 = 25.

4. How many 3 digit natural numbers are there which can be expressed as a perfect square, perfect cube
and a perfect fourth power?
(a) 2 (b) 3 (c) 1 (d) 0
Solution: Let N = x , N = y and N = z . Therefore, N will contain 12th power (LCM of 2, 3 and 4) of a
2 3 4

natural number. Therefore, N = a12 = (a4)3 = (a3)4= (a6)2. The smallest such number is 212 = 4096. There is
no three digit number like that.


3 | C A T C L A S S E S B Y A s h a n k D u b e y

5. A student gets 3 marks for a correct answer and 1 mark is deducted for a wrong answer. If she has
done 80 questions in all and has been awarded only 180 marks, how many of them were wrong?
(a) 65 (b) 15 (c) 64 (d) 20
Solution: If she has done question correctly it means she has done (80 ) questions wrong.
So, 3 x 1 x (80 ) = 180
4 80 = 180
4 = 260
= 65
Hence, she has done 65 correct answer and (80 65) = 15 wrong.

Second Method: If she had done all the 80 correctly she would have got 240 (= 80 x 3) marks, but if she
marks one wrong answer she is liable to lose 4 marks (3 + 1). Thus for every wrong answer she loses 4
marks. Now she has lost 60 marks (240 180 = 60). This implies that she has got 15 answers
!"
wrong = 15 .
!

6. There were 90 questions in an exam. If 3 marks were awarded for every correct answer and 1 mark
was deducted for every wrong answer, how many different net scores were possible in the exam?
(The student can choose to not attempt a question)
(a) 120 (b) 358 (c) 359 (d) 360

Solution: The maximum marks that can be achieved is 3 x 90 = 270 and the minimum marks that can be
achieved is 90. So, from 90 to 270, we should have all the scores at a difference of 1. i.e. 90, 89,
88, 0, .267, 268, 269, 270. Therefore, there should be 90 + 1 + 270 = 361 scores possible. But we
cannot achieve scores of 269 (90 correct & 1 wrong is not possible), 268 (90 correct & 2 wrong is not
possible) and 265 (89 correct & 2 wrong is not possible). You can obtain rest of the scores. Therefore,
total number of scores possible = 361 3 = 358.

7. Numbers 1,2,3,n are written in sequence. Numbers at odd places are struck off and a new
sequence is formed. The same process is repeated until only a single number remains. What is the
final number left if n= 528?
(a) 256 (b) 264 (c) 512 (d) none of these
Solution: After the first removal, the 2 , 4 , 6 , 8 , 10 , 12 , 14 , 16 , will be left. After the second removal,

4 , 8 , 12 , 16 , 20 , 24 ,. numbers will be left. After the third removal, 8, 16 , 24 , 32 , ..numbers will be


left In short, after the nth removal, the first number would be (2n) number in the original number. To
reduce 528 numbers to a single number, we need to perform the halving operation 9 times. Therefore,
the number left would be (29) or 512, which is the highest power of 2 in the given set.

8. Find the total number of squares in a chessboard.
(a) 64 (b) 172 (c) 204 (d) none of these
Solution: In the given question, they are asking how many squares of any dimension from 1x1 to 8x8 are
there on a chess board. The key is to think how many positions there are where square of each size can
be located. A 2x2 square, for example, can be located in 7 locations horizontally and 7 locations
vertically, i.e. in 49 different positions. Consider the table below:


4 | C A T C L A S S E S B Y A s h a n k D u b e y


Size Horizontal Positions Vertical Positions Positions
1x1 8 8 64
2x2 7 7 49
3x3 6 6 36
4x4 5 5 25
5x5 4 4 16
6x6 3 3 9
7x7 2 2 4
8x8 1 1 1
Total 204

In total there are 204 positions. This is the sum of the number of possible positions for all the
different sized squares. Basically, we have to find the sum of squares of 1, 2, 3.7, 8. The sum is
8 x 9 x (2 x 8 + 1)/6 = 204

Quick Recall
! (!!!)
! Sum of first n natural numbers = .
!
! !!! (!!!!)
! Sum of squares of first n natural numbers =
!
!(!!!) !
! Sum of cubes of first n natural numbers =
!
! Sum of first n even numbers = n(n + 1)
! Sum of first n odd numbers = !

Even and Odd Numbers


All the natural numbers, which are divisible by 2 are known as Even numbers and all the
natural numbers which are not divisible by 2 are known as Odd Numbers.
e.g. 2, 4, 6, 8,..etc are even and 1,3, 5, 7, are odd numbers.

Most questions on odd/even numbers test your understanding of their interplay, which is illustrated in
the following table:

Even Even = Even Odd Odd = Even Even Odd = Odd Odd Even = Odd
Even x Even = Even Odd x Odd = Odd Odd x Even = Even Odd x Even = Even
Even Even = Even or Odd Odd Odd = Odd Even Odd = Even Odd Even = (never divisible)
(Odd)Even = Odd (Even)Odd = Even


5 | C A T C L A S S E S B Y A s h a n k D u b e y

Rule of Simplification or Calculation

VBODMAS - The order of operations (sometimes called operator precedence) is a rule used to clarify
which procedures should be performed first in a given mathematical expression. "Operations" mean
addition, subtraction, multiplication, division, etc. If it is not a number it is probably an operation.

In simplification of the expression, the following order needs to be followed.

V Vinculum
B Bracket ( )
O Of Of
D Division
M Multiplication X
A Addition +
S Subtraction
Brackets: - They are used for the grouping of things or entities. The various types of brackets are:

Types of bracket:-

1. [ ] Called as square (or big) bracket.


2. { } Called as curly bracket or brace.
3. ( ) Called as round (or common) bracket.

So first of all we solve the inner most brackets moving outwards. Then we perform of which means
multiplication then Division, Addition and Subtraction.

! Addition and Subtraction can be done together or separately as required.


! Between any two brackets if there is no any sign of + or it means we have to do
multiplication e.g.,
(8) (2) = 8 x 2 = 16
[4 (5) + 7] = 20 + 7 = 27

Some important rules regarding the sign convention in mathematical operations:

(a) + (b) = + (a + b) i.e. (+) + (+) = + (4) + (7) = 11


(a ) + (b) = (b a) i.e. () + (+) = + If the numerical value of + is greater (3) + (8) = 5
(a) + ( b) = (a b) i.e. () + (+) = If the numerical value of is greater ( 8) + (3) = 5
(a) + (b) = (a + b) i.e. () + () = ( 5) + ( 3) = 8

(a) x (b) = ab i.e. (+) x (+) = + (2) x (5) = 10


(a) x (b) = ab i.e. () x (+) = (2) x (5) = 10
(a) x ( b) = ab i.e. (+) x () = (2) x ( 5) = 10
( a) x (b) = ab i.e. () x () = + (2) x (5) = 10


6 | C A T C L A S S E S B Y A s h a n k D u b e y

Examples

1. 45 [28 {37 (15 k)}] = 58, the value of k is


(a) 19 (b) 39 (c) 19 (d) None of these
Solution: Option (c)
45 [28 {37 15 + k)}] = 58
45 [28 {22 + k)}] = 58
45 [6 k] = 58
45 6 + k = 58 k = 19

2. Simplify: 5 + 1/7 of {30-(21+8-4)+ 1/2 of 4} 3
(a) 3 (b) 4 (c) 5 (d) 9
Solution: Option (a)
5 + 1/7 of {30 (21+8-4)+ 1/2 of 4} 3
= 5 + 1/7 of {30 (25)+ 1/2 of 4} 3
= 5 + 1/7 of {30 (25)+ 2} 3 = 5 + 1/7 of 7 3 = 5 + 1 3 = 3

Prime Numbers
All the numbers that have only two factors, 1 and the number itself, are called prime numbers. Hence, a
prime number can only be written as the product of 1 and itself. The numbers 2, 3, 5, 7, 1137, etc. are
prime numbers. Thus-
2, 3, 5, 7, 11, 13, 17, 19, 23, 29, 31, 37, 41, 43, 47, 53, 59, 61, 67, 71, 73, 79, 83, 89, 97,...are prime
numbers.

Important Concept - The spacing p(n+1)-p(n) between neighboring prime numbers goes as - 1, 2, 2, 4, 2, 4,
2, 4, 6, 2, 6, 4, 2, 4, 6, 6, 2, 6, 4, 2, 10, 6, 8,..so that the difference, except for the first, are even
numbers. This observation makes plausible the Goldbach conjecture that any even number can be
represented as the sum of two primes. Thus 64 = 59+5 = 41+23 = 17+47 etc. Also one notices that the
number of primes in a given interval decreases with increasing number n. As first noticed by both
Gauss and Legendre the approximate number of primes N less than n goes as n/ln(n) (that is, the
number divided by its natural logarithm). This is referred to as the Prime Number Theorem and gives
the estimate of n/ln(n)=145 ( to the nearest integer)at n=1000 compared to the actual larger number of
168.

! A more precise statement of this is that if one randomly selects an integer i from the set {1, 2, ..., N},
the probability that i is prime tends to 1/ln(n) in the limit of large N.


! 2 is the smallest and only even prime number.
! There are total 15 prime numbers up to 50.
! There are total 10 prime numbers from 50 to 100.
! All the prime numbers > 3 will be of the format 6K 1 or 4k 1. But vice versa is not always
true.


7 | C A T C L A S S E S B Y A s h a n k D u b e y

Hunting for Primes

A prime sieve is a fast type of algorithm for finding primes. There are many
prime sieves. The simple sieve of Eratosthenes (250 BC), the sieve of
Sundaram (1934), the still faster but more complicated sieve of Atkin, (2004),
and various wheel sieves are most common. A prime sieve works by creating
a list of all integers up to a desired limit and progressively removing

composite numbers (which it directly generates) until only primes are left.
This is the most efficient way to obtain a large range of primes; however, to
find individual primes, direct primality tests are more efficient.

Examples

1. If + 1 is a prime number, then p must be power of:


(a) 2 (b) 3 (c) 5 (d) 12
!
Solution: Since 2 + 1 is a prime number so
! !
2! + 1 = 2; 2! + 1 = 3; 2! + 1 = 5; 2 ! = 2! + 1 = 17; 2 ! = 2! + 1 = 257.
Hence, the value of p is 2 or the power of 2.

2. For how many prime numbers p is p2 + 15p 1 also a prime number?
(a) 0 (b) 1 (c) 2 (d) 3
Solution: When p = 3, the expression gives a prime number (53). When p is not equal to 3, p2 will be of
the form 3k + 1 as every square number is of the form 3k or 3k + 1.
Therefore,
p2 + 15p 1
3k + 1 + 15p 1
3k + 15p, a multiple of 3 (which is not prime). Therefore, for only p = 3, we do get a prime number
(53) from the expression. Option (b)

3. A, B, C, D and E are five prime numbers, not necessarily consecutive. Sum of these five prime numbers
= 264. It is also given that A < B < C < D < E. What is the value of A?
(a) 53 (b) 59 (c) 47 (d) None of these
Solution: Sum of these five prime numbers (A + B + C + D + E) = 264 (Even), which is only possible:
even + odd + odd + odd + odd = even.
As we know, 2 is only even prime number. So, the value of A should be 2.

4. The number of positive integers n in the range 12 n 40 such that the product (n 1) (n 2) (n
3)..321 is not divisible by n is (CAT 2003)
(a) 7 (b) 8 (c) 6 (d) 29
Solution: The product (n 1) (n 2) (n 3)..321 will not be divisible by n only when this product does
not contain factors of n, i.e. n does not have any factor among {(n-1), (n-2), (n-3).3, 2, 1}. This is only
possible if n is a prime number. The prime numbers in the range given are 13, 17, 19, 23, 29, 31, and 37.
There are 7 such numbers in all.


8 | C A T C L A S S E S B Y A s h a n k D u b e y

How to test whether a number N is prime or not?


Step 1: Take the approximate value of square root of N.
Step 2: Then divide the given number by all the prime numbers below the square root obtained.
Step 3: If the number is divisible by any of these prime numbers then it is not a prime number else it is a
prime number.

Example: Is 241 a prime number?
Solution: When we take the square root of 241 it is approximate 15, so we consider 16. Now we divide
241 by all the prime numbers below 16. Since 241 is not divisible by 2, 3, 5, 7, 11 and 13. So it is a prime
number.

Coprime Numbers
Two natural numbers are called co prime (or relatively prime) numbers if they have no common factor
other than 1 or, in other words, the highest common factor i.e. HCF between co prime numbers is 1.
Example: (8, 25); (14, 27), (8, 9), (17, 19) etc.

Composite Numbers
A number other than 1, which is not a prime number is called a composite number. In other words, all
the numbers that have at least three factors are called composite numbers. e.g. 4, 6, 8, 9, 10, 12, etc.

! 1 is neither prime nor composite.
! 4 is the smallest composite number.

Examples

1. The number of composite numbers between 101 and 120 is:


(a) 11 (b) 12 (c) 14 (d) 16
Solution: There are 4 prime numbers between 101 and 120 viz., 103, 107, 109, and 113. Hence, the
number of composite numbers between 101 and 120 is 20 4 = 16.

2. N =11111.111 (91 times). N is
(a) prime number (b) composite number (c) perfect square (d) cant say
Solution: N =11111.111 (91 times).
Since 91 = 7 x 13, we can write
11...1(91 1's) = (1111111)x(10000001000000100000010000010... 13 1's in the right, with 6 0's between
them)
or = (1111111111111)x(10000000000001000000000000100000000000010000000000010... -- 7 1's in
the right, with 12 0's between them)

! Therefore, the 111111(91 times) is divisible by (1111111) as well as (1111111111111). So, It is a


composite number.


9 | C A T C L A S S E S B Y A s h a n k D u b e y

3. Factorials
The product of n consecutive natural numbers starting from 1 to n is called as the factorial n.
n! = 1 x 2 x 3 x 4 x 5 x 6 x 7 x x (n 2) x (n 1) x n
e.g. 5! = 1 x 2 x 3 x 4 x 5 = 120; 6! = 1 x 2 x 3 x 4 x 5 x 6 = 720


! 0! = 1 and 1! = 1
! n! always ends with zero if n 5
! The product of n consecutive natural numbers is always divisible by n!, where n! = 1 2 3 4
5. n

Examples

! !
1. If ! = , then the value of is:
! !
(a) 5 (b) 6 (c) 9 (d) 12
!!! ! .. ! ! ! !
Solution: ! =
!!! !
= . !
! = + 2 + 3 + 4
Now according to answer options, Option (a): 5! 7 x 8 x 9; Option (c): 9! 11 x 12 x 13 but
Option (b): 6! = 8 x 9 x 10
720 = 720. Hence, n = 6 is the correct answer.

2. The appropriate value of n for the relation (n! + 1) = + is:


(a) 3 (b) 4 (c) 5 (d) None of these
Solution: Lets consider n = 3,
(3! + 1) = 3 + 1 !
7 16
Now, If we consider n = 4.
(4! + 1) = 4 + 1 !
(24 + 1) = 5 !
25 = 25. Hence Option (b) is correct.

3. If n + n! = , then the value of n is:
(a) 4 (b) 5 (c) 6 (d) 7
Solution: Consider n = 5, then
5 + 5! = 5!
5 + 120 = 125
125 = 125. Option (b) is correct.

4. If = ! Where > > 1 and = then the value of + is:
(a) 20 (b) 60 (c) 90 (d) 272
Solution: Let us consider b = 2 (b >1); then = 4! 4 = 12
!

& ! = (4 2)! = 2! = 2 Hence it is impossible. Again consider n = 3, then ! = 9! 9 = 720 and


! = (9 3)! = 6! = 720
Thus we get b = 3 and = 9 the probable values
Now ! + ! = 9! + 3! = 81 + 9 = 90. Hence Option (c) is correct.


10 | C A T C L A S S E S B Y A s h a n k D u b e y

5. The value of (1.2.3.9).(11.12.13.19).(21.22.23.29).(31.32.33..39)....(91.92.9399)
! ! !
(b) (c) (d) None of these

Solution: (1.2.39).(11.12.13..19).(21.22.23.29).(91.92.93.99)
!" !" !"
= (1.2.39) (11.12.13..19) (21.22.23.29) .(91.92.93.99)
!" !" !"

!.!.!.!..!! !!! !!!


= !".!".!"...!" = !"#$$% !"! = !"#$$ !"!" . Option (c).

6. Given = for any natural number .If ( + ) = , then what is the value of
?
(a) 4 (b) 5 (c) 2 (d) None of these

Solution: As we know, n! = n(n 1)!. The given function is the factorial function.
= 1
+ 2 = ( + 2) + 1
+ 2 = ( + 2) ( + 1) .(1)
( + 2) = 20 {Given in the question, compare this equation with equation (1)}
( + 2) + 1 = 20 ( + 2) + 1 = 5 x 4 = 3. Therefore the answer is (d).

7. If n is an odd natural number, what is the highest number that always divides n (n2 1)?
(a) 6 (b) 12 (c) 24 (d) None of these
2
Solution: n (n 1) = (n 1) n (n + 1), which is a product of three consecutive numbers. Since n is
odd, the numbers (n 1) and (n + 1) are both even. As they are two consecutive even numbers one of
these numbers will be a multiple of 2 and the other will be a multiple of 4.

Hence, their product is a multiple of 8. Since one out of every three consecutive numbers is a multiple of
3, one of the three numbers will be a multiple of three. Hence, the product of three numbers will be a
multiple of 8 3 = 24. Hence, the highest number that always divides n (n2 1) is 24.

! ! ! !
8. Find the sum of all values of n which satisfy + = 240
! ! !
Solution: Let n! = k. Taking 4! off both sides and simplifying, we get
k2 5760k + 3628800 = 0 k2 (6! +7!)k + 6! 7! = 0 k = 6! and 7! n = 6 and 7
Sum = 13

9. What is the remainder if (2n)! is divided by (n!)2?
(a) 0 (b) 2 (c) 4 (d) 1
Solution: (2n)! = 1 2 3 4 (n 1) n (n + 1) 2n = (n)! (n + 1) (n + 2) 2n. Since (n
+ 1) (n + 2) 2n is a product of n consecutive numbers, it is divisible by n!. Hence, the product (n)!
(n + 1) (n + 2) 2n is divisible by n! n! = (n!)2. The remainder therefore is 0.



11 | C A T C L A S S E S B Y A s h a n k D u b e y

Perfect Number
When the sum of all the factors (including 1 but excluding the number itself) of the given number is the
same number then this number is called as perfect number. E.g., 6, 28, 496, 8128,.. etc. [As the factors
of 28 are 1, 2, 4, 7, 14 and 28. Now, we can see (1 + 2 + 4 + 7 + 14 = 28). Hence 28 is a perfect number.]

! So far only 27 perfect numbers are known.

Perfect Squares
A square number or perfect square is the product of some integer with itself. For example, 9 is a square
number, since it can be written as 33 or (-3) x (-3).

! All the perfect squares can be expressed as 3k or 3k+1 and 4k or 4k + 1. But vice versa is not
always true
! Square of a natural number can only end in 00, 1, 4, 5, 6, and 9.
! No perfect square can end in 2, 3, 7, 8 or single 0.
! The tens digit of every perfect square is even unless the square is ending in 6 in which case
the tens digit is odd.
! Square of any prime number (>3) can be expressed as 6k + 1.
! A square number cannot be a perfect number.

1. If is a natural number which is a perfect square, then the number + must end in:
(a) 0 or 5 (b) 0 or 1 or 9 (c) 0 or 2 or 6 (d) 0 or 4 or 8
Solution: 1 + 1 = 2 4 + 4 = 6
9+ 9 = 12 16 + 16 = 20
25 + 25 = 30 36 + 36 = 42
Thus, we get the unit digit as 0, 2 and 6. Hence (c) is the correct option.

2. The expression 1! + 2! + 3! + 4! +..+ n! (where n 5) is not a/an:
(a) Composite number (b) Multiple of 3 (c) Perfect square (d) odd number
Solution 1 (short cut): 1! + 2! + 3! + 4! + 5! = 1 + 2 + 6 + 24 + 120 = 153, which is a composite odd
number and a multiple of 3. Since, the options (a), (b) and (d) are ruled out so option (c) is the correct
one. That is, the sum cannot be a perfect square.

Solution 2 (theoretical):
n=1 k = 1! = 1, a perfect square
n=2 k = 1! + 2! = 1 + 2 = 3
n=3 k = 1! + 2! + 3! = 1 + 2 + 6 = 9, a perfect square
n=4 k = 1! + 2! + 3! + 4! = 1 + 2 + 6 + 24 = 33
n=5 k = 1! + 2! + 3! + 4! + 5! = 1 + 2 + 6 + 24 + 120 = 153

Now note that even factorial above 5 will always end in 0 (because 5! ends in 0). Therefore, for n 5,
the sum of factorials till n will always have as the units digit the same as the units digit of 1! + 2! + 3! +
4!, that is 3. Therefore, we have proved that for n 5, the sum of factorials till n will always end in 3. We
also know that no perfect square ends in 2, 3, 7, 8 or single 0. The correct answer is option (c).


12 | C A T C L A S S E S B Y A s h a n k D u b e y

3. Find the smallest positive integer n for which (22 1)(32 1)(42 1) (n2 1) is a perfect square.
(a) 6 (b) 8 (c) 12 (d) None of these
Solution: n term = (n 1) = (n + 1)(n 1) series: (22 1)(32 1)(42 1)(n2 1)
th 2

= 1 3 2 4 3 5 (n 2) (n) (n 1) (n + 1)
= 2 x n x (n + 1) k2, because all the other terms are squared.
The first value of n which makes 2 x n x (n + 1) a perfect square is n = 8.

4. For which integer n is 28 + 211 + 2n is a perfect square?
(a) 10 (b) 11 (c) 12 (d) None of these
Solution: In order to write the above expression in the form (a + b)2 = a2 + 2ab + b2, we note that 28 =
(24)2 and 211 = 2 24 26.
Therefore, we need the square of 26 2n = (26)2 = 212 n = 12.

5. 1 and 8 are the first two natural numbers for which 1 + 2 + 3 + ... + n is a perfect square. Which
number is the 4th such number?
(a) 49 (b) 144 (c) 98 (d) None of these
! (!!!)
Solution: 1 + 2 + 3 + + n = = ! n(n + 1) = 2 !
!
Now n and (n + 1) will have no factor in common. Since RHS is twice the square of a natural number, one
of n and n + 1 will be twice of a perfect square and the other will be a perfect square. As twice of a
perfect square will be even, the other square will be odd. We start investigating the odd squares and
their neighbors. The fourth such numbers we get are 288 289.
Therefore, ! is (288 x 289)/2 => n = 288 (Answer)

Triangular Numbers

A triangular number is obtained by adding the previous number to the nth position in the sequence of
triangular numbers, where the first triangular number is 1. The sequence of triangular numbers is given
as follows

1, 3, 6, 10, 15, 21, 28, 36, etc.

1 = 1
3 = 1 + 2
6 = 1 + 2 + 3
10 = 1 + 2 + 3 + 4
15 = 1 + 2 + 3 + 4 + 5
21 = 1 + 2 + 3 + 4 + 5 + 6
. .
. .
. .
! = !(!!!)
=
!

What happens when we add consecutive triangular numbers 1 + 3? The addition gives the number 4
which is a square. What happens when we add 1 + 3 + 5? Again, our addition gives us a perfect square-


13 | C A T C L A S S E S B Y A s h a n k D u b e y

the number 9. If we take 4 balls and arrange them, we can easily make a square like figure. Same goes
with 9 and with all the numbers which are perfect squares.

Corollary 1: The sum of two consecutive triangular numbers is always a perfect square.
Proof: The sum of the nth and the (n+1)th such numbers is [n(n+1)/2 + (n+1)(n+2)/2] = (n+1)2

Following figures would definitely help you to understand it geometrically:-

Examples

1. A child was asked to add first few natural numbers (that is 1 + 2 + 3 +) so long his patience
permitted. As he stopped, he gave the sum as 575. When the teacher declared the result wrong, the
child discovered he had missed one number in the sequence during addition. The number he missed
was: (CAT 2001)
(a) less than 10 (b) 10 (c) 15 (d) more than 15
Solution:
! (!!!)
1 + 2 + 3 + 4 + + n =
!
! (!!!)
> 575 ( + 1) > 1150. (Now, take approximate square root of 1150)
!
! (!!!)
( + 1) = 34 x 35 = 1190 = 595.
!
Sum should be 595 but he got 575 after missing a number. The required number is 595 575 = 20.

2. All the pages numbers from a book are added, beginning at page 1. However, one page number was
mistakenly added twice. The sum obtained was 1000. Which page number was added twice?
(a) 44 (b) 45 (c) 10 (d) 12
Solution: Let the total number of pages in the book be n. Let the page number be repeated.
! (!!!)
+ = 1000
!
! (!!!)
Thus, < 1000 ( + 1) < 2000. (Now, take approximate square root of 2000)
!
( + 1) = 44 x 45.
! (!!!)
= 990. Sum should be 990 but he got 1000 after adding one number twice. The required number
!
is 1000 990 = 10. Option (c)

3. The 288th term of the series a, b, b, c, c, c, d, d, d, d, e, e, e, e, e, . is
(a) u (b) v (c) w (d) x
Solution: The number of terms of the series forms the sum of first n natural numbers i.e.
! (!!!)
= 288 ( + 1) = 576 (take approximate square root of 576).
!
n = 24. The 288th term will be the 24th letter which is x.


14 | C A T C L A S S E S B Y A s h a n k D u b e y

Whole Numbers
When zero is included in the list of natural numbers, the numbers are known as whole numbers. The
group of whole numbers starts from 0 and including 1, 2, 3, 4, 5,.. and so on.

Integers
A group which consists of natural numbers, negative integers (- 1, - 2, - 3..) and zero is known as the
set of integers.

Examples

1. When a two-digit number is subtracted from the other two-digit number, which consists of the same
digits but in reverse order, then the difference comes out to be a two-digit perfect square. The
number is:
(a) 59 (b) 36 (c) 37 (d) More than one
Solution: Let x , y tens and unit digits of the original number then: (10 + y) (10y + ) = 9( y).
Since the difference between number is a perfect square. So this difference can be only (9 x 4) = 36,
because 36 is the only two digit perfect square contains 9 as a factor. But there are total 5 numbers
possible viz., 15, 26, 37, 48, 59.
Since the only condition is that (10 + y) (10y + ) = 9( y) = 36 ( y) = 4. Option (d)

2. A three digit number which on being subtracted from another three digit number consisting same
digits in reverse order gives 594. The minimum possible sum of all the three digits of this number is:
(a) 6 (b) 8 (c) 9 (d) 12
Solution: Let x , y and z be the hundred, tens and unit digits of the original number then
(100z + 10y + x) (100x + 10y + z) = 594
99(z x) = 594 (z x) = 6
So the possible values of (x, z) are (1, 7), (2, 8) and (3, 9). Again the tens digit can have the values viz., 0,
1, 2, 3, , 9. So the minimum possible value of x + y + z = 1 + 0 + 7 = 8. Option (b)

3. Find the sum of all two-digit positive integers which exceed the product of their digits by 12.
(a) 28 (b) 39 (c) 67 (d) None of these
Solution: Let the two-digit integer be ab. Therefore, 10a + b = ab + 12 10a 10 + b ab = 2
(a 1)(10 b) = 2
Numbers are 28 or 39. Sum = 28 + 39 = 67.

4. A two-digit number is 18 less than the square of the sum of its digits. How many such numbers are
there?
(a) 1 (b) 2 (c) 0 (d) More than 2
Solution: Let the two-digit number be AB, where A and B are single digits.
Therefore, 10A + B = (A + B)2 18. Now, the highest value of 10A + B can be 99, therefore the highest
value of (A + B)2 18 can also be 99. Also, (A + B)2 will be greater than 18 to keep the R.H.S. positive.
(A + B)2 = 25, 36, 49, 64, 81, 100.
10A + B = 7 (not possible), 18 (not possible), 31 (not possible), 46 (not possible), 63, or 82. We see
that two pairs (A, B) = (6, 3) and (8, 2) satisfy the above condition.


15 | C A T C L A S S E S B Y A s h a n k D u b e y

5. The numbers 123 456 789 and 999 999 999 are multiplied. How many times does digit 9 come in the
product?
(a) 0 (b) 1 (c) 2 (d) 3
Solution: Option (a)
123456789 999999999 = 123456789 (1000000000 1) = 123456789000000000 123456789
= 123456788876543211

Fractions

Suppose you have lent Rs. 4,000 to your friend in the last month. Now, you have asked him to
return your money. But he is paying you only Rs. 2,000 and rest amount he wants to return you in the
next month. It means he is paying the total amount not in a one lot but in a fraction that means in
parts

So, we can say that when any unit of a thing is divided into equal parts and some parts are
! ! ! !
considered, then it is called a fraction. Examples include , , , etc.
! ! ! !

Types of fraction: There are several types of fraction but we can categorize them into three parts.

(i) Proper Fraction: If the numerator in a fraction is smaller than the denominator, assuming both are
positive, the fraction is said to be a proper fraction. Proper fractions represent numbers between 0 and
1.
! ! !
Examples include , , etc.
! !! !"

(ii) Improper Fraction: If the numerator in a fraction is larger than the denominator (assuming both are
positive), the fraction is said to be an improper fraction. Improper fractions represent numbers greater
than 1 and are sometimes called top-heavy fractions.
! ! !
Examples include , , etc.
! ! !

(iii) Mixed Fraction: Mixed fractions comprise two parts, a whole number followed by a fraction. Improper
fractions represent numbers which can be written as a mixed fraction, as part whole number and part
! !
fraction. Take the improper fraction , which can be written as 1 , a mixed fraction.
! !
! ! !
Examples include 3 , 5 , 1 etc.
! ! !


16 | C A T C L A S S E S B Y A s h a n k D u b e y

Examples

1. A fraction becomes 4 when 1 is added to both the numerator and denominator and it becomes 7
when 1 is subtracted from both the numerator and denominator. The numerator of the given fraction
is:
(a) 2 (b) 3 (c) 7 (d) 15
!
Solution: Let the fraction be , then
!
!!! !!!
= 4 ..(1) and = 7.(2)
!!! !!!
Solving equations (1) and (2),
= 15 and y = 3

2. A has certain amount in his account. He gives half of this to his eldest son and one third of remaining
to his youngest son. The amount with him now is:
(a) 1/3 of the original (c) 1/6 of the original
(b) 3/4 of the original (d) 2/3 of the original
! !
Solution: Let A was having 1. He gives half of this to his eldest son 1 =
! !
! ! !
Now one third of remaining to his youngest son = x =
! ! !
! ! !
Thus the amount with him now = 1 = .
! ! !

3. Two sets A and B are given below.
A = , , , ,
B = , , , ,
How many different proper fractions can be made by picking the numerator from one of the sets and
the denominator from the other set?
(a) 24 (b) 20 (c) 12 (d) None of these
Solution:
A = {1, 2, 4, 8, 16}
B = {1, 3, 9, 27, 81}
The number of possible proper fractions when denominator is equal to 2, 4, 8 and 16 are 1, 2, 2 and 3
respectively.
The number of possible proper fractions when denominator is equal to 3, 9, 27 and 81 are 2, 4, 5 and 5
respectively.
Since no two of these fractions can be equal, the answer = 24.

Alternate method: All the combinations can result in a proper fraction except when 1 is chosen
from both the sets.
So the answer = 5 5 1 = 25 1 = 24.



17 | C A T C L A S S E S B Y A s h a n k D u b e y

(iv) Continued Fraction: A continued fraction consists of the fractional denominators.

!
Example includes 3 + ! , These fractions are solved from the bottom towards upside.
!! !
!! !
!! !
!! !!


4. Simplify the following expression: 1 +
!
!
!
!

! ! ! !
Solution: 1 + ! = 1 + ! = 1 + ! = 1 + ! = 1 + !!! ! = 1 + !! = !"!
!! ! !! ! !! ! !! ! !
!! ! !! ! !! ! !!
!
!! ! !!
!! ! !
!

5. Which of the following is showing correct relation:

(a) A > B (b) A < B (c) A B = 2014 (d) A = B


Solution: Option (a)
As both A and B are symmetric, we can check the relations between them taking some smaller numbers
! ! !
too. 1 + = 1.5, 1 + = 1.4285, 1 + = 1.4331
!
! ! ! = 1.4333, 1 + !
!! !! ! !! !
! !! !!
! !
!!
!

Therefore, we can see that when the last term is odd, the value is less than the previous term.
When the last term is even, the value is more than the previous term.

Rational Numbers
!
Any number that can be expressed as the ratio of any two integers i.e, in the form of , where a and b
!
are two integers coprime to each other and b 0 are called rational numbers. These numbers contain
decimal expansion that either do not exist (as in 5 which is 5/1), or terminate (as in 2.9 which is 29/10),
or repeat with a pattern (as in 2.333... which is 7/3).


18 | C A T C L A S S E S B Y A s h a n k D u b e y

Examples

1. Convert 0.77777777777into rational form.


Solution: Let = 0.7777777
10 = 7.777777777777..
10 = 7 + 0.777777777
10 = 7 +
!
9 = 7 =
!

2. Convert 0.3131313131into rational form.
Solution: Let = 0.3131313131
100 = 31.31313131..
100 = 31 + 0.313131
100 = 31 +
!"
99 = 31 =
!!

Rule: To express a recurring fraction in rational form, write the recurring digits once in the numerator
and write as many 9s in the denominator as are the number of recurring digits.
!"# !"#$
Example, 0.abcabcabc. = and 0.abcdabcdabcd.. =
!!! !!!!


! !"# !"#$!!%&' !"#$ !"#$$%& !"#$
! The form of a purely recurring number =
! !" !"#$ !! ! !" !"#$%& !" !"#"$% !" !"# !"#$!!%&' !"#$

3. Convert 0.17555555.into rational form.


Solution: Let A = 0.17555555.
100A= 17.5555.. (1)
1000A = 175.555 (2)
Subtract equation (1) from equation (2).
900A = 175 17
!"#!!" !"#
A = =
!"" !""

4. Convert 3.15474747.into rational form.
Solution: Let R = 3.15474747.
100R= 315.474747..
!"
100R= 315 +
!!
!"#!#
100R=
!!
!"#!#
R =
!!""


19 | C A T C L A S S E S B Y A s h a n k D u b e y

Rule: To write a fraction, which has both recurring and non-recurring parts, in a rational form, do the
following steps:

! Numerator: (Number formed by writing all the digits once) (Number formed by writing all the
nonrecurring part once) = 31547 315 = 31232.

! Denominator: Number of 9s equal to number of recurring digits followed by number of zeroes
equal to non-recurring digits after the decimal.

5. Let D be a recurring decimal of the form D = 0. .., where digits and lie between
0 and 9. Further, at most one of them is zero. Which of the following numbers necessarily produces an
integer, when multiplied by D? (CAT 2000)
(a) 18 (b) 108 (c) 198 (d) 288
Solution: D = 0. ! ! ! ! ! ! .,
!! !!
D = . So D must be multiplied by 198 as 198 is a multiple of 99.
!!
Hence, Option (c) is correct.


6. M and N are integers such that 0 N 9 and = . = 0.9N59N59N5..Then the value of M + N

is equal to
(a) 752 (b) 789 (c) 853 (d) 927
! !!! ! !!! !!! !"
Solution: = = M =
!"# !!! !" !" !"
As 30 is not divisible by 37 and M is a whole number 9N5 is a three digit multiple of 37 which begins in 9
& ends in 5
9N5 = 925 N = 2 M = 750 M + N = 752

7. is a number that is divided by xy where < and gives a result 0. .,
then find the value of .
(a) 11 (b) 33 (c) 66 (d) 99
Solution: Let D = 0. ! ! ! ! ! !
!! !!
D = .
!!

As we know, ! ! is a number that is divided by gives result D.


!! !! !! !!
= = 99.
!! !"

+
8. What is the value of ?

!!"#$ ! !!"## !!"## (!! !!) !" !"
Solution:
!!"#$ ! !!"#"
= !!"#" (!! !!) = !" = !"


20 | C A T C L A S S E S B Y A s h a n k D u b e y

Irrational Numbers
!
Any number that cannot be expressed in the form , where a and b are two integers co prime to each
!
other and q 0 are called irrational number (imaginary or complex numbers are not included in
irrational numbers). These numbers have decimal expansion that never terminate and never repeat with
a pattern. The most famous irrational number is 2; also known as Pythagorass constant.

Examples include , e, 2, 2 + 3, 5 2 etc.

2, 2 + 3, 5 2 etc are irrational quantities also called as Surds. If you can't simplify a number to
remove a square root (or cube root etc) then it is a surd.

Examples:

!
1. Express the value of as a fraction whose denominator is rational.
!
! !!! ! ! !!! ! (! ! ! ! !)(! ! ! ! !) !!" ! !" !
Solution: = = =
!"! !" ! !! ! ! (! ! ! ! !)(! ! ! ! !) !"


2. Express the value of ! !
as a fraction whose denominator is rational.
! ( ! ! !" ! !")
Solution: ! ! !" ! !"
= ( ! ! !" ! !")( ! ! !" ! !")

( ! ! !" ! !") ( ! ! !" ! !") !" ( ! ! !" ! !")


= ! ! = ! !"
= !"

! ! !" ! !"

! !
3. If = !
and = !
, then the value of + +
!"! !" ( !"! !")( !"! !")
Solution: = = ( = ( 18 + 17)! = 35 + 2 306
!"! !" !"! !")( !"! !")
!
Similarly, = 18 17 = 35 2 306
! ! !
! + ! + = 35 + 2 306 + 35 2 306 + 35 !
2 306

= 3675 + 1224 = 4899

Real Numbers

The set of real numbers is made up of all the Rational and Irrational Numbers i.e. numbers that can be
expressed on the number line is call as real numbers. In other words, all the real numbers can be felt or
experienced in the real world.

! !!
Examples include 2, 0, 5, , 11, 7, e, , , etc.
! !


21 | C A T C L A S S E S B Y A s h a n k D u b e y

Imaginary Numbers

These numbers are formed by the imaginary number i (i = 1) i.e. if the square of a number is negative
then this number is called as an imaginary number. In other words, Imaginary numbers are those
numbers about which we can just imagine but cannot physically perceive.

Examples include 3, 1, i, 3i, 9.3i, ()i, etc.

An imaginary number is denoted by i, where = 1

Note:

I. ! = 1, ! = , ! = 1, ! = , ! = 1 etc.
II. ! = !!!! = ! ! x ! = 1 x ! = !
III. If both a, b are negative i.e. imaginary then x
3 x 7 = 3 x 7 = 21 ! = 21
But 3 x 7 = 3 7 21

Examples

1. What is the value of + + +


(a) 1 (b) 1 (c) 0 (d) i
Solution: !"# + !"# + !"" + !"#
= ! !"
+ ! !"!!
+ ! !"!!!
+ ! !"!!

= 1 + + ( 1) + () = 0. Hence, option (c) is correct.

! ! ! !
2. What is the value of ?
! ! ! !
(a) 1 (b) 1 (c) 0 (d) i
! ! ! !" ! !
Solution: = = 1, = x = 1, = 1, !" = 1, !" = 1, and so on.
! !"

! ! ! ! !" ! ! !" !! !" ! ! !" !!! ! !!! ! !


Hence, = = 1
! !" ! ! !" ! ! !! !! !" ! ! !" ! ! ! ! !! ! !!!


3. What is the value of + (!) + (!) + (!) ?

(a) 1 (b) 0 (b) 1 (d) Cannot be determined
! ! ! !
Solution: ! + + ! (!!!) + ! (!!!)
! ! (!!!)

! ! ! !
= ! ! + ! ! ! + ! ! ! ! + ! ! ! !

! ! !! ! !! ! !
= ! ! + ! ! x ! ! + ! ! x ! ! + ! ! x ! !

! !! !! ! ! !! !! !
= ! ! + ! ! + ! ! + ! ! = ! ! + ! ! + ! ! + ! ! = 0.


22 | C A T C L A S S E S B Y A s h a n k D u b e y

Complex Numbers

A Complex Numbers is a combination of a real number and an imaginary number in the form a + ib.
where a and b are purely real numbers and i = 1, is an imaginary number. It is denoted by z = a + ib
where real z = a and imaginary z = b.

Examples include 3 + 5i, 7 + (4)i, (often written as 7 - 4i).

Conjugate Complex Number: The complex number z = a + ib and z = a ib are called the complex
conjugate of each other, where i = 1, b 0 and a and b are real numbers.

!
! A number in the form is written in the form of a complex number by multiplying both
!!!"
numerator and denominator by the conjugate of a + ib, i.e. a ib.

! !!!" !!!" !!!" ! !"


= = = = , which is in the form x + iy.
!!!" (!!!")(!!!") ! ! !(!!)! !! ! !! !! ! !! !! ! !!

Examples

!
1. What is the smallest positive integer n for which = ?
!
(a) 2 (b) 4 (c) 8 (d) 16
!
!!! ! (!!!)(!!!) ! (!!!)!
Solution: = = = ! = 1 = ! n = 4.
!!! (!!!)(!!!) !

Hence, option (b) is correct.

!
2. What is the value of ?
!
(a) 1 (b) 1 (c) i (d) i
!
!!! ! (!!!)(!!!) ! (!!!)!
Solution: = = = ! =
!!! (!!!)(!!!) !

Hence, option (c) is correct.

Remember: (1+i)/(1-i) = i

!
3. If = p + iq then the value of (p, q) is:
!
(a) (0, 1) (b) (0, 0) (c) (1, 0) (d) (- 1, 0)
!!! !"# (!!!)(!!!) !"# !!! !"# !"#
Solution: = = = = 1
!!! (!!!)(!!!) !

Thus, p + iq = 1 p = 1 and q = 0
So the value of (p, q) = (1, 0). Hence, option (c) is correct.



23 | C A T C L A S S E S B Y A s h a n k D u b e y

Involution
The process of multiplication of number several times by itself is known as Involution. We use the same
method in some algebraic expression as follows.

1. (a + b)2 = a2 + b2 + 2ab
2. (a b)2 = a2 + b2 2ab
3. (a + b) + (a b) = 2(a2 + b2) (By adding the difference of formula 1 and 2)
2 2

4. (a + b)2 (a b)2 = 4ab (By taking the difference of formula 1 and 2)


5. (a2 b2)= (a + b)(a b)
6. (a + b)3 = a3 + 3a2b + 3ab2 + b3 = a3 + b3 + 3ab(a + b)
3
7. (a b) = a3 3a2b + 3ab2 b3 = a3 b3 3ab(a b)
8. a3 + b3 = (a + b) (a2 + b2 ab)
3 3
9. a b = (a b)(a2 + b2 + ab)
10. (a + b + c)2 = a2 + b2 + c2 + 2(ab + bc + ca)
11. a + b + c 3abc = (a + b + c) (a2 + b2 + c2 ab bc ca)
3 3 3


Examples
1. Find the value of + + 3abc if a + b + c = 12 and ab + bc + ca = 47.
(a) 36 (b) 42 (c) 30 (d) None of these
Solution: a + b + c = 12
+ + ! = ! + ! + ! + 2( + + ) = 144
! + ! + ! + 2 x 47 = 144 ! + ! + ! = 50
Now, since ! + ! + ! 3abc = ( + b + c)( ! + ! + ! )
! + ! + ! 3abc = 12(50 47) = 12 x 3 = 36.

2. If ( + ) = 17 and ( ) = 1, then the value of is:
(a) 27 (b) 35 (c) 72 (d) None of these
Solution:
( + ) = 17 .. (1)
( ) = 1 (2)
Subtract equation (2) from (1)
2 = 16 = 8 b = 64.
Put the value of b is equation (2) = 81.
= 64 81 = 8 x 9 = 72.

!
3. Find the value of
! !

!!! ! ! !!! ! !!!!!!! { !!! ! !!!! !!!! !!! ! }
Solution: = = {777 333} = 444
!!! ! !!!! !!!! !!! ! !!! ! !!!! !!!! !!! !

4. is at least divisible by:
(a) 71 and 23 (b) 23 and 74 (c) 71 and 94 (d) 23, 72 and 94
Solution: ! + ! + ! = 3abc when a + b + c = 0
94! + ( 23)! + (71)! = 3(94)(23)(71) as (94 23 71 = 0)
Given expression is divisible by 94, 23 and 71.


24 | C A T C L A S S E S B Y A s h a n k D u b e y

DIVISIBILITY RULES

Divisibility by 2, , , , .. Or 5, , , , ..

A number is divisible by 2 or 5, 2! or 5! ,2! or 5! ,2! or 5! , 2! or 5! .. 2! or 5! when the number


formed by the last one, two, three, four, five..n digits is divisible by 2 or 5, 2! or 5! ,2! or 5! ,2! or 5! ,
2! or 5! .. 2! or 5! respectively.

Numbers Divisibility Rule


2 or 5 Last digit
or Last two digits
or Last three digits
.. .
or Last n digits

Examples: 1246384 is divisible by 8 because the number formed by the last three digits i.e. 384 is
divisible by 8. The number 89764 is divisible by 4 because the number formed by the last two digits, 64
is divisible by 4.

Divisibility by 3 and 9
If sum of the digits of the given number is divisible by 3 and 9 then the actual number will also be
divisible by 3 and 9 respectively.
e.g. 12357 is divisible by 3 since the sum of the digits 1 + 2 + 3 + 5 + 7 = 18 is divisible by 3
Similarly, 3277953 is divisible by 9, since 3 + 2 + 7 + 7 + 9 + 5 + 3 = 36 is divisible by 9.

Divisibility by 7, 11, and 13
A number can be divisible by 7, 11 and 13 if and only if the difference of the number formed by the last
three digits and the number formed by the rest digits is divisible by 7, 11 and 13 respectively.

Examples

1. Is 139125 divisible by 7?
Solution: we take the difference as given below
139 125 = 14
Since, the difference is divisible by 7.
Hence the given number is also divisible by 7.

2. Is 1234567 divisible by 13?
Solution: we take the difference as given below
1234 567 = 667
Since 667 is not divisible by 13.
Hence the number is not divisible by 13.

Divisibility by Composite Numbers
Whenever we have to check the divisibility of a number N by a composite number C, the number N
should be divisible by all the prime factors (the highest power of every prime factor) present in C.


25 | C A T C L A S S E S B Y A s h a n k D u b e y

Divisibility by 6
A number is divisible by 6 only when it is divisible by 2 and 3 both. So first of all see that the number is
even or not then we check for the divisibility by 3.

Divisibility by 10
A number is divisible by 10 is and only if when it is divisible by both 2 and 5. So it can be easily observed
that a number is divisible by 10 must ends up with zero at the right hand.

Divisibility by 12
A number is divisible by 12 only when it is divisible by 4 and 3 both at the same time. So first of all check
the divisibility by 4 then 3.

Divisibility by 15
A number is divisible by 15 only when it is divisible by 3 and 5 both simultaneously. So first of all check
the divisibility of the number by 5 then 3.
Thus we can conclude that any number which is divisible by a composite number must be divisible by all
its prime factors.

! Any six-digit, or twelve-digit, or eighteen-digit, or any such number with number of digits
equal to multiple of 6, is divisible by each of 7, 11 and 13 if all of its digits are same.

Examples

1. Find the digit A if the number 888888A999999 is divisible by 7, where both the digits 8 and 9 are 50
in number.
(a) 3 (b) 0 (c) 7 (d) more than 2
Solution: We know that 888888 and 999999 will be divisible by 7. Hence 8 written 48 times in a row and
9 written 48 times in a row will be divisible by 7.
888888A999999 = 888..(48 times) 88A99 99948 times
Hence we need to find the value of A for which the number 88A99 is divisible by 7. By trial we can find A
is = 5.

2. A number consisting entirely of the digit one is called a repunit; for example, 11111. What is the
smallest repunit that is divisible by 63?
Solution: To be divisible by 63, the number has to be divisible by 9 & 7. We have seen above, that any
number with number of digits equal to multiple of 6, is divisible by each of 7, 11 and 13 if all of its digits
are same. Therefore, to be divisible by 7, the repunit has to be 111111 or 111111111111 or
111111111111111111 and so on
Now, to be divisible by 9, the repunits sum of digits has to be divisible by 9. The minimum such number
(multiple of both 6 & 9) is 18.
Therefore, the required repunit is 111111111111111111.






26 | C A T C L A S S E S B Y A s h a n k D u b e y

Exercise 1.1

1. A number N = 897324P64Q is divisible by both 8 8. How many numbers are there between 200
and 9. Which of the following can be the value and 800, which are divisible by both 5 and 7?
of (P + Q) (a) 16 (b) 17
(a) 2 (b) 5 (c) 15 (d) None of these
(c) 10 (d) None of these
9. How many numbers are there between 100
2. Ashank had forgotten his 6 digit bank account and 700 which are divisible by neither 5 nor 7?
number but only remembered that it was of the (a) 409 (b) 410 (c) 411 (d) 412
form X515X0 and was divisible by 36. What was
the value of X? 10. The number which when divided by 33 leaves
(a) 4 (b) 7 (c) 8 (d) 9 no remainder and is closer to 1000 is:
(a) 990 (b) 999 (c) 1023 (d) 1025
3. The first 20 natural numbers from 1 to 20 are
written next to each other to form a 31 digit 11. When a number N is divided by a proper
number N = 12345671920. What is the divisor D then it leaves a remainder of 14 and
remainder when N is divided by 16? if thrice of that number is divided by the same
(a) 0 (b) 4 (c) 7 (d) 9 divisor D, the remainder is 8. Again if 4 times
of the same number is divided by D the
4. A certain number N when multiplied by 13, the remainder will be:
resultant value consists entirely of sevens; the (a) 5 (b) 22
value of N is: (c) 35 (d) cant say
(a) 58829 (b) 123459
(c) 59829 (d) 56783 12. A number when divided by 5 gives a number
which is 8 more than the remainder obtained
5. How many numbers between 1 and 1000 are on dividing the same number by 34. The least
divisible by 7? such number is:
(a) 177 (b) 143 (a) 75 (b) 175
(c) 142 (d) 176 (c) 680 (d) None of these

6. A number of the form 10! 1 is always 13. When a natural number divided by a certain
divisible by 11 for every n is a natural number, divisor, we get 15 as a remainder. But when
where n is: the 10 times of the same number is divided by
(a) Odd number (b) Prime number the same divisor we get 6 as a remainder. The
(c) Even number (d) Cant say maximum possible number of such divisors is:
(a) 6 (b) 7 (c) 15 (d) 16
7. How many numbers are divisible by 3 in the set
of numbers {297, 298, 299, 300,..497}? 14. A certain number C when divided by ! it
(a) 66 (b) 67 leaves a remainder of 13 and when it is divided
(c) 68 (d) None of these by ! it leaves a remainder of 1, where ! and


27 | C A T C L A S S E S B Y A s h a n k D u b e y

! are the positive integers. Then the value of 0.abababab... is expressed as a fraction in
!! ! lowest terms. How many different
! + ! is, if = :
!! !
denominators are possible?
(a) 27 (b) 36 (c) 54 (d) cant say
(a) 3 (b) 4 (c) 5 (d) 6


15. The number 523abc is divisible by 7, 8 and 9.
23. The product of the ages of some teenagers is
Then a b c is equal to
10584000. The sum of their ages is equal to
(a) 10 (b) 180
(a) 85 (b) 86 (c) 87 (d) 88
(c) Either (a) or (b) (d) None of these


24. 53!" 27!" is certainly divisible by
16. How many four-digit numbers abcd with
(a) 7 (b) 9 (c) 10 (d) 11
distinct digits which is divisible by 4, such that

bacd is divisible by 7, acbd is divisible by 5, and
25. 43!!! + 34!!! is certainly divisible by
abdc is divisible by 9?
(a) 2 (b) 5 (c) 9 (d) 11
(a) 2 (b) 3 (c) 5 (d) 6


26. If S = 5!"!! + 11!"!! + 17!"!! where n is any
17. Sum of five consecutive integers is A. Find the
whole number, then S is always divisible by
sum of next five consecutive integers.
(a) 7 (b) 17 (c) 19 (d) 33
(a) A + 20 (b) A + 30

(c) A + 25 (d) None of these
27. Let M and N be single-digit integers. If the

product 2M5 13N is divisible by 36, how
18. Sum of four 2-digit consecutive odd integers
many ordered pairs (M, N) are possible?
when divided by 10 results in a perfect square.
(a) 2 (b) 3 (c) 4 (d) 8
How many such sets of four 2-digit numbers

are possible?
28. When a certain two digit number is added to
(a) 4 (b) 6 (c) 8 (d) 2
another two digit number having the same

digits in reverse order, the sum is a perfect
19. The number A4531B, where A and B are single-
square. How many such two digit numbers
digit numbers, is divisible by 72. Then A + B is
are there?
equal to
(a) 10 (b) 4 (c) 6 (d) 8
(a) 5 (b) 7 (c) 8 (d) 4


29. A three-digit number abc is divisible by 7 if
20. The remainder when 888222888222888222.
(a) 3a + b + c is divisible by 7
(9235 digits) is divided by 5! is
(b) a + 2b + c is divisible by 7
(a) 1 (b) 38 (c) 47 (d) 103
(c) 2a + 3b + c is divisible by 7

(d) 2a + 2b + c is divisible by 7
21. N the least positive integer that is eleven times

the sum of its digits. Then N is divisible by
30. A = {179, 180, 181, ., 360}. B is a subset of A
(a) 4 (b) 7 (c) 9 (d) 15
such that sum of no two elements of B is

divisible by 9. The number of elements in B
22. The single digits a and b are neither both nine
cannot exceed
nor both zero. The repeating decimal
(a) 102 (b) 81 (c) 82 (d) 101


28 | C A T C L A S S E S B Y A s h a n k D u b e y

FACTORS AND MULTIPLES OF A NUMBER

If one integer can be divided by another integer an exact number of times, then the first number is said
to be a multiple of the second, and the second number is said to be a factor of the first. e.g. 56 is
multiple of 8 because 8 goes into 56 an exact number of times (6 times in this case). Similarly, 8 is factor
of 56.

Factors: For a natural number N, all the numbers, including 1 and N itself, which divide N completely are
called factors or divisors of N.

Multiples: For a natural number N, all the numbers, which we will get after multiplying any natural
number in the given number, are called multiples of N.

Factorization: It is the process of splitting any number into the form, where it is expressed only in terms
of the most basic prime factors. e.g. 12 = 2! x 3. This is the factorized form of 12.

Number of factors of a given number

Let us assume a number, say 36, then find the number of factors.

36 = 1 x 36 = 2 x 18 = 3 x 12 = 4 x 9 = 6 x 6
We see that there are total 9 factors namely, 1, 2, 3, 4, 6, 9, 12, 18 and 36.
But for the larger numbers it becomes difficult to find total number of factors. So, lets try to understand
the concept behind it.
36 = 2! x 3!
Any factor of 36 will have powers of 2 equal to either 2! or 2! or 2! .
Similarly, any factor of 36 will have powers of 3 equal to either 3! or 3! or 3!

To make a divisor/factor of 36, we will have to choose a power of 2 and a power of 3. A power of 2 can
be chosen in 3 ways out of 2! or 2! or 2! . Similarly, a power of 3 can be chosen in 3 ways.

Therefore, the number of factors = 3 3 = 9.

Notice that we have added 1 each to the powers of 2 and 3 and multiplied.

! Let N be a composite number such that N = ( ) ( ) ( ) . Where , , . are


prime factors. Then, the number of factors of N = (a + 1)(b + 1)(c + 1).

Examples

1. Find the total number of factors of 360:


(a) 24 (b) 18 (c) 12 (d) 36

Solution: 360 = 2! x 3! x 5! .

Therefore number of factors = (3 + 1)(2 + 1)(1 + 1) = 24.


29 | C A T C L A S S E S B Y A s h a n k D u b e y

2. The total number of divisors of 1050 except 1 and itself is:
(a) 24 (b) 28 (c) 18 (d) 22
!
Solution: 1050 = 2 x 3 x 5 x 7
Therefore number of factors = (1 + 1)(1 + 1)(2 + 1)(1 + 1) = 24. But we have to exclude 1 and 1050. So
there are only 24 2 = 22 factors of 1050 except 1 and 1050.

Number of odd and even factors of a given number:

Let us assume a small number 90 then find the odd number of factors.

90 = 2 x 3! x 5 = 1 x 90 = 2 x 45 = 3 x 30 = 5 x 18 = 6 x 15 = 9 x 10
Thus there are only 6 odd factors namely 1, 3, 5, 9, 15 and 45 and 6 even factors namely 2, 6, 10, 18, 30
and 90. To get the number of odd or even factors of a number N first of all express the number N in
prime factors form.

Lets try to understand concept behind it. As we know, an odd number does not have a factor of 2 in it.
Therefore, we will consider all the factors having powers of 3 and 5 but not 2. Therefore, ignoring the
powers of 2, the number of odd factors = (2 + 1)(1 + 1) = 6.

Similarly, for even number of factors, we will consider all the factors having powers of 2, 3 and 5 but not
. Therefore, ignoring the 2! , the number of even factors = 1x(2 + 1)x(1 + 1) = 6.

! Let N be a composite number such that N = () ( ) ( ) .. where , .. are odd prime


factors. Then, the number of even factors of N = (a)(b + 1)(c + 1). and number of odd factors
of N = (b + 1)(c + 1).

Examples

1. How many divisors of 2160 are odd numbers?


(a) 12 (b) 6 (c) 8 (d) None of these
! !
Solution: 2160 = 2 x 3 x 5.
An odd number does not have a factor of 2 in it. Therefore, we will consider all the divisors having
powers of 3 but not 2. Therefore, ignoring the powers of 2, the number of odd divisors = (3 + 1) (1 + 1)
= 4 2 = 8.

2. Find the number of odd factors of 24.


(a) 1 (b) 2 (c) 8 (d) 3
Solution: 24 = 2! x 3
An odd number does not have a factor of 2 in it. Therefore, we will consider all the divisors having
powers of 3 and 5 but not 2. Therefore, ignoring the powers of 2, the number of odd divisors = (1 + 1) =
2.

3. How many factors of 2160 are even numbers?
(a) 40 (b) 8 (b) 5 (d) 32
Solution: 2! x 3! x 5.
Number of even factors of 2160 = (4)(3 + 1)(1 + 1) = 32.


30 | C A T C L A S S E S B Y A s h a n k D u b e y

4. Find the number of even factors of 24.
(a) 4 (b) 6 (c) 8 (d) None of these
!
Solution: 24 = 2 x 3
Number of even factors of 24 = (3)(1 + 1)= 6.

5. How many divisors of the number have unit digit equal to 5?
(a) 20 (b) 24 (c) 16 (d) 4
Solution: For unit digit equal to 5, the number has to be a multiple of 5 and it should not be a multiple of
2 otherwise the unit digit will be 0. To be a multiple of 5, the powers of 5 that it can have is 51, 52, 53 or
54. The powers of 3 can be 30, 31, 32, 33 or 34.
Therefore, the number of factors which have a unit digit of 5 = 4 5 = 20.

6. How many divisors of 360 are not divisors of 540?
(a) 6 (b) 18 (c) 24 (d) 12
Solution: The best option here is to find the number of common divisors of 360 and 540. For that we
find the highest common powers of all the common prime factors in 360 and 540.
Now, 360 = 23 32 5 and 540 = 22 33 5.
The number of common factors would be made by 22 32 5. The number of factors made by this = 3
3 2 = 18. Therefore, the two numbers will have 18 factors in common.
Number of factors of 360 = 4 3 2 = 24 Number of factors of 360 which are not factors of 540 = 24
18 = 6.

Reverse Operations on Factors:

1. Find the smallest number with 10 factors.
(a) 162 (b) 80 (c) (d) None of these
Solution: 10 = 2 5 = (1 + 1)(4 + 1)
The number is of the form ! ! x ! ! , where ! and ! are prime. To find the smallest such
number, we give the highest power to smallest prime factor, i.e. 2, and the next highest power to next
smallest prime number, i.e. 3, and so on.
Therefore, the smallest number = 24 31 = 48.

2. Find all the numbers less than 100 which have exactly 8 factors.
(a) 0 (b) 1 (c) 8 (d) 10
Solution: To find the number of factors of a number, we used to add 1 to powers of all the prime factors
and then multiply them together. Now, given the number of factors, we will express this number as a
product and then subtract 1 from every multiplicand to obtain the powers.
8 = 2 2 2 = (1 + 1) (1 + 1) (1 + 1).

Therefore, the number is of the form ! ! x ! ! x ! ! , where ! , ! and ! are prime. The numbers
can be 2 3 5 = 30, 2 3 7 = 42, 2 3 11 = 66, 2 3 13 = 78, 2 5 7 = 70.
8 = 4 2 = (3 + 1) (1 + 1). Therefore, the number is of the form ! ! x ! ! , where ! , ! and ! are
prime. The numbers can be 23 3 = 24, 23 5 = 40, 23 7 = 56, 23 11 = 88, 33 2 = 54.

The number can also be of the form ! ! , but there is no such number less than 100. So, there are 10
numbers less than 100 which have exactly 8 factors. Option (d)


31 | C A T C L A S S E S B Y A s h a n k D u b e y

3. The number of factors of every natural number from 1 to 1000 is calculated. Find the number of
factors of that natural number, which has highest number of factors?
(a) 32 (b) 48 (c) 40 (d) 21
Solution: The number less than 1000 which can incorporate highest number of prime factor is = 2 3 5
7 = 210. Now we are looking for highest multiple of 210 that is less than 1000. The multiple is 210 4 =
840 which has 32 divisors

Placement of factors:

As we know, its difficult to understand through variables. Lets understand by numbers


only. Lets consider some different numbers and write the factors in ascending order.

Factors of 16 = 1, 2, 4, 8 and 16 (5 factors)


Factors of 36 = 1, 2, 3, 4, 6, 9, 12, 18 and 36 (9 factors)
Factors of 45 = 1, 3, 5, 9, 15 and 45 (6 factors)

After observing the given pattern, we should get an idea that number of factors can be either
odd or even. If the number of factors is odd, we can find out the central position if all the factors are
written in ascending order. The central position will always be exactly at one of the factors. We already
have seen 36 has an odd number of factors (9 factors) and center will lie exactly at 6 which is at
5th position. If a number has 19 factors, the center position will be 10th factor.

! For perfect squares, which have odd number of factors, the center position will lie
exactly at the square root of that perfect square.

Examples

1. If factors of 100 are arranged in ascending order. What is the 5th factor of 100?
Solution: 100 = 22 52 = 9 factors.
Factors of 100: 1, 2, 4, 5, 10, 20, 25, 50 and 100, Since 100 have 9 factors, the center position will be
5th position and it is occupied by 10 which is the square root of 100.

2. If factors of 144 are arranged in ascending order, how many factors of 144 are more than the 8th factor
of 144?
(a) 5 (b) 6 (c) 7 (d) None of these
Solution: 144 is a perfect square. It has 15 factors. If we denote 144 by N, then 12 will be denoted by
. It will lie exactly at the center (8th position) of all the factors arranged in an ascending order. There
are 7 factors between 1 and N, and 7 factors are also present between and N. Option (c)

If number of factors is even, we can find out the central position if all the factors are written in
ascending order. The center position will always lie between two factors. For e.g. if there are 4 factors,
center position will always be between 2nd and 3rd factor. If a number has 8 factors, center position will
always be between 4th and 5th factor.
Factors of 12 = 1, 2, 3, 4, 6 and 12 (6 factors)
Factors of 24 = 1, 2, 3, 4, 6, 8, 12 and 24 (8 factors)


32 | C A T C L A S S E S B Y A s h a n k D u b e y

If we take 12 as N, will be lie between 3 and 4 which happens to be 3rd and 4th factor of 12. There are
two factors between 1 and , and 2 factors also lie between and N.

If we take 24 as N which has 8 factors, the center will lie between 4th and 5th factor which happens to be
4 and 6 in the increasing order sequence. There are 3 factors between 1 and , and there are exactly 3
factors between and N. (N = 24 = 4.4 approx.)

! If there are x number of factors between 1 and , then there will exist x number of factors
between and N, for N being a natural number.

Examples

1. If there are 20 factors between 1 and , find the total number of factors of N if it is given that N is a
natural number but is not a natural number.
Solution: As we know, is not a natural number, it should strike our mind that N is not a perfect
square. If there are 20 factors between 1 and , there would be exactly 20 factors between and N.
We also know that 1 and the number itself are factors of any natural number. So, total number of
factors of N will be = 20 + 20 + 2 = 42 factors.

Note: If N is a perfect square in the above question, answer will be 43. Because would also have
been part of the group of factors.
The number of factors existing between 1 and , and between and N is the same because factors
of all natural numbers exist in pairs. Like 12 can be written as product of two natural numbers in
following way: 12 = 1 12
12 = 2 6
12 = 3 4

These are the three ways in which 12 can be written as product of two natural numbers. But what are 1,
12, 3, 4, 2 and 6. They are factors of 12. And (1 x 12), (2 x 6) and (3 x 4) give us 12. Because (1, 12) are
factors of 12 which are situated at equal distance from the center when all the factors of 12 are written
in ascending order.
1st 2nd 3rd 4th 5th 6th
Factors of 12 1 2 3 4 6 12

For factors of 12, center will lie between 3 and 4. If we take one factor to the left of the center and one
factor to the right of center, we will get factors 3 and 4. When we multiply them we will get 12. Similarly
when we move 2 places to the left of the center and 2 places to the right, we get factors 2 and 6, when
we multiply them we get 12. And finally when we move 3 places to the left of the center and 3 places to
the right of center, we get 1 and 12. When we multiply them, we will again get 12. This pattern will be
found in all the natural numbers.

! Factors of any number which are equidistant from the center, when multiplied with each
other will always result in that particular number.


33 | C A T C L A S S E S B Y A s h a n k D u b e y

Examples

1. If the product of factors of a natural number N at 12th position and 25th position results in N, then find
the total number of factors of N.
(a) 37 (b) 30 (c) 34 (d) 36
Solution: If factor at 12th position multiplied with factor at 25th position results in N, then factor at
11th position multiplied with 26th position will also result in N. That should be sufficient to find the
answer. There would be 11 factors on the left hand side of the 12th factor, and since factors exist in pairs
and factors equidistant from center always result in number, there would be 11 factors on the right of
factor at 25th position, so N has total 36 factors. Once you got hold of logic, these problems can be
solved in a flash mentally!

Note: We can observe something more. Add up the positions and see, you will get a pattern.
12th position + 25th position = 37. (While multiplying factors at 12th & 25th position gives us N)
11th position + 26th position = 37. (While multiplying factors at 11th & 26th position gives us N)
10th position + 27th position = 37. (While multiplying factors at 10th & 27th position gives us N)
9th position + 28th position = 37...(While multiplying factors at 9th & 28th position gives us N)

All the factors which are at these positions when multiplied with each other will result in original
number and the sum will always be constant which is 1 greater than total number of factors. So, we
could have solved the above problem by just adding 12 + 25 = 37 and subtracting 1 from it, giving us 36
total factors.

2. Which factor will occupy the 68th position if all the factors of 62 52 142 are written in ascending
order?
(a) 420 (b) 210 (c) 160 (d) None of these
Solution: We just need to find the number of factors of given number and then we can think about
applying logic of placement of factors.
62 52 142 = 24 32 52 72
Number of factors = (4 + 1)(2 + 1)(2 + 1)(2 + 1) = 5 3 3 3 = 135 factors. If the number of factors of a
number is odd, then that number has to be a perfect square. It means that the center position when all
the factors are written in ascending order will be (135 + 1)/ 2 = 68th position. That is what the question
is asking. And we know, in case of perfect square, center position always belongs to the square-root of
that number. So, answer is square root of 62 52 142 = 6 5 14 = 420.

Sum of factors of a given number:


Let us assume a number, say 240, then find the sum of factors.
240 = 2! x 3! x 5!
The sum of factors will be given by:
(2! + 2! + 2! + 2! + 2! ) (3! + 3! )( 5! +5! ) = 31 x 4 x 6 = 744

Lets try to understand the concept behind it.


Let N be a composite number such that N = (! )! (! )! (! )! ..
Then the sum of all the factors of N
= (! )! + (! )! + + (! )! (! )! + (! )! + + (! )! (! )! + (! )! + + (! )!


34 | C A T C L A S S E S B Y A s h a n k D u b e y

In the above expression, There are 3 G.P. with common ratio ! , ! and ! respectively. We need to find
summation of the given G.P.

!! (!!!) !! !! (!!!) !! !! (!!!) !!
= x x x .
(!! !!) (!! !!) (!! !!)

! Let N be a composite number such that N = ( ) ( ) ( ) .. where , , .. are
(!) ! (!) ! (!) !
prime factors. Then, the sum of factors of N = x x x .
( !) ( !) ( !)

Examples

1. What is the sum of factors of 1200?
(a) 3,844 (b) 3,600 (c)30 (d)None of these
Solution: 1200 = 2! x 3 x 5! .
!! !! !! !! !! !! !" ! !"#
Then, the sum of factors of 1200 =
!!!
x !!!
x !!!
= ! !
= 3,844.

2. What is the sum of factors of 1200 such that the factors are divisible by 15?
(a) 1,395 (b) 2,790 (c) 3648 (d)
Solution: 1200 = 2! x 3 x 5! .
If factors are divisible by 15, it should have 3! and 5! in it. Thus, sum of factors divisible by 15 = 2! +
2! + 2! + 2! + 2! 3! 5! + 5! = 2,790

3. Find the sum of even factors of x x x .
Solution: All the even divisors of the number will have powers of 2 equal to one of 2, 22, or 23.
Therefore, sum of even divisors = (2 + 22 + 23) (1 + 3 + 32 + 33 + 34) (1 + 5 + 52) x (1 + 7 + 72 + 73)
! !! !! !! !! !! !! !! !!
!!!
x !!! x !!!
x !!! = 21,005,600

4. Find the sum of odd factors of x x x .
Solution: An odd number does not have a factor of 2 in it. Therefore, we will consider all the factors
having powers of 3, 5 and 7 but not 2. Therefore, ignoring the powers of 2, sum of odd factors = (1 + 3 +
!! !! !! !! !! !!
32 + 33 + 34) (1 + 5 + 52) x (1 + 7 + 72 + 73) = x x = 1,500,400.
!!! !!! !!!

Product of factors
Let us assume a small number 24 and see the factors
24 = 1 x 24 = 2 x 12 = 3 x 8 = 4 x 6
Now, it is obvious from the above explanation that the product of factors of 24 = (1 x 24)(2 x 12)(3 x 8)(4
x 6) = {24 x24x 24x 24} = 24! = 24(!"#$%& !" !"#$%&')/!

! Let N be a composite number such that N = ( ) ( ) ( ) .. where , , .. are
prime factors. Then, the product of factors of N = / , where is the total number of factors
of N.



35 | C A T C L A S S E S B Y A s h a n k D u b e y

Examples

1. What is the product of factors of 3600?
Solution: 3600 = 2! x 3! x 5!
Therefore, number of factors = 5 x 3 x 3 = 45
Thus, the product of factors = 2! x 3! x 5! !"/! = 2! x 3! x 5! !"
= 60 !"


2. Find the product of factors of 560?
Solution: 560 = 2! x 5 x 7
Therefore, number of factors = 5 x 2 x 2 = 20
!"
!"
Thus, the product of factors = 560 ! = 560

Number of ways of expressing a composite number as a product of two factors

Let us consider an example of a small composite number 90.
Then 90 = 1 x 90 = 2 x 45 = 3 x 30 = 5 x 18 = 6 x 15 = 9 x 10
So it is clear that the number of ways of expressing a composite number as a product of two
! !
factors = ! x the number of total factors of 90 = ! x 12 = 6

! Let N be a composite number such that N = ( ) ( ) ( ) .. where , , .. are
prime factors. If N is not a perfect square, Then, the number of ways N can be written as a
! ! ( !)
product of two numbers = x the number of total factors of N =


Examples

1. Find the number of ways of expressing 180 as a product of two factors.
(a) 8 (b) 9 (c) 18 (d) None of these
Solution: 180 = 2! x 3! x 5
Number of factors = (2 + 1)(2 + 1)(1 + 1) = 18
!"
Hence, there are total = 9 ways in which 180 can be expressed as a product of two factors.
!

2. Find the number of ways of expressing 60 as a product of two factors.
(a) 6 (b) 5 (c) 7 (d) 30
Solution: The factors of 60 are 1, 2, 3, 4, 5, 6, 10, 12, 15, 20, 30, and 60. Now,
60 = 1 60 = 2 30 = 3 20 = 4 15 = 5 12 = 6 10.
!"
Hence, there are total = 6 ways in which 60 can be expressed as a product of two factors.
!

Note of caution: In perfect squares, factors occur in pairs except for the square root for numbers which
are perfect squares. If we express any perfect square N as a product of two factors namely and ,
and you also know that since in this case appears two times but it is considered only once while
calculating the number of factors so we cannot divide the odd number exactly by 2. So if we have to
consider these two same factors then we find the number of ways of expressing N as a product of two
(!"#$%& !" !"#$%&' ! !)
factors = .
!


36 | C A T C L A S S E S B Y A s h a n k D u b e y

If N is a perfect square, then,


( ! )
! The number of ways N can be written as a product of two numbers =

! The number of ways N can be written as a product of two distinct numbers =
( ! )



Examples

1. Find the number of ways expressing 36 as a product of two factors.
(a) 4 (b) 3 (c) 6 (d) 5
Solution: 36 = 2! x 3!
Number of factors = (2 + 1)(2 + 1) = 9
!!!
Hence the number of ways expressing 36 as a product of two factors = = 5.
!

2. In how many ways can 225 be expressed as the product of two distinct factors?
(a) 5 (b) 4 (c) 3 (d) 6
! !
Solution: 225 = 3 x 5
Total number of factors = (2 + 1)(2 + 1) = 9
(!!!)
So the number of ways expressing 225 as a product of two distinct prime factors = = 4
!
Note: Since the word distinct has been used therefore we do not include 225 = 15 x 15

3. How many ordered pairs of integers, (a, b) satisfy the equation ab = 110?
(a) 9 (b) 8 (c) 18 (d) 16
Solution: 110 = 2 5 11. Hence, the number of divisors of 110 is = 2 2 2 = 8. Hence, the number of
positive ordered pairs of x and y = 8 [as (2, 55) is not the same as (55, 2)]. Also, since we are asked for
integers, the pair consisting of two negative integers will also suffice. Hence the total number of ordered
pairs = 2 8 = 16.

! A perfect square has odd number of factors. In other words, any number which has odd
number of factors is a perfect square.
! The squares of prime numbers have exactly three factors.

Examples

1. There are 100 doors in a row that are all initially closed. You make 100 passes by the doors
starting with the first door every time. The first time through, you visit every door and toggle the
door (if the door is closed, you open it, if its open, you close it). The second time you only visit every
2nd door (door #2, #4, #6). The third time, every 3rd door (doors #3, #6, #9),.. etc, until you only visit
the 100th door. How many doors will remain toggled (open became closed & closed became open)
after the last pass?
(a) 10 (b) 90 (c) 25 (d) None of these
Solution: A very interesting question indeed! Where to start?
The number of times a door will be toggled is based on the number of divisors the locker number has.
For example, door #6 will be toggled on pass 1, 2, 3 and 6. Further note that most numbers have an
even number of divisors. This makes sense since each divisor must have a matching one to make a pair
to yield the product. For example, 1x6=6, 2x3=6.


37 | C A T C L A S S E S B Y A s h a n k D u b e y

The only numbers that do not have an even number of divisors are the perfect square numbers, since
one of their divisors is paired with itself. For example, door #9 is toggled an odd number of times on
passes 1, 3 and 9 since 1x9=9 and 3x3=9.
Thus, all non-square numbered lockers will end up closed and all square numbered lockers will end up
open. Number of perfect squares up to 100 = 10.

2. How many numbers are less than 1000 have exactly three factors?
(a) 25 (b) 10 (c) 9 (d) None of these
Solution: As we know, only squares of prime numbers have exactly three factors. So, we need to find
out prime numbers squares up to 1000.
312 = 961.
Number of prime numbers up to 31 = 11.
There are 11 numbers up to 100 which have exactly three factors.

! The number of ways in which a composite number can be resolved into two factors which are
prime to each other = 2(n 1), where n is the number of different prime factors of the number.

Examples

1. The number of ways of factorizing 210 320 53 74 into two factors, a and b, such that gcd(a,b) = 1 is
(a) 16 (b) 8 (c)12 (d) None of these
Solution:
Method I - We have to assign these prime factors and their powers to one of the two factors. As the two
factors will be prime to each other, we will have to assign a prime factor with its power (for example 210)
completely to one of the factors. For every prime factor, we have two ways of assigning it. Therefore,
the total number of ways = 2 2 2 2 = 16. As we are not looking for ordered pairs, the required
!"
number of ways = = 8
!
Method II Just use the above formula. Total number of such ways = 2(4 1) = 8

2. The number of ways of factorizing 91,000 into two factors, and b, such that > 1, b > 1 and gcd(,b)
= 1 is
(a) 7 (b) 15 (c) 32 (d) None of these
! !
Solution: 91,000 = 2 x 5 x 7 x 13
As the two factors will be prime to each other, we will have to assign a prime factor with its power (for
example 23) completely to one of the factors. For every prime factor, we have two ways of assigning it.
Therefore, the total number of ways = 2 2 2 2 = 16. As we are not looking for ordered pairs, the
!"
required number of ways = = 8. But, in this case we cannot assign all prime factors to one number and
!
1 (because > 1, b > 1) to other.
So, the required answer is = {2(n 1) 1} = {8 1} = 7

Writing a natural number as difference of squares of two natural numbers:

Let p and q be two natural numbers. Then ! ! = (p + q)(p q). If (p + q) and (p q) are odd, (p+
q)(p q) is also odd. Hence any odd number can be expressed as the difference of two perfect squares.


38 | C A T C L A S S E S B Y A s h a n k D u b e y

If (p + q) and (p q) are even, then (p + q)(p q) is a multiple of 4. Hence all multiples of 4 can be
expressed as difference of two perfect squares

+ Number + =
Odd Even Odd Odd Odd x Odd
Odd Odd Even Even Even x Even
Even Odd Odd Odd Odd x Odd
Even Even Even Even Even x Even

If N is divisible by 2 but not by 4, then one of (p + q) and (p q) is always even and other is odd. This
results in fractional values of p and q.
So, the numbers which are divisible by 2 but not by 4 cannot be written as a difference of two perfect
squares. [38 = 19 x 2. Take (p + q) = 19 and p q = 2. We get p = 10.5 and q = 8.5]

! All (4k + 2) numbers cannot be expressed as the difference of two perfect squares.

Type 1 When the natural number is odd



Examples

1. In how many ways can 45 be written as the difference of squares of two natural numbers?
(a) 2 (b) 3 (c) 4 (d) 5
2 2
Solution: (x y ) = 45, where x and y are natural numbers.
(x + y) (x y) = 45, where (x + y) will always be greater than (x y), since x and y are natural numbers.
Now,
(x + y) (x y) = 1 x 45
(x + y) (x y) = 3 x 15
(x + y) (x y) = 5 x 9

Now since, (x + y) is greater than (x y), (x + y) will always correspond to the larger number which is 45
and (x y) will correspond to the smaller number which is 1.

Case 1: If we solve x + y = 45 and x y =1, will we get natural number solutions for x and y. Answer is
yes, we will get x = 23 and y = 22 which are natural numbers.
Case 2: If we take x + y = 15 and x y = 3, on solving we get x = 9 and y = 6.
Case 3: If we take x + y = 9 and x y = 5, on solving we get x = 7 and y = 2.

So, all the three ways are working since we are getting natural number values for x and y. So, in case of
any odd number, all the ways will give us solution. We just need to find out number of factors of that
odd number and divide it by 2 to get the required number of ways.

So, we could have just found out the number of factors of 45 and divided it by 2, we would have arrived
!
at the answer. Number of factors of 45 = 6. The required answer = = 3 ways.
!


39 | C A T C L A S S E S B Y A s h a n k D u b e y

2. In how many ways can 315 be written as the difference of squares of two natural numbers?
(a) 12 (b) 6 (c) 5 (d) None of these
Solution: Again 315 is an odd number, so we will find out number of factors of 315 = 32 51 71 = 12
factors. If 315 have 12 factors, which mean we can write 315 as difference of squares of two natural
numbers in 6 ways.

Type 2 When the natural number is even

1. In how many ways can 60 be written as the difference of squares of two natural numbers?
(a) 1 (b) 2 (c) 3 (d) 4
Solution: (x2 y2) =60, where x and y are natural numbers.
(x + y) (x y) = 60, where (x + y) will always be greater than (x y), since x and y are natural numbers.
Now,
(x + y) (x y) = 1 x 60
(x + y) (x y) = 2 x 30
(x + y) (x y) = 3 x 20
(x + y) (x y) = 4 x 15
(x + y) (x y) = 3 x 20
(x + y) (x y) = 5 x 12
(x + y) (x y) = 6 x 10

Case 1: First way, (x + y) = 60 and (x y) = 1 is not valid because when we solve them simultaneously,
values of x and y will not be natural numbers.
Case 2: (x + y) = 30 and (x y) = 2 is valid since summation of 30 and 2 is even and on solving we will
get x = 16 and y = 14.

So, we just need to check which pair gives us even x even form; those pairs will only result in natural
numbers. Only two pairs (2 x 30) and (6 x 10) give us even x even form; only these pairs will give us
required solutions. So, answer is 2 ways.

Note: We can write (x + y) (x y) = even even. This means that both (x + y) and (x y) has to be even.
The standard form of representing an even number is 2n where n is a whole number. So, we can assume
(x + y) as 2n and (x y) as 2m where n > m as (x + y) will always be greater than (x y), since x and y are
natural numbers.
So, our equation can be rewritten as (2n) (2m) = 60. On solving, it reduces to nm = 15. should be 2 ways
as number of factors of 15 are 4 and since we are finding out number of ways of writing 15 as the
product of two natural number, we will be needed to divide the number of factors by 2, i.e. the answer
is 2 ways.

2. In how many ways can 80 be written as the difference of squares of two natural numbers?
(a) 5 (b) 4 (c) 3 (d) 2
Solution: 80 is an even number, so divide it by 4. We get 20 and then find out the number of factors of
20. 20 have 6 factors. If a number has 6 factors, it can be written in 3 ways. So, answer is 3 ways.


40 | C A T C L A S S E S B Y A s h a n k D u b e y

3. The number of solutions of the equations = 1614 + , where both m and n are integers, is:
(a) 1 (b) 2 (c) 5 (d) 0
2 2
Solution: m n = 1614 in how many ways.
(m + n) (m n) = 1 1614 = 2 807 = 6 269.
Only those pairs will work where both of them are even. In this case none of the pairs are in the form of
even x even, in all the pairs one of them is even and another one is odd. So, answer should be 0 ways or
no way.
Note: As we have seen previously, any number of the form 4n+2 cannot be expressed as a difference of
the squares of two natural numbers. 1614 happens to be of the same form.

Integral Solutions

Such problems fall in the domain of algebra, but our understanding of factors can help us solve them in
quicker time. We will learn the standard way of solving; after that you can learn the short-cut through
observation.

Examples


1. Find the natural number solution of + =

! ! !
Solution: + = 10a + 10b = ab ab 10a 10b = 0.
! ! !"
We will add 100, {multiply coefficient of a which is ( 10) with that of b which also is ( 10)}, on both
sides so that the expression gets factorized easily.
10a + 10b = ab ab 10a 10b + 100 = 100.
On factorizing, it becomes (a 10) (b 10) = 100.
Now, we need to find out, how many ways 100 can be written as a product of two natural numbers.

(a 10) (b 10) = 100, find out the number of factors of 100 which comes to 9 (How? 100 = 2252
number of factors = 3x3). If a number has 9 factors, 5 pairs can be made.
(a 10) (b 10) = (1 100) = (2 50) = (4 25) = (5 20) = (10 10).

Since we are talking about natural number solutions, every pair will give us two natural number
solutions except the last pair. We have already seen in case of a perfect square, the square root of that
number multiplied with itself results in the number. Since both numbers are identical, we will just get
one solution and not two.

If (a 10) = 1, then (b 10) = 100. On solving them a and b will be natural numbers. But, it could be
other way round also that (a 10) takes 100 and (b 10) = 1. Similarly, we will get 2 natural number
solutions each for remaining three pairs.

But what about the last pair? If (a 10) = 10, then (b 10) = 10. Even if we take the value other way
around, we would not get anything new. So, answer is 9 natural number solutions.




41 | C A T C L A S S E S B Y A s h a n k D u b e y

What if the question asked us to find the integral solutions to the same question?

`Still the logic will remain unchanged; but now every pair will give us 4 integral solutions except the last
pair. If (a 10) = 1 then (b 10) = 100. Also, if (a 10) takes 100 and (b 10) = 1. And If (a 10) = 1,
then (b 10) = 100. On solving this also, we will get integral values for a and b. And the last one will be
If (a 10) = 100, then (b 10) = 1.

So, four pairs will give us 4 integral solutions each. So, we got 4 4 integral solutions up till now. What
about the last pair, will it give us two integral solutions or just one? Lets find out.

If (a 10) = 10, then (b 10) = 10. This will be the first one. If we take the second part which is If (a 10)
= 10, then (b 10) = 10. On solving this, we get we get a = 0 and b = 0. This value is not acceptable as
despite 0 being an integer. Because, if we substitute the value of a and b as 0, the expression will
become undefined. So, final answer is 17 integral solutions.

Note: In all such problems asking for integral solutions, there will be one pair which on solving gives us
the value of a and b as 0; so, we have to be cautious there.


2. How many natural and integral number of solution of + = possible?

Solution: We can simplify it immediately as it of the same pattern as (a 12) (b 12) = 144 as 12 will be
multiplied with 12 to make 144.
Again if we are looking for natural number solution, we will find out number of factors of 144 which is
15. If a number has 15 factors, we can make 8 pairs and every pair will give us two natural number
solutions except the last pair which will only result into one natural number solution. So, answer is 7 2
+ 1 = 15 natural number solutions.

For Integral solutions: We have to be careful, since one pair will result in values of a and b as 0 which
will not be acceptable. And every pair will give us 4 integral solutions and the last pair which will be 12
12 will just one integral solution. So, answer is 4 7 + 1 =29 integral solutions.


3. Find the number of integral solutions to + =

(a) 23 (b) 24 (c) 12 (d) 11

Solution: 10b + 15a = ab; or ab 15a 10b = 0. The coefficient of a and b are ( 15) and (10)
respectively. Multiply them and add on both sides, we get

ab 15a 10b + 150 = 150. On factorizing, we get (a 10) (b 15) = 150.

Now, if we are looking for integral number of solutions, we will find out the number of factors of 150
which is 12. If a number has 12 factors, we can form 6 pairs or write them into 6 ways.

So, 6 pairs are there, every pair will give us 4 integral solutions, except one pair which will give us just 3
integral solutions. The pair which contains 10 and 15 will give us 3 integral solutions.


42 | C A T C L A S S E S B Y A s h a n k D u b e y

If (a 10) = 10, then (b 15) = 15; will give integral solutions for a and b.
If (a 10) = 15, then (b 15) = 10; will give integral solutions for a and b.
If (a 10) = 10, then (b 15) = 15; not acceptable since a= 0 and b = 0.
And if (a 10) = 15, then (b 15) = 10; will give integral solutions for a and b.
So, answer is 6 4 1 = 23 integral solutions.

The number of ways a number can be written as sum of two or more consecutive numbers
Lets understand this interesting concept through examples:

Examples

1. In how many ways can 15 be written as the sum of two or more consecutive positive integers?
(a) 0 (b) 1 (c) 2 (d) 3
Solution: 15 = 7 + 8
15 = 4 + 5 + 6
15 = 1 + 2 + 3 + 4 + 5. Therefore, the number of ways = 3
Now 15 = 3 x 5
Number of odd factors = 4 (How? There is no 2 here, therefore all the factors are odd factors = 2 x 2)
The number of ways 15 can be written as sum of two or more consecutive positive integers
= (Number of odd factors 1)

2. In how many ways can 90 be written as sum of two or more consecutive positive integers?
(a) 2 (b) 3 (c) 4 (d) 5
Solution: Consecutive positive integers are an A.P. with common difference of 1.
!
Sum of an A.P. ! = 2 + 1 , where d = 1 and n is number of terms.
!
!
2 + 1 1 = 90
!
2 + 1 = 180.
Now, if n is even then 2 + 1 will be odd and if n is odd then 2 + 1 will be even. So,
give number (180) can be written in the form of {odd x even}.

2 + 1 = 1 x 180
= 3 x 60
= 5 x 36
= 9 x 20
= 15 x 16
= 45 x 4
There are 6 values are possible for n. But we shall not consider n = 1. Because, we are looking for sum of
two or more consecutive positive integers.

! The number of ways a number N can be written as sum of two or more consecutive positive
integers = {Number of odd factors of N 1}





43 | C A T C L A S S E S B Y A s h a n k D u b e y

Exercise 1.2

1. How many factors of 36!" are perfect cubes? that N is a natural number but N is not a
(a) 624 (b) 561 natural number.
(c) 642 (d) None of these (a) 15 (b) 28

(c) 30 (d) 29
2. How many divisors of 21600 are perfect

squares?
9. If all the factors of 800 are written in ascending
(a) 12 (b) 2
order, find which factor will occupy the 13th
(c) 6 (d) None of these
position?

(a) 100 (b) 160
3. A positive integer is ASHANK if it has 8 positive
(c) 10 (d) 80
factors that sum up to 3240. For example, 2006

is ASHANK because its 8 positive factors, 1, 2,
10. Find the number of ways in which 6! can be
17, 34, 59, 118, 1003 and 2006, sum up to 3240.
written as the product of two natural numbers.
Find the smallest positive ASHANK number.
(a) 14 (b) 15
(a) 657 (b) 256
(c) 30 (d) 16
(c) 3024 (d) None of these


11. Find the number of solutions to a b = 24,
4. Let A = set of all divisors of 8100 and B = set of
given that a and b are natural numbers.
all divisors of 21600. What is the product of the
(a) 4 (b) 8
elements of A U B?
(c) 2 (d) 16
(a) 2!"# x 3!"" x 5!" (b) 2!"" x 3!"" x 5!"

(c) 2!"" x 3!"" x 5!" (d) 2!"# x 3!"" x 5!"
12. Find the number of solutions in which 120 can

be written as the product of two integers.
5. Find the number of solutions to a b = 196,
(a) 8 (b) 16
where a and b are distinct integers.
(c) 14 (d) 32
(a) 16 (b) 18

(c) 20 (d) 5
13. In how many ways can 144 be written as

product of two natural numbers?
6. Find the number of ways in which 6! can be
(a) 4 (b) 6
written as the product of two integers
(c) 7 (d) 8
(a) 15 (b) 30

(c) 6 (d) None of these
14. Find the number of solutions for a b = 144,

where a and b are integers.
7. N is the smallest number that has 5 factors.
(a) 15 (b) 30
How many factors does (N 1) have?
(c) 16 (d) 31
(a) 2 (b) 3

(c) 4 (d) 5
15. In how many ways can 50 be written as the

difference of squares of two natural numbers?
8. If there are 14 factors between 1 and N, find
(a) 0 (b) 2
the total number of factors of N. If it is given
(c) 1 (d) 3


44 | C A T C L A S S E S B Y A s h a n k D u b e y

16. In how many ways can 80 be written as the (a) 1 (b) 2
difference of squares of two natural numbers? (c) 3 (d) 4
(a) 3 (b) 2
(c) 4 (d) 6 24. How many factors of 10!" end with a zero?
(a) 21 (b) 90
17. In how many ways can 960 be written as the (c) 100 (d) 121
difference of squares of two natural numbers?
(a) 10 (b) 20 25. How many perfect squares are the divisors of
(c) 5 (d) None of these the product 1! 2! 3!8!?
(a) 120 (b) 240
18. How many integers between 1 and 1000, both (c) 360 (d) 720
inclusive, can be expressed as the difference of
the squares of two non - negative integers? 26. Let M be the greatest number divisible by 8,
(a) 748 (b) 300 such that no digit from 0 to 9 is repeated in M.
(c) 750 (d) 250 What is the remainder when M is divided by
1000?
19. Let N = 2!" 3!" . How many factors of ! are (a) 320 (b) 120
less than N but do not divide N completely? (c) 104 (d) 96
(a) 387 (b) 208
(c) 310 (d) 180 27. If the number 2718AB6 is divisible by 72 (A and
B are single digits), then the number of
! ! ! possible ordered pairs (A, B) is
20. Find the integral solution of =
! ! !
(a) 2 (b) 4
(a) 21 (b) 20
(c) 6 (d) None of these
(c) 23 (d) None of these


!"!
!
21. Find the natural number solutions to + =
! ! 28. The smallest natural number n such that is
!
! ! !
(a) 10 (b) 20 a perfect square is
(c) 18 (d) 16 (a) 110 (b) 143
(c) 1430 (d) None of these
22. N is the smallest natural number which when
multiplied by 7 gives a product P. Every digit of 29. Find the sum of even divisors of 96 and odd
P is one. The product when N is multiplied by 8 divisors of 3600.
is (a) 589 (b) 639
(a) 127 784 (b) 125 384 (c) 651 (d) 735
(c) 126 984 (d) 181 384
30. Find the sum of divisors of 544 which are
23. The digits 1, 2, 3, 4, and 5 are each used once to perfect square.
compose a five-digit number abcde such that (a) 21 (b) 32
the three-digit number abc is divisible by 4, bcd (c) 42 (d) 64
is divisble by 5, and cde is divisble by 3. Find the
digit a.


45 | C A T C L A S S E S B Y A s h a n k D u b e y

HCF AND LCM

We have discussed in detail about the factors. Now we will move towards Common Factor, Highest
Common Factor (HCF) and Least Common Factor (LCM).

Highest Common Factor (HCF)

HCF stands for Highest Common Factor. It is also known as GCD (Greatest Common Divisor). This is the
largest number that can divide the given set of numbers. In other words, HCF of two or more than two
numbers is the greatest possible number that can divide all these numbers exactly, without leaving any
remainder.

Find the HCF of 36 and 60.

36 = 2 x 2 x 3 x 3 = 12 x 3
60 = 2 x 2 x 3 x 5 = 12 x 5
So, the HCF of 36 and 60 = 2 x 2 x 3 = 12

Thus we can see that 12 is the greatest common divisor since it can divide exactly both 36 and 60.

Basically there are two methods of finding the HCF:
(i) Factor Method (ii) Division Method

(i) Factor Method: In this method, first we break the numbers into prime factors then take the product
of all the common factors. This obtained resultant product is known as the HCF of the given numbers.

Examples

1. Find the HCF of 840 and 1260.
Solution:
840 = 2 x 2 x 2 x 3 x 5 x 7
1260 = 2 x 2 x 3 x 3 x 5 x 7
So the product of common factors = 2 x 2 x 3 x 5 x 7 = 420

2. Find the HCF of 750, 6300 and 18900.
Solution:
750 = 2 x 3 x 5 x 5 x 5
6300 = 2 x 2 x3 x 3 x 5 x 5 x 7
18900 = 2 x 2 x 3 x 3 x 3 x 5 x 5 x 7
So the product of common factors = 2 x 3 x 5 x5 = 150.
Hence 150 is the HCF of 750, 6300 and 18900.

(ii) HCF by Division Method: Consider two smallest numbers then divide the larger one of them by the
smaller one and then divide this divisor by the remainder and again divide this remainder by the next
remainder and so on until the remainder is 0.


46 | C A T C L A S S E S B Y A s h a n k D u b e y

Examples

1. Find the HCF of 420 and 1782.
Solution:


Hence, the last divisor 6 is the HCF of 420 and 1782.
If you watch carefully the above example you will find that the HCF is the factor of difference of the
given numbers. So there is another way to find HCF.
You can divide the given number by their lowest possible difference if these numbers are divisible by
this difference then this difference itself is the HCF of the given number. If this difference is not HCF
then any factor of this difference must be HCF of the given number.

2. Find the HCF of 42 and 105.
Solution: The difference = 105 42 = 63.
Now divide 42 and 105 by 63 but none of these is divisible by their difference 63. So we factorize 63 and
then divide 42 and 105 by the factor of 63. The greatest factor of 63 which can divide 42 and 105 will be
the HCF of the given numbers.
So 63 = 1 x 63, 3 x 21, 7 x 9
The factors of 63 are 1, 3, 7, 9, 21 and 63.
Obviously we divide 42 and 105 by 21 (since the division by 63 is not possible) and we see that the given
numbers are divisible hence 21 is the HCF of 42 and 105.

3. Three company of soldiers containing 120, 192, and 144 soldiers are to be broken down into smaller
groups such that each group contains soldiers from one company only and all the groups have equal
number of soldiers. What is the least number of total groups formed?
(a) 19 (b) 14 (c) 24 (d) 13
Solution: The least number of groups will be formed when each group has number of soldiers equal to
the HCF. The HCF of 120, 192 and 144 is 24. Therefore, the numbers of groups formed for the three
companies will be 5, 8, and 6, respectively. Therefore, the least number of total groups formed = 5 + 8 +
6 = 19.

4. The HCF of two numbers is 12 and their sum is 288. How many pairs of such numbers are possible?
(a) 4 (b) 3 (c) 8 (d) 6
Solution: If the HCF if 12, the numbers can be written as 12x and 12y, where x and y are co-prime to
each other. Therefore, 12x + 12y = 288 x + y = 24.
The pair of numbers that are co-prime to each other and sum up to 24 are (1, 23), (5, 19), (7, 17) and
(11, 13). Hence, only four pairs of such numbers are possible. The numbers are (12, 276), (60, 228), (84,
204) and (132, 156).


47 | C A T C L A S S E S B Y A s h a n k D u b e y

5. The HCF of two numbers is 12 and their product is 31104. How many such numbers are possible?
(a) 1 (b) 2 (c) 3 (d) 4
Solution: Let the numbers be 12x and 12y, where x and y are co-prime to each other. Therefore, 12x
12y = 31104 xy = 216. Now we need to find co-prime pairs whose product is 216.
216 = 23 33. Therefore, the co-prime pairs will be (1, 216) and (8, 27). Therefore, only two such
numbers are possible.

6. Find the HCF of ( ) and ( )
(a) 2 (b) 5 (c) 31 (d) None of these
Solution: (2 !"#
1) = { 2 ! !"
1} divisible by (25 1). (an bn is always divisible by a b)
Similarly, (2!" 1) = { 2! ! 1} divisible by (25 1). (an bn is always divisible by a b)
5
(2 1) = 2 !"# !" !"#,!" 1 = 31

Least Common Multiple (LCM)
LCM stands for Least Common Multiple. In other words, LCM is the lowest number divisible by all the
numbers taken. This is also the first number that comes common in the list of multiples of all the
numbers taken.
Find the LCM of 10 and 25.
1st few multiples of 10 = 10, 20, 30, 40, 50, 60,.
1st few multiples of 25 = 25, 50, 75, 100,
Now, we can see 1st number common in the list of multiples of 10 and 25 = 50. Hence, 50 is the LCM of
10 and 25.

Basically, there are two methods to find the LCM.
(i) Factor Method (ii) Division Method

(i) Factor Method: Resolve the given numbers into their prime factors, and then we find the highest
index, among the given numbers, of all the prime numbers. The LCM is the product of all these prime
numbers with their respective highest indices.

Examples

1. Find the LCM of 72, 288 and 540.
Solution: 144 = 24 32, 288 = 25 32, 1080 = 22 33 5. The prime numbers present are 2, 3 and 5. The
highest indices (powers) of 2, 3 and 5 are 5, 3 and 1, respectively.
Hence the LCM = 25 33 5 = 4320.

2. Find the LCM of 72x3y2 and 48x4y.
Solution: 72x3y2 = 2332x3y2 and 48x4y = 243x4y. The highest indices of 2, 3, x and y are 4, 2, 4 and 2
respectively.
Hence, the LCM = 2432x4y2 = 144x4y2.

(ii) Division Method: Write down all the numbers in a line then divide these numbers by the least
common prime factors say 2, 3, 5, 7, 11..to the given numbers, then write the quotients just below the
actual numbers. If any number is not divisible by such a prime factor then write this number as it is just
below itself, then continue this process of division by considering higher prime factors, till the quotient
in the last line is 1.


48 | C A T C L A S S E S B Y A s h a n k D u b e y

Examples

1. Find the LCM of 420, 3003, 6270
Solution:


Thus the required LCM = 2 x 2 x 3 x 5 x 7 x 11 x 13 x 19 = 1,141,140

! Product of two numbers = Product of their LCM and HCF
Note- This formula is applicable only for two numbers.

2. What is the least possible number of five digits which is divisible by 32, 36 and 40?
Solution: The number which is divisible by 32, 36 and 40 it must be the common multiple of all the given
numbers. Since we need such a least number then we have to find out the LCM of 32, 36 and 40.
The LCM of 32, 36 and 40 = 1440
But it is a four digit number. So we can take integral multiples of 1440 which must be divisible by the
given numbers. So when we multiply 1440 by 7. We get the required result i.e., 10,080.
Thus 10080 is the least possible 5 digit number which is divisible by 32, 36 and 40.

3. Find the highest number less than 1800 that is divisible by each of the numbers 2, 3, 4, 5, 6 and 7.
Solution: The LCM of 2, 3, 4, 5, 6 and 7 is 420. Hence 420, and every multiple of 420, is divisible by each
of these numbers. Hence, the number 420, 840, 1260, and 1680 are all divisible by each of these
numbers. We can see that 1680 is the highest number less than 1800 which is multiple of 420. Hence,
the highest number divisible by each one of 2, 3, 4, 5, 6 and 7, and less than 1800 is 1680.

4. Find the number of numbers lying between 1000 and 1,00,000 which are divisible by 15, 35 and 77
and are even too.
Solution: The required numbers must be the multiples of the LCM of 15, 35 and 77.
Now the LCM of 15, 35 and 77 = 1155.
So the other numbers are 1155, 2310, 3465, .., 99330
Thus there are total 86 numbers which are divisible by 15, 35 and 77 but only 43 numbers are even.
Thus there are total 43 required numbers.

5. Three bells in the Siva temple toll at the interval of 48, 72 and 108 second individually. If they have
tolled all together at 7:00 AM then at how many times these will these toll together till the 7: 00 PM
on the same day.
Solution: The three bells toll together only at the LCM of the times they toll individually.
Thus the LCM of 48, 72 and 108 is 432 seconds.
The total time since 7:00 AM till 7:00 PM
= 12 x 60 x 60 seconds.


49 | C A T C L A S S E S B Y A s h a n k D u b e y

Now since all these bells toll together at the interval of 432 seconds. So the number of times when they
will toll together
!"#$% !"#$ !" !" !"
= = = 100
!"#$% !"#$%&'( !" !"##$%& !"#$!!!" !"#
But since the bells toll together at 7:00 PM also. Hence total 101 = (100 + 1) times these bells will toll
together in the given duration.

6. The HCF of two numbers is 13 and their sum is 2080. How many such pairs are possible?
Solution: The numbers can be written as 13x and 13y where x and y are co-prime to each other.
Therefore, you quickly form the equation 13x + 13y = 2080 x + y = 160.
Now the concept of Eulers comes to your aid. The number of ways you can write 160 as sum of two co
!"#$%& !" !"#$%&' !"!!"#$% !" !"! (!) ! ! !
prime numbers = = = x 160 1 1 = 32.
! ! ! ! !

7. For how many pairs (a, b) of natural numbers is the LCM of a and b 23571113?
Solution: Let's solve for the powers of 2. One of the numbers will have 23 in it, as the LCM has 23. Now
the other number can have the powers of 2 as 20, 21, 22, and 23. Therefore, number of pairs will be 4: (23,
20), (23, 21), (23, 22), and (23, 23) and the number of ordered pairs will be 2 4 - 1 = 7 (we cannot count
the pair (23, 23) twice).
Similarly ordered pairs for powers of 5 = 2 8 - 1 = 15.
Number of ordered pairs for powers of 7 = 2 14 - 1 = 27.
Total ordered pairs (a, b) = 7 15 27 = 2835


! HCF of fractions =



! LCM of fractions =


Examples

1. Four runners started running the race in the same direction around a circular path of 7 km. Their
speeds are 4, 3, 9 and 3.5 km/hr individually. If they started their race at 6 oclock in the morning,
then at what time they will be at starting point?
! ! ! !
Solution: Time required by everyone to complete one revolution individually is , , , and hours.
! ! ! !.!
Therefore everyone must reach at the starting point after the time of the LCM of the individual time
period for one revolution.
! ! ! ! ! ! ! ! !"# !" !,!,!,! !"
So the LCM of , , , = , , , = = = 14 hours.
! ! ! !.! ! ! ! ! !"# !" !,!,!,! !


2. Three wheels can complete 60, 36, 24 revolutions per minute respectively. There is a red spot on each
wheel that touches the ground at time zero. After how much time, all these spots will simultaneously
touch the ground again? (CAT 1998)
(a) 5 s (b) 5/3 s (c) 7.5 s (d) 6 s
Solution:
Wheel 1 makes 60 revolutions in 1 minute.
Therefore time taken for 1 revolution = 60/60 = 1s


50 | C A T C L A S S E S B Y A s h a n k D u b e y

Wheel 2 makes 36 revolutions in 1 minute.
Therefore time taken for 1 revolution = 60/36 = 5/3 s
Wheel 3 makes 24 revolutions in 1 minute.
Therefore time taken for 1 revolution = 60/24 = 5/2 s
The red spot will be seen after the LCM of these times, which will be LCM of numerators and HCF of
denominators. Hence, the required answer will be 5.

HCF of decimals
Step 1. First of all equate the number of places in all the numbers by using zeroes, wherever
required.
Step 2. Then considering these numbers as integers find the HCF of these numbers.
Step 3. Put the decimal point in the resultant value as many places before the right most digit
as that of in every equated number.

Examples

1. Find the HCF of 0.0005, 0.005, 0.15, 0.175, 0.5 and 3.5
Solution:
0.0005 5
0.0050 50
0.1500 1500
0.1750 1750
0.5000 5000
3.5000 35000
Then the HCF of 5, 50, 1750, 5000 and 35000 is 5. So the HCF of the given number is 0.0005 (Since there
are four digits in all the adjusted decimal place.

2. Find the HCF of 0.9, 0.36 and 1.08.
Solution:
0.90 90
0.36 36
1.08 108
Now the HCF of 90 , 36 and 108 is 18.
So the HCF of the given numbers is 0.18

LCM of decimals
Step 1. First of all equate the number of places in all the numbers by using minimum possible
number of zeroes at the end of the decimal numbers, wherever required.
Step 2. Now consider the equated numbers as integers and then find the LCM of these
numbers.
Step 3. Put the decimal point in the LCM of the numbers as many places as that of in the
equated numbers.


51 | C A T C L A S S E S B Y A s h a n k D u b e y

Examples

1. Find the LCM of 1.8, 0.54 and 7.2


Solution
1.80 180
0.54 54
7.20 720
Now the LCM of 180, 54 and 720 is 2160. Therefore the required LCM is 21.60

2. Find the LCM of 4.44, 37 and 55.5
Solution:
4.44 444
37.0 3700
55.5 5550
Now the LCM of 444, 3700, 5550 is 11100
Hence the required LCM is 111.00 = 111

To find the number of numbers that are less than or equal to a certain natural number n, and
that are divisible by a certain integer:

To find the number of numbers, less than or equal to n, and that are divisible by a
certain integer p, we divide n by p. The quotient of the division gives us the number of numbers
divisible by p and less than or equal to n.

Examples

1. How many numbers between 1 and 400, both included, are not divisible either by 3 or 5?
(a) 187 (b) 400 (c) 133 (d) 213
Solution: We first find the numbers that are divisible by 3 or 5. Dividing 400 by 3 and 5, we get the
quotients as 133 and 80 respectively. Among these numbers divisible by 3 and 5, there are also numbers
which are divisible both by 3 and 5 i.e. divisible by 3 x 5 = 15. We have counted these numbers twice.
Dividing 400 by 15, we get the quotient as 26.
Hence, the number divisible by 3 or 5 = 133 + 80 26 = 187
Hence, the numbers not divisible by 3 or 5 are = 400 187 = 213.

2. How many numbers between 1 and 1200, both included, are not divisible by any of the numbers 2, 3
and 5?
(a) 320 (b) 880 (c) 240 (d) None of these
Solution: We first find the number of numbers divisible by 2, 3, or 5. from set theory we have
n(AUBUC) = n(A) + n(B) + n(C) n(AB) n(BC) n(AC) + n(ABC)
n(2U3U5) = n(2) + n(3) + n(5) n(6) n(15) n(10) + n(30)
n(2U3U5) = 600 + 400 + 240 200 80 120 + 40 = 880
Hence number of numbers not divisible by any of the numbers 2, 3, and 5 = 1200 880 = 320.


52 | C A T C L A S S E S B Y A s h a n k D u b e y

Number of numbers less than or co - prime to a given number:

If N is a natural number such that N = x x x where P, Q and R are different prime numbers
and x, y and z are natural numbers. then the number of natural numbers less than and prime to N =

N


Through above formula, not only we can find the numbers which do not contain any of the prime factors
of N but also the numbers which do not contain some selected prime factors of N. Lets try to
understand through examples:

1. How many of the first 120 natural numbers are not divisible by any of 2, 3 and 5?
(a) 23 (b) 40 (c) 21 (d) 32
Solution: 120 is a multiple of 2, 3 and 5. Therefore, we need to find the number of numbers which are
! ! ! ! ! !
less than and prime to 120. Number of numbers = 120 1 1 1 = 120 x x x = 32.
! ! ! ! ! !

2. How many of the first 120 natural numbers are not divisible by any of 2 and 5?
(a) 32 (b) 48 (c) 40 (d) 21
Solution: Unlike the previous problem, this problem only asks for number not divisible by only 2 factors
of 120, i.e. 2 and 5. Therefore, in the formula we remove the part containing the factor of 3 and
calculate the numbers of numbers prime to 120 with respect to prime factors 2 and 5. The required
! !
number = 120 1 1 = 48.
! !

Exercise 1.3

1. Two alarm clocks ring their alarms at regular (a) 9: 08 : 04 (b) 9 : 08 : 24
intervals of 50 seconds and 48 seconds. If they (c) 9 : 08 :44 (d) None of these
first beep together at 12 noon, at what time will
they beep again for the first time? 4. The HCF of 2472, 1284 and a third number N is
(a) 12:10 P.M. (b) 12:11 P.M. 12. If their LCM is 2! x 3! x 5 x 103 x 107, then
(c) 12:12 P.M. (d) None of these the number N is:
2! x 3! x 7 (b) 2! x 3! x 107
2. Four bells toll together at 9:00 A.M. They toll (c) 2! x 3! x 5 (d) None of these
after 7, 8, 11 and 12 seconds respectively. How
many times will they toll together again in the 5. Two equilateral triangles have the sides of
next 3 hours? lengths 34 and 85 respectively. What is the
(a) 3 (b) 4 (c) 5 (d) 6 greatest length of a tape that can measure both
of them exactly and how many such equal parts
3. On Mahatma Gandhi Marg, three consecutive can be measured respectively?
traffic lights change after 36, 42 and 72 seconds (a) 17, 7 (b) 16, 10
respectively. If lights are first switched on at (c) 18, 8 (d) None of these
9:00 A.M., at what time will they change
simultaneously?


53 | C A T C L A S S E S B Y A s h a n k D u b e y

6. A farmer wants to plant 44 apple trees, 66 such tiles and the minimum number of tiles,
banana trees and 110 mango trees in equal given that the length and breadth are integers is
rows (in terms of number of trees). Also, he (a) 858 (b) 864 (c) 868 (d) 872
wants to make distinct rows of trees (only one
type of tree in one row). The number of rows 13. How many pairs of numbers are possible
(minimum) that are required are: whose LCM is 280 and HCF is 7?
(a) 2 (b) 3 (c) 10 (d) 11 (a) 1 (b) 2 (c) 3 (d) 4

7. Find the HCF of (3!"# 1) and (3!" 1). 14. P (x, y) denotes the least common factor of x
3!" 1 (b) 3! 1 and y greater than 5 and Q (x, y) denotes the
!"
(c) 3 1 (d) None of these highest common multiple of x and y less than
![! !",!" , ! (!",!")
100. Find
![! !",!" , ! !",!" ]
8. The last digit of the LCM of 3!""# 1 and
(a) 1/24 (b) 1/12 (c) 1/6 (d) 1/48
3!""# + 1 is

(a) 2 (b) 4 (c) 6 (d) 8
15. LCM of two natural numbers and their product

is one of the numbers. Which of the following
9. How many of the first 1200 natural numbers are
statements is true?
either prime to 6 or to 15?
(a) HCF of the two given numbers and their
(a) 320 (b) 400 (c) 360 (d) None of these
product is 1.

(b) LCM of the two given numbers and their
10. The LCM of the two quotients, obtained when
product is the square of the larger number
two numbers and are divided by their HCF is
(c) The data is inconsistent
77. LCM of another pair of quotients, when
(d) None of the above
other two numbers c and d have the same HCF

as that of and , and when divided by it, is
16. The sum of two numbers is 462 and their HCF
221. Which of the following could be, ratio of
is 22. What is the maximum possible number
the sum of a and b to that of the difference
of pairs that satisfy these conditions?
between c and d?
(a) 8 (b) 2 (c) 3 (d) 6
(a) 4: 17 (b) 11: 4 (c) 2: 9 (d) 39: 2


17. ab and ba are two digit numbers, a and b are
11. LCM of two numbers A and B = ! x ! , where
distinct digits from 0 to 9. Difference of ab and
P and Q are prime numbers and and y are
ba is HCF of ab and ba. What is the value of a +
positive whole numbers. How many set of
b?
values are possible for A and B?
(a) 6 (b) 7 (c) 8 (d) 9
(a) ( + ) (b) ( )
! !

(c) ( + ) (d) None of these
18. Which of the following does not divide

5!! 3!! given that HCF (n, 4) = 2?
12. The square floor of dimensions 72 cm x 72 cm
(a) 7 (b) 11 (c) 13 (d) 317
has to be laid with rectangular tiles whose

length and breadth are in the ratio 3:2. The
19. At a bookstore, MODERN BOOK STORE is
difference between the maximum number of
flashed using neon lights. The words are


54 | C A T C L A S S E S B Y A s h a n k D u b e y

! ! !
individually flashed at the intervals of 2 s, 4 s 25. If N = = 0.pqrs is an non terminal
! ! !"#
! decimal, where, a, m, n, q are natural numbers
and 5 s respectively, and each word is put off
!
& GCD (a, 271) = 1, then What would be 98th
after a second. The least time after which the
digit after decimal of N?
full name of the bookstore can be read again is
(a) p (b) q (c) r (d) Data inadequate
(a) 49.5 s (b) 73.5 s (c) 220.5 s (d) 85.5 s


26. The LCM of three positive integers X, Y and Z
20. If in a set of three natural numbers X, Y, Z; X is
is 119! . Find the total number of ordered
the H.C.F of the three numbers then
triplets (X, Y and Z)
(a) One of Y, Z is the L.C.M. (X, Y, Z)
(a) 400 (b) 361 (c) 289 (d) 225
(b) LCM (X, Y) = LCM (Y, Z) = LCM (Z, X)

(c) LCM (X, Y, Z) = LCM (Y, Z)
27. How many pairs of numbers X and Y (X<Y) are
(d) XYZ = HCF (X, Y, Z) x LCM (X, Y, Z)
present such that the HCF (X, Y) = 6 and LCM

(X, Y) = 120?
21. If LCM (a, b) = 2! x 3! x x 4! x 5! then how
(a) 1 (b) 2 (c) 3 (d) 4
many different ordered pair of (a, b) are

possible
28. LCM of two distinct no is 1003. How many
(a) 3465 (b) 3464 (c) 1616 (d) 1617
values of HCF are possible?

(a) 1 (b) 2 (c) 3 (d) 4
22. If HCF (a, b) = 2! x 3! x x 4! x 5! then how

many different ordered pair of (a, b) are
29. What will be the HCF of all numbers of
possible
form ! , where n is an integer greater
(a) 1733 (b) 1617 (c) 809 (d) 810
than or equal to 2?

(a) 6 (b) 2
23. How many numbers are co - prime to 70
(c) 42 (d) None of these
between 200 and 500 including both?

(a) 102 (b) 100 (c) 104 (d) None of these
30. The numbers in the sequence 101, 104, 109,

116 are of the form 100 + ! , where n = 1, 2
24. If HCF (a, b) = 13 and a + b = 169000 then How
. Any 2 consecutive numbers are taken and
many unordered (a, b) are possible?
their HCF is found. The maximum value of the
(a) 2400 (b) 2350
HCF is
(c) 2450 (d) None of these
(a) 101 (b) 203 (c) 401 (d) 801


55 | C A T C L A S S E S B Y A s h a n k D u b e y

HIGHEST POWERS OF A NUMBER IN A FACTORIAL


Suppose we have to find the highest power of k that can exactly divide n!, we divide n by k, n by ! , n
!
by ! .. and so on till we get equal to 1(where, [x] means the greatest integer less than or equal to
!!
! ! ! !
x) and then add up as + + + ..+
! !! !! !!

Examples

1. What is the largest power of 5 in 121!?

!"! `"#"
Solution: + = 24 + 4 = 28. (We cannot do it further since 121 is less than 5! = 125)
! !!

Hence, 5 is involved 28 times as a factor in 121!

2. What is the largest power of 2 that can divide 269!?

!"# !"# !"# !"# !"# !"# !"# !"#


Solution: + + + + + + +
! !! !! !! !! !! !! !!

= 134 + 67 + 33 + 16 + 8 + 4 + 2 + 1 = 265.

Thus the greatest power of 2 is 265 that can divide exactly 268!.

Now, you must have noticed that we consider only integral values of the quotient as [q] which gives an
integer just less than or equal to q. So we can do this calculation in a slightly easier manner. Here we just
divide the given number and then succeeding quotients will be divided by the same remainder as in case
of successive division, So, we solve previous problem in this manner.

So we can see that this method of calculation is easier since we need not know the values of 2! , 2! , 2! ,
, 2! etc. Also the division by 2 is easier than the division by 2 ! , where is any large integer.


56 | C A T C L A S S E S B Y A s h a n k D u b e y

3. What is the largest power of 7 that can exactly divide 780!?
Solution:

Thus the highest power of 7 is 128 by which 777! can be divided.



4. What is the highest power of 72 in 100!?
Solution: Since this method is applicable only for the prime factors. So we solve it by breaking 72 in its
prime factors.
72 = 2! 3! . Therefore, we need to find the highest power of 2! and 3! in 72!.



72 = 8 x 9. Since to make a 8 we need 2 x 2 x 2 i.e. three times. So we need to divide 97 by 3 and get 32.
And, for 9 we need 3 x 3 i.e. two times. So we need to divide 48 by 2 and get 24.
So 2!" x 3!" 8!" x 9!"
Thus the largest power of 72 is 24. That can exactly divide 100!

5. What is the highest power of 81 that can divide 1802!?
Solution: 81 = 3!
So,


Thus we get 3!"# . But we need to find out for 3! .
Therefore 3!"# = 3! !!" x 3! = 81 !!" x 3
Therefore the largest power of 81 is 224 which can divide exactly 1802!.


57 | C A T C L A S S E S B Y A s h a n k D u b e y

Number of zeroes at the end of the product of numbers:

We know that 10 = 2 x 5, 100 = 2! x 5! etc. So we can say that "" number of zeroes at the end of the
product we need exactly "" combinations of 5 x 2

! The number of zeroes at the end of the product depends upon 2 x 5, but the condition is that
(a) x gives number of zeroes if < y
(b) x gives number of zeroes if y <

Examples

1. What is the number of zeroes at the end of 10!?


Solution: 10! = 1 x 2 x 3 x 4 x 5 x 6 x 7 x 8 x 9 x 10.
It is obvious from the above expression that there are only two 5s and eight 2s. Since the number of 5s
are less than number of 2s. So, number of 5s will be effective to form the combination of 2 x 5. Thus
there are only 2 zeroes at the end of 10!.

2. Find the number of zeroes at the end of 100!?
(a) 24 (b) 97 (d) 23 (d) None of these
Solution: Since we know that the zeroes at the end of any product are due to the presence of 10 as the
factor of the product and the number of zeroes depends upon the number of times 10 is involved.


So there are 24 combinations of 2 x 5 it means there will be 24 zeroes at the end of 100!
(Need to check for only 5 since it has smaller power.)

3. Find the number of zeroes at the end of the product of first 100 multiples of 10.
(a) 1000 (b) 24 (b) 100 (c) 124
Solution: (10 x 20 x 30 x 40 x .. x 1000)
= 10!"" [1 x 2 x 3 x 4 x x 100]
= 10!"" [100!]
As we know from previous example, highest power of 5 in 100! is 24.
Number of zeroes = 100 + 24 = 124.



58 | C A T C L A S S E S B Y A s h a n k D u b e y

4. How many natural numbers n are there, such that n! ends with exactly 30 zeroes?
(a) 0 (b) 1 (c) 3 (d) 4
Solution:
According to question n! should have 30 zeroes in the end. If n = 100 (taking randomly keeping in view
30 is the number of zeroes]
!"" !""
100! has + =24 ,So n should be greater than 100. Next multiple of 5 is 105. But 105 = 5 x 21,
! !!
has only one 5. Number of zeroes will increase by 1 only. Similarly 110, 115 and 120 also have one 5.
Number of zeroes (from 120! to 124!) = 29. Now, the next multiple of 5 is 125 and 125 contains three
5s. So, number of zeroes will increase by 3. Number of zeroes in 125! = 29 + 3 = 32. So, there is no
factorial of a number end with 30 zeroes.

5. n! has x number of zeroes at the end and (n + 1)! has (x + 3) zeroes at the end. 1n1000. How many
solutions are possible for n?
(a) 8 (b) 7 (c) 1 (d) 4
Solution: We can see that increasing the natural number by 1, we are gathering 3 more powers of 5.
Therefore, (n + 1) is a multiple of 125 but not a multiple of 625 as it would result in 4 powers of 5.
Therefore, (n + 1) will be equal to all the multiples of 125 minus 625.
Total number of multiples of 125 less than 1000 = 8
Total number of multiples of 625 less than 1000 = 1
The required answer is (8 1) =7

6. If n! and (4n)! end with 25 zeroes and 106 zeroes respectively, then which of the following is a factor
of n?
(a) 25 (b) 26 (c) 27 (d) 28
Solution:
Step 1:
According to question n! should have 25 zeroes in the end.
Here you have to try a bit to know the range of n. If n=100 (taking randomly keeping in view 26 is the
number of zeroes]
!"" !""
100! has + =24 ,So n should be greater than 100.
! !!
Next multiple of 5 is 105. But 105 = 5 x 21, has only one 5. Number of zeroes will increase by 1 only.
From 105 to 109, there is no multiple of 5. So, the range of n is in between 105 to 109.

Step 2:
As the next condition is (4n)! has 106 zeroes in the end. For getting the actual value just verify the value
of n within the range {105 to 109}. You will get n = 108. 108 = 4 x 27. The required answer = 27.





59 | C A T C L A S S E S B Y A s h a n k D u b e y

Exercise 1.4

1. A number is formed by multiplying the 1st 100 10. Find the smallest positive integer n such that
natural number multiples of 5. What will be the n! is a multiple of 10!""# .
highest power of 25 in that number? (a) 8045 (b) 8055
(a) 124 (b) 24 (c) 8060 (d) None of these
(c) 62 (d) None of these
11. n! has 100 trailing zeroes in base 35. Find the
2. The index of the highest power of 3 that divides minimum value of n.
1011121314151617181920.6869 is (a) 608 (b) 609 (c) 610 (d) 606
(a) 0 (b) 1 (c) 2 (d) 47
12. What is the smallest n such that n! ends in
3. If N is a natural number and ! has 16 factors, exactly 91 zeroes in base 91
what is the maximum number of factors that (a) 1105 (b) 1100
! can have? (c) 1010 (d) none of these
(a) 20 (b) 21 (c) 25 (d) 32
13. How many numbers less than or equal to 1000
4. What is the highest power of 30 in 50! are divisible by 125?
(a) 10 (b) 9 (a) 6 (b) 7 (c) 8 (d) 9
(c) 12 (d) None of these
14. How many numbers less than or equal to
5. If 146! Is divisible by 6! then find the maximum 10000 are divisible by 121?
value of n. (a) 80 (b) 81 (c) 82 (d) 83
(a) 70 (b) 74 (c) 75 (d) 76
15. The quotient obtained by dividing n by 121 and
6. Find the number of divisors of 15! the number 11 are co prime to each other.
(a) 2016 (b) 1008 Find the number of possible values of n if n is
(c) 4032 (d) none of these less than or equal to 10000. (It means that the
7. How many zeroes are present at the end of 24! highest power of 11 contained in the number n
+ 25!? is exactly equal to two.)
(a) 3 (b) 4 (c) 5 (d) 6 (a) 82 (b) 76
(c) 75 (d) none of these
8. 30! when expressed in base 12 ends with k
zeroes. Find k. 16. How many zeroes are present at the end of 25!
(a) 14 (b) 13 + 26! + 27! + 28! + 30!
(c) 12 (d) none of these (a) 5 (b) 6 (c) 7 (d) 8

9. Find the least value of n such that n! has exactly 17. What is the product of all factors of the
2394 zeroes. number N = 6! x 10! , which are divisible by 5?
(a) 9575! (b) 9580! (a) 2!"# x 3!"# x 5!"# (b) 2!"# x 3!"# x 5!"#
(c) 9585! (d) None of these (c) 2!"# x 3!"# x 5!"# (d) 2!"# x 3!"# x 5!"#


60 | C A T C L A S S E S B Y A s h a n k D u b e y

18. What digit does a represent, if 35! = 25. If A is the product of 100 multiples of 2 and B is
10333147966386144929a6665133752320000 the product of first 10 multiples of 5, then find
!
0000? the number of zeroes at the end of .
!
(a) 4 (b) 6 (c) 2 (d) 1 (a) 18 (b) 16 (c) 14
(d) 12 (e) 10
19. How many 4-digit multiples of 3 can be formed
26. What is the largest power of 12 that would
using the digits 2 and 3 only?
divide 49!
(a) 4 (b) 6 (c) 5 (d) 7 (a) 20 (b) 21 (c) 22
(d) 23 (e) 24
20. Given that 2!"#" is a 606 digit number whose
first digit is 1, how many elements of the set S 27. How many consecutive zeroes will be there at
= 2! , 2! , 2! , . 2!""# have a first digit of 4? the end of the product 2! !! + 4! !! +
(a) 199 (b) 197 6! !! + 8! !! + 10! !"! ?
(a) 10! + 6! (b) 2 (10!) (c) 10! + 8! + 6!
(c) 192 (d) 195
(d) 6! + 8! + 2(10!) (e) 6! + 2(8!) + 2(10!)

(!!)!
21. If has 112 trailing digits at the end the 28. What is the remainder when n! + (n! + 1) + (n!
!!
2) + (n! + 3) + .+ (n! 2006) is divided by
what is the digital sum of n
1003 for n = 1003?
(a) 5 (b) 6 (c) 7
(a) 0 (b) 1
(d)8 (e) 9 (c) 2006 (d) None of these

22. If a! x b! = (a x b)! Where a and b both are less Directions for questions 29 and 30: Answer the
than equal to 100 then how many ordered pair questions on the basis of the information given
of (a ,b) be possible. below.
The highest common factor and least common
(a) 0 (b) 100 (c) 200 (d) 202
multiple of two numbers A and B is 18 and 2268
respectively.
23. If N is a number which is equal to sum of
factorial of all its digits then how many such 29. How many pairs (A, B) are possible?
natural numbers are possible? (a) 4 (b) 6 (c) 8
(a) 1 (b) 2 (c) 3 (d) 10 (e) 12
(d) 4 (e) More than
30. Find the minimum possible sum of two such
24. If 2100! = 504 ! x q, where q is not a multiple numbers.
of 7, then find the value of P. (a) 360 (b) 396 (c) 402
(a) 346 (b) 305 (c) 388 (d) 414 (e) None of these
(d) 308 (e) 348


61 | C A T C L A S S E S B Y A s h a n k D u b e y

REMAINDERS

We know that when a number (N, the dividend) is divided by another smaller number (D, the divisor) its
gives us a quotient (Q) and a remainder (R) which is less than N. If a number N is divided by a number D
(where N D) and it leaves remainder (R),

Dividend (N) = Divisor (D) x Quotient (Q) + Remainder (R)

Example: When 18 is divided by 11, it leaves a remainder 7; in other words 18 = 11 x 1 + 7.

Basic Remainder Theorem

(i) Summation Rule: Suppose the numbers N1, N2, N3, N4 when divided by a common divisor D, give
quotients Q1, Q2, Q3, Q4 and remainders R1, R2, R3, R4..., respectively.

Therefore, N1 = D Q1 + R1,
N2 = D Q2 + R2,
N3 = D Q3 + R3,
N4 = D Q4 + R4 and so on.

Let S be the sum of N1, N2, N3, N4
Therefore, S = (N1) + (N2) + (N3) + (N4)... = (D Q1+ R1) + (D Q2+ R2) + (D Q3+ R3) + (D Q4+ R4)..
= D(Q1 + Q2 + Q3 + Q4 +..) + (R1 + R2 + R3 + R4 + .)
= D x K + (R1 + R2 + R3 + R4 + .); where K is some number.

In the above equation, since only the summation R1 + R2 + R3 + R4 +...is free of D, therefore, the
remainder when S is divided by D is the remainder when (R1 + R2 + R3 + R4 +...) is divided by D

! The remainder of the sum of two numbers a and b when divided by n is equivalent to the sum
of their individual remainders when divided by n.

Examples
1. What is the remainder when the sum (1998 + 1999 + 2000) is divided by 7?
(a) 1 (b) 3 (c) 4 (d) 5
Solution: The remainders when 1998, 1999, and 2000 are divided by 7 are 3, 4, and 5 respectively.
Hence the final remainder is the remainder when the sum (3 + 4 + 5) = 12 is divided by 7. Therefore,
remainder = 5

2. What is the remainder when (4 + + + + ) is divided by 6?
(a) 1 (b) 2 (c) 3 (d) 4
Solution: The remainder when 4 is divided by 6 is 4
The remainder when 4! is divided by 6 is 4
The remainder when 4! is divided by 6 is 4
The remainder when 4! is divided by 6 is 4
The remainder when 4! is divided by 6 is 4
So required remainder can be obtained by dividing (4 + 4 + 4 + 4 + 4) = 20 by 6. Thus the required
remainder is 2.


62 | C A T C L A S S E S B Y A s h a n k D u b e y

3. Find the remainder when (4 + + + + + .+ ) is divided by 6?


(a) 1 (b) 2 (c) 3 (d) 4
Solution: The remainder when 4 is divided by 6 is 4
The remainder when 4! is divided by 6 is 4
The remainder when 4! is divided by 6 is 4
The remainder when 4! is divided by 6 is 4
The remainder when 4! is divided by 6 is 4
Thus the remainder is always 4.
!!!!!!!! !"#$% !"#
So required remainder = = . Thus the remainder is zero.
! !

4. A number when divided by 238 leaves a remainder 79. What will be the remainder when that number
is divided by 17?
(a) 9 (b) 10 (c) 11 (d) 12
Solution: Let N be a number when divided by 238 leaves a remainder 79.
N = 238K + 79 = (17 x 14 x K + 79). The remainders when 238k and 79 are divided by 17 are 0 and 11
respectively. Hence the final remainder is (0 + 11) = 11

(ii) Product Rule: Suppose the numbers N1, N2, N3, N4 when divided by a common divisor D, give quotients
Q1, Q2, Q3, Q4 and remainders R1, R2, R3, R4..., respectively.

Therefore, N1 = D Q1 + R1,
N2 = D Q2 + R2,
N3 = D Q3 + R3,
N4 = D Q4 + R4 and so on.

Let P be the product of N1, N2, N3
Therefore, P = N1N2N3N4.. = (D Q1 + R1)(D Q2 + R2)(D Q3 + R3)( D Q4 + R4)...
= D K + R1R2R3R4... where K is some number

In the above equation, since only the product R1R2R3R4 is free of D, therefore the remainder when P
is divided by D is the remainder when the product R1R2R3 is divided by D.

! The remainder of the product of two numbers a and b when divided by n is equivalent to the
product of their individual remainders when divided by n.

Examples

1. What is the remainder when (1421 x 1423 x 1425) is divided by 12?
(a) 0 (b) 3 (c) 6 (d) 9 (CAT 2000)
Solution: The remainders when 1421, 1423, and 1425 are divided by 12 are 5, 7, and 9 respectively.
Hence the final remainder is the remainder when the product 5 7 9 = 315 is divided by 12. Therefore,
remainder = 3. Option (b)
2. What is the remainder when the product 1998 1999 2000 is divided by 7?
(a) 3 (b) 4 (c) 5 (d) 6
Solution: the remainders when 1998, 1999, and 2000 are divided by 7 are 3, 4, and 5 respectively. Hence
the final remainder is the remainder when the product 3 4 5 = 60 is divided by 7. Therefore,
remainder = 4. Option (b)


63 | C A T C L A S S E S B Y A s h a n k D u b e y

3. What is the remainder if is divided by 8?


(a) 1 (b) 7 (c) 5 (d) None of these
Solution: 9!" is a product (9 x 9 x 9 x(35 times)). The remainder when 9 is divided by 8 is 1. Hence
the remainder when 9!" is divided by 8 is the remainder obtained when product 1 x 1 x 1 x.. is divided
by 8. Therefore, remainder = 1.

4. What is the remainder when is divided by 7?
(a) 1 (b) 2 (c) 4 (d) 6
!"#$
Solution: 2 is again a product (2 2 2... (2013 times)). Since 2 is a number less than 7 we try to
convert the product into product of numbers higher than 7. Notice that 8 = 2 2 2. Therefore we
convert the product in the following manner- 22013 = 8671 = 8 8 8... (671 times).
The remainder when 8 is divided by 7 is 1. Hence the remainder when 8671 is divided by 7 is the
remainder obtained when the product 1 1 1... is divided by 7. Therefore, remainder = 1. Option (a)

5. What is the remainder when is divided by 7?
(a) 1 (b) 2 (c) 4 (d) 6
Solution: This problem is like the previous one, except that 2015 is not an exact multiple of 3 so we
cannot convert it completely into the form 8 ! . We will write it in following manner- 22015 = 8671 4. Now,
8671 gives the remainder 1 when divided by 7 as we have seen in the previous problem. And 4 give a
remainder of 4 only when divided by 7. Hence the remainder when 22015 is divided by 7 is the remainder
when the product 1 4 is divided by 7. Therefore, remainder = 4

6. What is the remainder when 4343 is divided by 9?
(a) 3 (b) 5 (c) 7 (d) None of these
Solution: Again 43 = (36 + 7) = (36 + 7)(36 + 7)....43 times = 36K + 7!"
!" !"

Hence remainder when 43!" is divided by 9 is the remainder when 7!" is divided by 9.
Now 7!" = 73 73 73. (14 times) 7 = 343 343 343... (14 times) 7.

The remainder when 343 is divided by 9 is 1 and the remainder when 7 is divided by 9 is 7.
Hence the remainder when 7!" is divided by 9 is the remainder we obtain when the product 1 1 1...
(14 times) 7 is divided by 9. The remainder is 7 in this case. Hence the remainder when 7!" is divided
by 9 is 7.

7. The remainder when 784 is divided by 342 is
(a) 0 (b) 1 (c) 49 (d) 341 (CAT 1999)
!"
Solution: 7 is a product (7 x 7 x 7 x(84 times)). Since 7 is a number less than 342 we try to convert
the product into product of numbers higher than 342. Notice that 343 = 7 7 7. Therefore we convert
the product in the following manner- 784 = 34324 = 343 343 x... (24 times). The remainder when 343 is
divided by 342 is 1. Hence the remainder when 34324 is divided by 342 is the remainder obtained when
the product 1 1 1... is divided by 343. Therefore, remainder = 1. Option (b)
The concept of negative remainder

17 = 18 0 + 17 or 17 = 18 1 1.
The remainder when 17 is divided by 18 is 17 in the first case and 1 in the second case. Hence, the
remainder when 17 is divided by 18 is 17 or 1.
When a number N < D gives a remainder R (= N) when divided by D, it gives a negative remainder of R
D.


64 | C A T C L A S S E S B Y A s h a n k D u b e y

For example, when a number gives a remainder of 21 with 23, it means that the number gives a negative
remainder of 21 23 = -2 with 23.

Examples

1. Find the remainder when 22 x 23 is divided by 8.
(a) 2 (b) 1 (c) 4 (d) 5
Solution: The obviously approach in this case would be: The remainders when 22 and 23 are divided by
8 are 6 and 7 respectively. Hence the final remainder is the remainder when the product 6 7 = 42 is
divided by 8. Therefore, remainder = 2.
However there is another option by which we can solve the same problem:
When 22 is divided by 8, the remainder is normally seen as +6. However, there might be times when
using the negative value of the remainder might give us more convenience.
Remainders by definition are always non negative. Hence, even when we divide a number like 23
by 5 we say that remainder is 2 (and not 3). However looking at negative value of the remainder it
has its own advantages in reducing calculation.
Thus, the remainders when 22 and 23 are divided by 8 are 2 and 1 respectively. The final remainder
is the remainder when the product 2 1 = 2 is divided by 8. Therefore, remainder = 2.

2. Find the remainder 32 x 33 x 34 is divided by 36.
Solution: The remainders when 32, 33 and 34 are divided by 36 are 4, 3 and 2 respectively. The
final remainder is the remainder when the product 4 3 x 2 = 24 is divided by 36. Therefore,
remainder = 36 24 = 12.

Examples

1. When is divided by 17, the remainder would be
(a) 1 (b) 14 (c) 16 (d) None of these (CAT 2002)
Solution: Again 2!"# = 16!" = 17 1 !" = (17 1)(17 1)....64 times = 17K + (1)!" .
Hence remainder when 2!"# is divided by 17 is the remainder when (1)!" is divided by 17.
(1)!" = 1. The remainder is 1 in this case. Hence the remainder when 2!"# is divided by 17 is 1.

2. What is the remainder when 752 is divided by 2402?
(a) 1 (b) 241 (c) 2401 (d) None of these
Solution: 7 = (74) = (2401) = (2402 1) = 2402K + (1)13 = 2402K 1.
52 13 13 13

Hence, the remainder when 752 is divided by 2402 is equal to 1 or 2402 1 = 2401.
Remainder = 2401.


3. What is the remainder when is divided by 7?
(a) 1 (b) 2 (c) 4 (d) 6

Solution: Lets use this as an opportunity to list down the steps for finding remainder when is
divided by D.
!
Step 1: Divide A by D. Let the remainder be R. Therefore, we have to find the when ! is divided by D.
!"
32 gives a remainder 4 when divided by 7. Therefore, we are trying to find the remainder when 4!" is
divided by 7.


65 | C A T C L A S S E S B Y A s h a n k D u b e y

Step 2: Find a power of R that gives a remainder of + 1 or 1 with D. If we find a power that gives a
remainder 1, twice of that power will give a remainder of + 1. Now we know that 4! = 64 gives a
remainder 1 when divided by 7.

Step 3: Find the remainder when ! is divided by that power. Here, find the remainder when 32!" is
!"
divided by 3. The remainder is 1. Therefore, 32!" can be written as 3K + 1 and 4!" can be written as
4!!!! or 4!! x 4.
Step 4: Now 4! gives a remainder 1 when divided by 7. Therefore, we need to find the remainder when
4 is divided by 7. Therefore, the remainder is 4.

Some Special Cases:

When both the dividend and the divisor have a factor in common.

Let N be a number and Q and R be the quotient and the remainder when N is divided by the divisor D.
Hence, N = Q D + R.
Let N = k A and D = k B where k is the HCF of N and D and k > 1. Hence kA = Q kB + R.
Let Q1 and R1 be the quotient and the remainder when A is divided by B. Hence A = B Q1 + R1.
Putting the value of A in the previous equation and comparing we get
k(B Q1 + R1) = Q kB + R R = kR1.
Hence to find the remainder when both the dividend and the divisor have a factor in common,

" Take out the common factor (i.e. divide the numbers by the common factor)
" Divide the resulting dividend (A) by resulting divisor (B) and find the remainder (R1).
" The real remainder R is this remainder R1 multiplied by the common factor (k).

Examples

1. What the remainder when 248 is divided by 48?
(a) 2 (b) 32 (d) 1 (d) 16
Solution: The common factor between 2 and 48 is 16 = 24.
48

Removing 16 from the dividend and the divisor we get the numbers 244 and 3 respectively.
The remainder when 244 is divided by 3 is 1.
Hence the real remainder will be 1 multiplied by common factor 16.
Remainder = 16.

2. What is the remainder when is divided by 24?
(a) 1 (b) 23 (c) 16 (d) 8
Solution: The common factor between 10 and 24 is 8 = 23.
200

Removing 8 from the dividend and the divisor we get the numbers 5200 x 2197and 3 respectively.
The remainder when 5200 = 25100 is divided by 3 is 1 and 2197 = 2 x 2196 is divided by 3 is 2.
Hence the real remainder will be 1 x 2 multiplied by common factor 8. Remainder = 16.

3. What is the remainder when 5 x is divided by 6 x ?
(a) 1 (b) 5 (c) 5 x (d) None of these
Solution: As we know that we cannot reduce the fractions to its lower terms, the remainder obtained
will be equal to 5 x 10! .


66 | C A T C L A S S E S B Y A s h a n k D u b e y

When dividend is of the form + or

Divisible by (A B) Divisible by (A + B)

m is even Yes Yes


m is odd Yes No
+ m is even No No
m is odd No Yes

Examples

1. The remainder, when ( + ) is divided by 19, is
(a) 4 (b) 15 (c) 0 (d) 18 (CAT 2004)
! ! !" !"
Solution: The dividend is in the form + . Now (15 + 23 ) will be divisible by 15 + 23 = 38.
Since the number is divisible by 38 it will certainly be divisible by 19. Hence, the remainder is 0.

2. What is the remainder when 3444 + 4333 is divided by 5?
(a) 3 (b) 4 (c) 1 (d) 0
x y
Solution: The dividend is in the form A + B . We need to change it into the form ! + ! .
3444 + 4333 = (34)111 + (43)111. Now (34)111 + (43)111 will be divisible by 34 + 43 = 81 + 64 = 145. Since the
number is divisible by 145 it will certainly be divisible by 5. Hence, the remainder is 0.

3. , where n is an integer > 0, is divisible by
(a) 13 (b) 127 (c) 559 (d) All of these (CAT 2002)
Solution: The dividend is in the form ! + ! . Now (7!! 6!! ) will be divisible by (7 + 6) = 13. Put n
= 1, 7! ! 6! ! = 7! 6! 7! + 6! = 127 x 559. As the expression is divisible by 13, 127 and 559.
Hence, the remainder is 0. Option (d).

4. What is the remainder when (5555)2222 + (2222)5555 is divided by 7?
(a) 1 (b) 0 (c) 6 (d) 3
Solution: The remainders when 5555 and 2222 are divided by 7 are 4 and 3 respectively. Hence, the
problem reduces to find the remainder when (4)2222 + (3)5555 is divided by 7.
Now (4)2222 + (3)5555 = (42)1111 + (35)1111 = (16)1111 + (243)1111. Now (16)1111 + (243)1111 is divisible by 16 +
243 or it is divisible by 259, which is a multiple of 7. Hence the remainder when (5555)2222 + (2222)5555 is
divided by 7 is zero.

5. If x = + + + , then x divided by 70 leaves a remainder of
(a) 0 (b) 1 (c) 69 (d) 35 (CAT 2005)
Solution: 16! + 17! + 18! + 19! = 16! + 19! + (17! + 18! ) will be divisible by (16 + 19) + (17 +
18) = (35 + 35) = 70. Since the number is divisible by 70. Hence, the remainder is 0.

6. 182000 + 122000 52000 1 is divisible by
(a) 323 (b) 221 (c) 299 (d) 237
Solution: Rule: an bn is divisible by both a + b and a b when n is even.
182000 + 122000 52000 1 = 182000 52000 + 122000 12000 182000 52000 is divisible by 13 and 23.
Similarly, 122000 12000 is divisible by 11 and 13. As 13 is the common factor, the whole expression is
divisible by 13.


67 | C A T C L A S S E S B Y A s h a n k D u b e y


182000 + 122000 52000 1 = 182000 1 + 122000 52000 182000 1 is divisible by 17 and 19.
Similarly, 122000 52000 is divisible by 7 and 17. As 17 is the common factor, the whole expression is
divisible by 17.
As the expression is divisible by both 13 and 17, it is divisible by 221.

Remainder in case of polynomial
In case of polynomials, we are not required to find the remainder by division method. By using the
factor theorem, we can find out the remainder.

When f(x) = a + bx + cx2 + dx3 + ex4... is divided by (x a)

The remainder when f(x) = a + bx + cx2 + dx3 +.. is divided by (x a) is f(a). Therefore, If f(a) =
0, (x a) is a factor of f(x).

Examples

1. What is the remainder when x4 3x2 + 1 is divided by (x 2)?
(a) 0 (b) 1 (c) 3 (d) 5
Solution: The remainder when the expression is divided by (x 2) will be f(2). Remainder = (2)4 3(2)2 +
1 = 5

2. What is the remainder when x3 + 2x2 + 5x + 5 is divided by (x + 1)?
(a) 1 (b) 0 (c) 1 (d) 5
Solution: The remainder when the expression is divided by (x (1)) will be f(1). Remainder = (1)3 +
2(1)2 + 5(1) + 5 = 1

3. If x3 3x2 + 4x c is divisible by (x 3), find the value of c.
(a) 6 (b) 8 (c) 10 (d) 12
Solution: Since the expression is divisible by (x 3), the remainder f(3) should be equal to zero
3! 33! + 43 = 0 27 27 + 12 c = 0, or c = 12. Option (d)

4. What is the value of y, if + 3 + 4, when it is divided by ( 2), gives a remainder of y?
(a) 0 (b) 1 (c) 4 (d) 8
Solution: Since the expression is divided by (x 2), the remainder f(2) should be equal to y
2! + 3 x 2! 2 + 4 = y
8 + 12 2 + 4 = y
y = 8. Option (d)

5. A number N gives a remainder of 7 when divided by D and a remainder of 20 when divided by 2D.
What will be the remainder obtained when 2N is divided by D?
(a) 0 (b) 1 (c) 3 (d) 7
Solution: N = DQ + 7 (Where Q = Quotient) .. (1)
N = 2D x P + 20 (Where P is Quotient) (2)
Compare equation (1) and equation (2):
We can say that when N = 2D x P is divided by D, remainder will be 7.


68 | C A T C L A S S E S B Y A s h a n k D u b e y

!!"!!" !!" !"
= +
! ! !
The remainder when 2DK is divided by D is 0.
The remainder when 20 is divided by D is 7.
The remainder when (0 + 7) is divided by D is 7 D = 13.
Now, 2N = 2(DQ + 7) = 4DQ + 14.
!! !"
Remainder obtained from = Remainder obtained from = 1.
! !"
Hence, remainder is 1.

6. The remainders when F(x) is divided by (x 99) and (x 19) are 19 and 99, respectively. What is the
remainder when F(x) is divided by (x - 19) (x - 99)?
(a) (118 x) (b) (x 118) (c) 1 (d) 0
Solution: F(x) = (x 99)a + 19 --- (1) and F(x) = (x 19)b + 99 --- (2). Multiplying (1) by (x 19) and (2) by
(x 99) we get
(x 19)F(x) = (x 99)(x 19)a + 19(x 19) and
(x 99)F(x) = (x 99)(x 19)b + 99(x 99)
Subtracting, we obtain:
! ! !" ! ! !! !
80F(x) = (x 99)(x 19)k 80x + 9440 F(x) = + 118. We know from (1) and (2) that
!"
F(x) does not have fractional coefficients. Therefore, k will be divisible by 80. Therefore, the remainder
when F(x) is divided by (x 99)(x 19) is (118 x).

Eulers Theorem
This is a very useful result. It might take a little time to understand and master Eulers theorem
completely but once understood, it is an asset!

If A and B are two numbers co - prime to each other, i.e. HCF (A, B) = 1 and B = x x x where
()
P, Q and R are prime numbers and x, y and z are natural numbers. Remainder = 1


Here, (B) = B


Examples

1. Find the remainder when 538 is divided by 63.
(a) 5 (b) 1 (c) 25 (d) 0
Solution: 5 and 63 are co - prime to each other, therefore we can apply Eulers theorem here.
! !
63 = 3! x 7 (63) = 63 1 1 = 36.
! !

Therefore, Remainder = Remainder = 5! = 25.


2. What are the last three digits of 371202?
(a) 069 (b) 169 (c) 269 (d) 369

Solution: The faster way to calculate the last three digits is to calculate the remainder by 1000. We can
see that 37 and 1000 are co - prime to each other. Therefore, we can use Eulers theorem here if its
useful.
! !
1000 = 2! x 5! (1000) = 1000 1 1 = 400.
! !
Therefore,


69 | C A T C L A S S E S B Y A s h a n k D u b e y

!"!"" !"!"" !"!"" !"!"" !"!"##
Remainder = 1 Remainder = Remainder = 1
!""" !""" !"""
!"!"#" !"!"## !"!
Remainder = Remainder = 369.
!""" !"""
1202
Hence, the last three digits of 37 are 369.

3. The remainder when 72006 is divided by 36 is
(a) 1 (b) 13 (c) 35 (d) 7
Solution: 7 and 36 are co - prime to each other, therefore we can apply Eulers theorem here.
! !
36 = 2! x 3! (36) = 36 1 1 = 12.
! !
!"#
!!""# !!" !! !!
Therefore, Remainder = Remainder = Remainder = 13.
!" !" !"


4. What is the remainder when is divided by 9?
(a) 1 (b) 2 (c) 4 (d) 7
!
Solution: Notice that 32 and 9 are co - prime. (9) = 9 1 = 6
!
!"!
Hence by Eulers theorem, Remainder = 1. Since the power is 32!" , we will have to simplify this
!
power in terms of 6k + r. Therefore, we need to find the remainder when 32!" is divided by 6.
!
!"!" !!" !! !"# !"# !"# !"#
Remainder = Remainder = Remainder = Remainder = Remainder
! ! ! !
!"#
= 4
!
!"
Therefore, 32!" = 32!!!! = 32! ! x 32!
!
!"! !"! !"! ! ! ! ! !"#
Remainder = Remainder = Remainder = Remainder = 4.
! ! ! !

5. What is the remainder when 8643 is divided by 132?
(a) 116 (b) 29 (d) 1 (d) none of these
Solution: Note that here 8 and 132 are not co - prime as HCF (8, 132) = 4 and not 1. Therefore, we
cannot apply Eulers theorem directly.
!!"# !!"#" !!"#$
Remainder = Remainder = 4 x Remainder . Now we can apply Eulers theorem.
!"# !"# !!
! ! !!" !!"#$ !!
(33) = 33 1 1 = 20 Remainder = 1 Remainder = Remainder = 29
! !! !! !! !!
Real remainder 4 x 29 =116

Fermats Theorem

! If P is a prime number and N is prime to P, then N is divisible by P.

Examples


1. If P is an integer, is always divisible by:

(a) 26 (b) 27 (c) 28 (d) 29
! !" ! !"
Solution: = . Since 29 is a prime number !" is always divisibly by 29.
!


70 | C A T C L A S S E S B Y A s h a n k D u b e y

2. What is the remainder when n7 n is divided by 42?
(a) 0 (b) 1 (c) 41 (d) None of these
Solution: Since 7 is prime, n n is divisible by 7. n7 n = n(n6 1) = n (n + 1)(n 1)(n4 + n2 + 1). Now (n
7

1)(n)(n + 1) is divisible by 3! = 6. Hence n7 n is divisible by 6 x 7 = 42. Hence the remainder is 0.



Fermat's Little Theorem


! If B in the above Eulers theorem is a prime number, then (B) = B = (B 1). Therefore,

(!)
if A and B are co - prime to each other and B is a prime number, Remainder = 1


Examples

1. What is the remainder when is divided by 13?
(a) 2 (b) 4 (c) 8 (d) 1
!
Solution: 13 is a prime number therefore (N) = 13 1 = 12. 2 and 13 are co prime to each
!"
!"#
!!" !!" !
other. Therefore, by Fermats little theorem: Remainder = 1 Remainder = 2
!" !"

2. What is the remainder when 5366 is divided by 17?
(a) 2 (b) 4 (c) 8 (d) 16
!
Solution: 17 is a prime number therefore (N) = 17 1 = 16. 17 and 53 are co prime to each
!"
other. Therefore, by Fermats little theorem:
!"!" !"!" !"!" !"!" !"!" !"! !"!
Remainder = 1 Remainder = Remainder = 4.
!" !" !"

3. What will be the remainder when N = + + +........... + is divided by
7?
(a) 1 (b) 2 (c) 4 (d) 5
Solution: By Fermats Little Theorem, 10! will give remainder as 1 with 7.
!"!" !"! !"! !"! !!
Remainder = Remainder = Remainder = Remainder = 4
! ! ! !
Similarly, all the other terms give remainder of 4 with 7. Therefore, total remainder = 4 + 4 + 4 (10
times) = 40.
Remainder of 40 with 7 = 5

Wilsons Theorem

! !!
! If P is a prime number then Remainder = = 0. In other words, (P 1)! + 1 is divisible by P if P is a

prime number. It also means that the remainder when (P 1)! is divided by P is (P 1), when P is prime.
Examples

1. What is the remainder when 96! is divided by 97?.
(a) 0 (b) 1 (c) 96 (d) None of these
!"!
Solution: By Wilsons theorem, we can see that 96! + 1 is divisible by 97 Remainder = (97 1) =
!"
96.


71 | C A T C L A S S E S B Y A s h a n k D u b e y

2. What is the remainder when 95! is divided by 97?.
(a) 0 (b) 1 (c) 96 (d) None of these
Solution: In the above example, we saw that the remainder when 96! is divided by 97 is 96. 96! = 97k
+ 96 96 95! = 97k + 96. The R.H.S. gives remainder 96 with 97 therefore L.H.S. should also give
remainder 96 with 97. L.H.S. = 96 95! which gives remainder 96 with 97.
Therefore, 95! should give remainder 1 with 97. Option (b)

Chinese Remainder Theorem

! If a number B = P Q, where P and Q are prime to each other, i.e., HCF(P, Q) = 1, and A is a number

such that Remainder = and remainder = then Remainder = Smallest number A,

which satisfies both the conditions.

Lets try to understand the application of Chinese Remainder Theorem through the following examples:

Examples
1. Find the remainder when 3101 is divided by 77.
(a) 1 (b) 0 (c) 76 (d) 47
Solution: 77 = 11 7.
!! !!"
By Fermats little theorem, Remainder = 1 and Remainder = 1
! !!
!"
!!"! !!" !! !! !! ! !!
Remainder = Remainder = Remainder = Remainder = 5 = !
! ! ! !
!"
!!"! !!"" ! !!" ! !!
Remainder = Remainder = Remainder = Remainder = 3 = !
!! !! !! !!
Therefore, to find the remainder we need to find the smallest number that gives remainder 5 with 7 and
3 with 11. Therefore, the number would satisfy the equation 7a + 5 = 11b + 3 11b 7a = 2 (a, b) =
(6, 4) and the number = 47. Therefore, remainder = 47.

2. What is the remainder when is divided by77?
(a) 36 (b) 1 (c) 0 (d) None of these
Solution: 77 = 7 x 11
Lets start by separately finding the remainders of 9!"#$ with 7 and with 11
!"#
!!"#$ !! !! !!"# !!
Remainder = Remainder = Remainder = 3. i.e. 9!"#$ is of the form 11a + 3
!! !! !!
and also
!"#
!!"#$ !! !!"#
Remainder = Remainder = Remainder = 1. i.e. 9!"#$ is of the form 7b + 1
! ! !
Now if we combine these two results, 9!"#$ is in the forms of 11a + 3 and 7b + 1.
Therefore, the number would satisfy the equation 11a + 3 = 7b + 1 7b 11a = 2 (a, b) = (3, 5) and
the number = 36. Therefore, remainder = 36.


3. What is the remainder when is divided by 117
(a) 0 (b) 1 (c) 116 (d) None of these
Solution: 117 = 9 x 13


72 | C A T C L A S S E S B Y A s h a n k D u b e y

!!" !
!"!" !"!!!! !"! !"! ! !"
Remainder = Remainder = Remainder = Remainder = 12. i.e.
!" !" !" !"
!!"
17!" = 13a + 12
!!" !!"
!"!" (!!)!" !!"
Remainder = Remainder = - 1 or 8 i.e. 17!" = 9b + 8
! !
The first value satisfying both is 116.

4. Find the remainder when 123456789101112..40 is divided by 36.
(a 16 (b) 24 (c) 28 (d) None of these
Solution: Hmmwhere to start? Lets try Chinese Remainder Theorem (CRT).
36 = 9 4. Therefore, we first find the remainders when this number is divided by 9 and 4. The
remainder by 9 would be the remainder when the sum of digits is divided by 9.
Sum of digits = 4 (1 + 2 + 3 + 4 + + 9) + 10 (1 + 2 + 3) + 4 = 180 + 60 + 4 = 244
Remainder by 9 = 1.
The remainder by 4 would be the remainder when the last two digits are divided by 4.
Remainder by 4 = 0.
Therefore, as per the CRT, to find the remainder we need to find the smallest multiple of 4 that gives
remainder 1 with 9. Therefore, the number would satisfy the equation 9a + 1 = 4b 4b 9a = 1 (a, b)
= (3, 7) and the number = 28. Therefore, remainder = 28. Option (c)

5. Find the remainder when 11212312341234512345678 is divided by 36.
(a) 21 (b) 12 (c) 30 (d) None of these
Solution: Again, we first find the remainders when this number is divided by 9 and 4.
The remainder by 9 would be the remainder when the sum of digits is divided by 9. The trick is in the
ability to find out the sum of digits.
Sum of digits = 1 8 + 2 7 + 3 6 + + 8 1 = 120 remainder by 9 = 3.
The remainder by 4 would be the remainder when the last two digits are divided by 4
remainder by 4 = 2.
The overall remainder would be the smallest number that gives remainder 3 with 9 and remainder 2
with 4. Therefore, the number would satisfy the equation 9a + 3 = 4b + 2 4b 9a = 1 (a, b) = (3, 7)
and the number = 30. Therefore, remainder = 30. Option (d)

Binomial Theorem

!
( + ) = nC0 + nC1 (!) + nC2(!) + . where nCr = ! ! !

Examples

1. What is the remainder when + is divided by 49?
(a) 0 (b) 14 (c) 35 (d) 42
!" !" 83 83
Solution: 6 + 8 = (7 + 1) + (7 1)
(7 + 1)83 = {83C0783 + 83C1 7!" +.. + 83C82 7 + 83C83}
= {783 + 83 x7!" +.. + 83 x 7 + 1}

(7 1)83 = {83C0783 1 ! + 83C1 7!" 1 ! +.. + 83C827 1 !"
+ 83C83 1 !"
}
= {783 83 x 7!" +.. + 83 x 7 1}


73 | C A T C L A S S E S B Y A s h a n k D u b e y

In the above expressions only the last two numbers are not multiple of 49. Hence remainder will come
only from (83 x 7 + 1) + (83 x 7 1) = 2 83 7
So the net remainder obtained from (6!" + 8!" ) is same as remainder obtained from 2 83 7
2 83 7 is divided by 49. Remainder = 35.

2. What is the remainder when + is divided by 36?
(a) 0 (b) 12 (c) 35 (d) 42
!" !" !" !"
Solution: (5 + 7 ) = 6 1 + 6 + 1
6 + 1 !" = {50C0650 + 50C1 6!" +.. + 50C49 6 + 50C50}
= {650 + 50 x6!" +.. + 50 x 6 + 1}

When 6 + 1 !" is divided by 36 (=6! ), except last two terms of the above expression, all the terms will
be divisible. Hence the remainder will come only from {50 x 6 + 1}
6 1 !" = {50C0650 1 ! + 50C1 6!" 1 ! +.. + 50C496 1 !" + 50C50 1 !" }
= {650 50 x 6!" +.. 50 x 6 + 1}
In this case too, the remainder will come only from { 50 x 6 + 1}
So the net remainder obtained from (5!" + 7!" ) is same as remainder obtained from {50 x 6 + 1} + { 50
x 6 + 1} = 2 is divided by 36. Remainder = 2

(a + b + c + ...) is always divisible by (a + b + c + ...) where
! All alphabets represent positive integers
! a, b, c, ... are in arithmetic progression
! n is odd.

Examples

1. If x = ( + + + ), then x divided by 70 leaves a remainder of
(a) 1 (b) 0 (c) 35 (d) 69 (CAT 2005)
Solution: As 16 + 17 + 18 + 19 = 70, therefore remainder = 0.

2. What is the remainder when 139 + 239 + 339 + 439 + ... + 1239 is divided by 39.
Solution: As we know, 1p + 2p + 3p + np is divisible by 1 + 2 + 3 + + n if p is odd. Therefore, remainder
= 0 as 1 + 2 + 3 + + 12 = 78 which is a factor of 13.

3. What is the remainder when ( + + + + + + ) is divided by 147?
(a) 0 (b) 6 (c) 6 (d) 42
Solution: As (33 + 36 + 39 + 42 + 45 + 48 + 51) = 294. If a number is divisible by 294, then it is going to be
divisible by all factors of 294 also, and 147 is a factor of 294. Hence, remainder obtained = 0.

! Any single digit number N written (p 1) times, where p is a prime number ( 2, 3, 5) is divisible by p.

Examples

1. What is the remainder when 9999999.(37 times) is divided by 19?
(a) 0 (b) 1 (c) 18 (d) 9
Solution: We know that 99999. (18 times) will be divisible by 19. Similarly, 999999 (36 times) will
be divisible by 19. Therefore, the remainder when 9999.. (37 times) is divided by 19 is 9.


74 | C A T C L A S S E S B Y A s h a n k D u b e y

2. What is the remainder when 7777.........(200 times) divided by 68?
(a) 1 (b) 9 (c) 67 (d) 57
Solution: 68 = 17 x 4
77777........(200 times) = 7777......77700 + 77 remainder with 4 = 1
77777...(200 times) = 777...77700000000 + 77777777
777777 (192 times) (Divisible by 17 as the number of 7s is a multiple of 16)
77777777 remainder with 17 = 6
The lowest number which gives remainder 1 with 5 and 6 with 17 is 57. Hence remainder = 57.

3. What is the remainder when 99999999. (425 digits) divided by 428?
(a) 427 (b) 9 (c) 1 (d) 223
Solution: 428 = 4 x 107
9999999.(106 digits) is divisible by 107. 106 x 4 = 424, it means up to 424 digits remainder is zero.
Finally, we need to divide only 9 by 107 Remainder = 9
99999999. (425 digits) = 999..99900 + 99 Remainder with 4 = 3; Therefore, the number would
satisfy the equation 107a + 9 = 4b + 3 4b 107a = 6 (a, b) = (2, 55) and the number = 223.
Therefore, remainder = 223. Option (d)

HCF With Remainders

Type: 1 The remainders are different

If numbers N1, N2, N3, N4 etc. give remainders R1, R2, R3, R4, respectively, when divided by the same
number P, then P is the HCF of (N1 R1), (N2 R2), (N3 R3), (N4 R4) etc.

Example

1. Find the greatest possible number with which when we divide 37 and 58, it leaves the respective
remainders of 2 and 3.
Solution: Since when we divide 37 and 58 by the same number then we get remainders 2 and 3
respectively.
So (37 2) and (58 3) must be divisible hence leaving the remainder zero. It means 35 and 55 both are
divisible by that number so the HCF of 35 and 55 is 5. Hence the greatest possible number is 5.

Type: 2 The remainders are the same in each case.

Example

1. Find the largest possible number with which when 60 and 98 are divided it leaves the remainders 3 in
each case.
Solution: Since 60 and 98 both leave the remainders 3 when divided by such a number.
Thus 57 = (60 3) and 95 = (98 3) will be divisible by the same number without leaving any remainder.
So the HCF of 57 and 95 is 19. Hence the largest possible number is 19.

Type: 3 The remainders are same in each case, but the value of remainder is unknown.

If numbers N1, N2, N3, N4 etc. give an equal remainder when divided by the same number P, then P is a
factor of (N1 N2), (N2 N3), (N3 N4).


75 | C A T C L A S S E S B Y A s h a n k D u b e y

Example

1. Find the largest possible number with which when 38, 66 and 80 are divided the remainders remains
the same.
Solution: In this case (since we dont know the value of remainder) we take the HCF of the differences of
the given numbers.
So the HCF of (66 38), (80 66), (80 38)
= HCF of 28, 14, 42 = 14
Hence 14 is the largest possible number which leaves same remainder (= 10) when it divides 38, 66 and
80.

2. Find the largest natural number that divides 364, 414, and 539 and leaves the same remainder in each
case.
Solution: Let the divisor be D and the remainder be R. Therefore,364 = Da + R, 414 = Db + R, 539 = Dc + R
Subtracting first equation from the second and the second equation from the third we get 50 = D (b a)
and 125 = D (c b). As D is the common factor in RHS of both the equation, it should be the common
factor on the LHS of both the equation. The HCF of 50 and 125 is 25. Therefore, the highest number can
be 25.

3. The integers 34041 and 32506, when divided by a three-digit integer n, leave the same remainder.
What is the value of n? (CAT 2000)
(a) 289 (b) 367 (c) 453 (d) 307
Solution: Let the divisor be D and the remainder be R. Therefore,
34041 = Da + R(1), 32506 = Db + R....(2)
Subtracting second equation from the first we get
D(b a) = 1535
D(b a) = 307 x 5. The possible three-digit value of D is 307.

4. Y is a set of 3 digit divisors which give same remainder when it divides 2 numbers 12714 & 13914.
What is the maximum possible number of elements in Y?
(a) 1 (b) 6 (c) 8 (d) none of these
Solution: Let the divisor be D and the remainder be R. Therefore,
12714 = Da + R...(1), 13914 = Db + R..(2)
Subtracting first equation from the second we get
D(b a) = 1200
D(b a) = 600 x 2
= 400 x 3
= 300 x 4
= 240 x 5
= 200 x 6
= 150 x 8
= 120 x 10
= 100 x 12
So, the possible 3 digit divisors are 100, 120, 150, 200, 240, 300, 400 and 600. The maximum possible
number of elements in Y is 8.



76 | C A T C L A S S E S B Y A s h a n k D u b e y

LCM with Remainders

Type: 1 When remainders are same for all the divisors.

If a number P always leaves a remainder R when divided by the numbers N1, N2, N3, N4 etc., then P =
LCM (or a multiple of LCM) of N1, N2, N3, N4.. + R.

Examples

1. What is the least possible number which when divided by 24, 32 or 42 in each case leaves the
remainder 5?
(a) 672 (b) 677 (c) 1349 (d) None of these
Solution: Since we know that the LCM of 24, 32 and 42 is divisible by the given numbers, so the required
number is
= (LCM of 24, 32, 42) + (5) = 672 + 5 = 677
Hence the least possible number is 677.

2. What is the least possible number which when divided by 21, 25, 27 and 35 leaves the remainder 2 in
each case?
(a) 4725 (b) 4727 (c) 5727 (d) None of these
Solution: Since we know that the LCM of 21, 25, 27 and 35 is divisible by the given numbers, so the
required number is
= (LCM of 21, 25, 27 and 35) + 2
= 4725 + 2 = 4727

3. What is the least number which when divided by 8, 12 and 16 leaves 3 as the remainder in each case,
but when divided by 7 leaves no remainder?
(a) 51 (b) 144 (c) 147 (d) None of these
Solution: The possible value = (LCM of 8, 12 and 16) + 3
= (48) + 3
Now put the value of m such that (48) + 3 becomes divisible by 7. So 3 x 48 + 3 = 147, which is the
least possible required number.

4. How many numbers are possible between 666 and 8888 which when divided by 24, 32 or 42 in each
case it leaves the remainder 5?
(a) 12 (b) 13 (c) 11 (d) None of these
Solution: Since we know that the LCM of 24, 32 and 42 is divisible by the given numbers, so the required
number is
= (LCM of 24, 32 and 42)n + 2
= 672n + 2
Since the form of such a number is 672n + 5, where n = 1, 2, 3, 4, .
So, the first number = 672 x 1 + 2 = 677 and the highest possible number in the given range
= 672 x 13 + 5 = 8736 + 5 = 8741
Thus the total numbers between 666 and 8888 are 13.

5. Find the lowest number which gives a remainder of 5 when divided by any of the numbers 6, 7, and 8.
Solution: The LCM of 6, 7 and 8 is 168. Hence, 168 is divisible by 6, 7 and 8. Therefore, 168 + 5 = 173 will
give a remainder of 5 when divided by these numbers.


77 | C A T C L A S S E S B Y A s h a n k D u b e y

Type: 2 When difference between the divisors and the remainders remain constant.

Examples

1. What is the least possible number which when divided by 16, 24 and 40, it leaves remainders 5, 13
and 29 respectively?
(a) 109 (b) 229 (c) 129 (d) None of these
Solution: Here difference between the divisors and the respective remainders is the same.
(16 5)
= (24 13)
= (40 29)
= 11
Hence the least possible number = (LCM of 16, 24, 40) 11 = 240 11 = 229

2. What is the least possible three digit number which when divided by 2, 3, 4, 5 and 6, it leaves
remainders 1, 2, 3, 4 and 5 respectively, but when divided by 11 leaves no remainder?
(a) 59 (b) 119 (c) 540 (d) None of these
Solution: Here again, the difference between the divisors and the respective remainders is the same.
(2 1) = (3 2)
= (4 3) = (5 4)
= (6 5) = 1
Hence the form of required number
= (LCM of 2, 3, 4, 5, 6)n 1
= 60n 1
Since the form of such a number is 60n 1, where n = 1, 2, 3, 4, .
But the number (60n 1) should be divisible by 11 hence at m = 9 the number becomes 539 which is
also divisible by 11. Thus the required number = 539.

3. What is the least possible 5 digit number which when divided by 2, 4, 6 and 8, it leaves the remainders
1, 3, 5 and 7 respectively.
(a) 10008 (b) 40017 (c) 10023 (d) 10007
Solution: The possible value = (LCM of 2, 4, 6, 8) n 1
= 24n 1
Since, the least 5 digit number is 10000. So the required number must be at least 10000. So putting the
value of m = 417, we get (10008 1) = 10007, which is the required number.

4. What is the smallest number which when divided by 9, 18, 24, it leaves remainders 5, 14 and 20
respectively?
Solution: The common difference between the divisor and the remainder is 4 (9 5 = 4, 18 14 = 4, 24
20 = 4). Now the LCM of 9, 18, and 24 is 72.
Now, 72 4
= 72 9 + 5
= 72 18 + 14
= 72 24 + 20.
Therefore, if we subtract 4 from 72, the resulting number will give remainders of 5, 14, and 20 with 9,
18, and 24.
Hence, the number = 72 4 = 68.


78 | C A T C L A S S E S B Y A s h a n k D u b e y

Successive Division
If the quotient in a division is further used as a dividend for the next divisor and again the latest
obtained divisor is used as a dividend for another divisor and so on, then it is called successive
division.

Examples

1. A number being successively divided by 3, 5 and 8 leaves remainders 1, 4 and 7 respectively. Find the
respective remainders if the order of divisors is reversed.
Solution:
Divisors Dividends Remainders
3 x
5 y 1
8 z 4
1 7

z = (8 x 1 + 7) = 15; y = (5z + 4) = (5 x 15 + 4) = 79; x = (3y + 1) = (3 x 79 + 1) = 238

Now,
Divisors Dividends Remainders
8 238
5 29 6
3 5 4
1 2

Respective remainders are 6, 4, 2.

2. What is the least possible number of 3 digits when successively divided by 2, 5, 4, 3 gives respective
remainders of 1, 1, 3, 1?
(a) 193 (b) 272 (c) 275 (d) None of these
Solution: The problem can be expressed as

Divisors Dividends Remainders
2 A
5 B 1
4 C 1
3 D 3
E 1

A = [{(((E x 3) + 1)4+ 3)5 + 1}2 + 1] = 120 E + 73
Where A is the required number
So for the least possible number E = 1
Then A = [{(((1 x 3) + 1)4+ 3)5 + 1}2 + 1] = 193

Note: We use the following convention while solving this type of problem. First we write all the divisors
as given below then their respective remainders just below them.


79 | C A T C L A S S E S B Y A s h a n k D u b e y



Where the arrow downwards means to add up and the arrow slightly upward (at the angle of 450)
means multiplication.

So we start from the right side remainder and move towards left since while writing down the divisors
and remainders.
Step 1. (1 x 4) + 3 = 7
Step 2. (7 x 5) + 1 = 36
Step 3. 36 x 2 + 1 = 73

But we are required to find a three digit number so the next higher numbers can be obtained just by
taking the multiples of the product of the divisors and then adding to it the least such number.

The next higher number = (2 x 5 x 4 x 3)m + 73 = 120m + 73
So by putting m = 1, 2, 3, we can get the higher possible numbers.
Now we need a least possible 3 digit number so we can get it by putting m = 1.
Hence the required number = 120 x 1 + 73 = 193. Option (a)

3. After the division of a number successively by 2, 3 and 5, the remainder obtained are 1, 2 and 3
respectively. What will be the remainder if 9 divides the same number?
(a) 53 (b) 75 (c) 41 (d) Cannot be determined
Solution: Write the divisors and remainders as given below


Then solve it as follows: [{(3 x 3) + 2}2 + 1] = 23
So, the least possible number is 23 and the higher numbers can be obtained as (2 x 3 x 5)m + 23 = 30m +
23.
So, the higher numbers are 53, 83, 113, 143, 173, 213, 333, etc.
But when we divide all these possible numbers by 9 we get the different remainders. So we cannot
conclude the single figure as a remainder. Option (d)

4. After the division of a number successively by 3, 4 and 7, the remainder obtained are 2, 1 and 4
respectively. What will be the remainder if 16 divides the same number?
(a) 53 (b) 75 (c) 41 (d) Cannot be determined
Solution: Write the divisors and remainders as given below


Then solve it as follows
[{(4 x 4) + 1}3 + 2] = 53
So, the least possible number is 53 and higher number can be obtained as (3 x 4 x 7)m + 53 = 84m + 53.
So, the higher numbers are 53, 137, 221, 305, 389, etc.


80 | C A T C L A S S E S B Y A s h a n k D u b e y

But when we divide all these possible numbers by 16 we get different remainders. So we cannot
conclude the single figure as a remainder. Option (d)

5. A number when divided successively by 6, 7 and 8, it leaves the respective remainders of 3, 5 and 4;
what will be the last remainder when such a least possible number is divided successively by 9, 7 and
5?
(a) 0 (b) 1 (c) 2 (d) 3
Solution: First we find the actual least possible number:


[[{(4 x 7) + 5} x 6] + 3] = [{(28 + 5) x 6} + 3] = 198 + 3 = 201
Now we divide 201 successively by 9, 7 and 5.

9 201
7 22 3 (Remainder)
5 3 1 (Remainder)
0 3 (Remainder)
So 3 is the last remainder.

6. How many numbers lie between 1000 and 10000 which when successively divided by 7, 11 and 13
leaves the respective remainders of 5, 6 and 7?
(a) 9 (b) 8 (c) 6 (d) 10
Solution: The least possible number can be obtained as

7 11 13

5 6 7
[{(7 x 11) + 6}7 + 5] = [{77 + 6}7 + 5]
= (83 x 7 + 5) = (581 + 5) = 586
The general form for the higher number is
(7 x 11 x 13)m + 586 = (1001)m + 586
So, the numbers can be obtained by considering m = 1, 2, so the first number is 1587 and the last
number is 9595 which can be attained at m = 9. So there are total 9 such numbers lying between 1000
and 10,000.

Some Special Problems:

1. How many numbers between 1 and 1000 are there such that + 3n + 5 is divisible by 121?
(a) 0 (b) 1 (c) 120 (d) 100
!
Solution: + 3n + 5 = (n 4)(n + 7) + 33. Now, 33 is divisible by 11 but not 121. n + 7 and n 4 are two
numbers with a difference of 11, therefore either both are divisible by 11 or both are not divisible by 11.
If both are divisible by 11, their product is divisible by 121 but 33 is divisible only by 11 therefore the
expression is not divisible by 121. If both are not divisible by 11, the expression is again not divisible by
121. Option (a)


81 | C A T C L A S S E S B Y A s h a n k D u b e y

2. What is the remainder when 11111.........(81 times) is divided by 81?
(a) 0 (b) 80 (c) 1 (d) 79
Solution: It can be proved that a number formed by writing any single digit 3n times will be divisible by
3n. In this case, n = 4. Therefore, the remainder is 0.

3. Let n! = 1 2 3 n for integer n 1. If p = 1! + (2 2!) + (3 3!) + + (10 10!), then
p+2 when divided by 11! leaves a remainder of (CAT 2005)
(a) 10 (b) 0 (c) 7 (d) 1
Solution: Lets start small
Let p = 1! = 1, p + 2 = 1 + 2 = 3. (p + 2) when divided by 2! leaves 1 as remainder.
Let p = 1! + (2 x 2!) = 5, p + 2 = 7. (p + 2) when divided by 3! gives remainder 1.
Let p = 1! + (2 x 2!) + (3 x 3!) = 25. P + 2 when divided by 4! gives remainder 1.
Now that we have got the patternthe answer should be 1.

Alternatively: nth term of the series = n n! = (n + 1 1) n! = (n + 1)! n! Therefore,
p = 2! 1! + 3! 2! + 4! 3! + + 11! 10! = 11! 1! p + 2 = 11! + 1 remainder by 11! = 1

4. Find the remainder when 1 2 + 2 3 + 3 4 + + 95 96 is divided by 97.
Solution: nth term of the series = n (n + 1) = n2 + n.
! !!! !!!! !(!!!) ! !!! (!!!) !" !" !"
Therefore, sum of the series (! + ) = + = = remainder
! ! ! !
by 97 = 0.

Exercise 1.5

1. What is the remainder when 51!"# is divided by 7. Find the remainder when 10!" is divided by 18?
7? (a) 1 (b) 2
(a) 1 (b) 2 (c) 4 (d) 6 (c) 4 (d) None of these

2. What is the remainder when 25!"# is divided by !"
8. What is the remainder when 33!" is divided
17? by 7?
(a) 2 (b) 4 (c) 13 (d) 15 (a) 2 (b) 4 (c) 5 (d) 6

!"
3. What is the remainder when 50!" is divided 9. What is the remainder when 10!" is divided by
by 11? 13?
(a) 3 (b) 4 (c) 6 (d) 7 (a) 0 (b) 1
(c) 2 (d) None of these
!"
4. What is the remainder when 30!" is divided
by 11? 10. Find the least number which when divided by 9,
(a) 3 (b) 5 (c) 6 (d) 9 10 and 11 gives remainders 2, 3 and 5
respectively.
5. Find the remainder when 2001000 is divided by (a) 543 (b) 992 (c) 1523 (d) 3543
17?
(a) 16 (b) 1 (c) 4 (d) none of these 11. What is the remainder when 10!" 7 is
divided by 3?
6. What is the remainder 5!" is divided by7? (a) 0 (b) 1
(a) 4 (b) 3 (c) 1 (d) 6 (c) 2 (d) None of these


82 | C A T C L A S S E S B Y A s h a n k D u b e y

12. When 629!" is divided by 21, find the 22. What is the remainder when 1515 is divided by
remainder. 65?
(a) 1 (b) 2 (c) 5 (d) 11 (a) 0 (b) 5 (c) 60 (d) 55

13. What is the remainder when 9!"" + 9!"# + 23. What is the remainder when 12!"# is divided
9!"# + + 9!"# is divided by 6? by 37?
(a) 0 (b) 1 (c) 2 (d) 3 (a) 3 (b) 11 (c) 34 (d) 36

14. What is the remainder when 2!! + 5!! + 8!! + 24. If we divide ! by 9, the remainder is 1, where
11!! + + 59!! is divided by 610? 100 200 and is an integer. How many
(a) 0 (b) 1 solutions are possible for ?
(c) 609 (d) None of these (a) 12 (b) 33 (c) 67 (d) 68

15. N is a 3 digit number which when divided by 25. Find the remainder of 5!! divided by 13, given
3,4,5 successively leaves remainders 2,1,3. that = (1!)2 + (2!)2 + .. (10!)2.
How many values can N take? (a) 1 (b) 6 (c) 8 (d) 12
(a) 21 (b) 15 (c) 14 (d) 16
26. Find the remainder when 402!" is divided by
16. What is the remainder when 120212 is divided 1000.
by 1000? (a) 696 (b) 496
(a) 120 (b) 1 (c) 296 (d) 896
(c) 0 (d) none of these 27. How many different remainders can result
when 100th power of a natural number is
17. N=9876543298765432..(82 digits). Find divided by 125?
the remainder when N is divided by 34. (a) 2 (b) 3 (c) 5 (d) 1
(a) 0 (b) 1 (c) 15 (d) 30
28. Find the remainder when 6! !! !""#$ is
18. Find the remainder when 10 + 11 + 12 + 13 divided by 13.
+ 14 + 15 is divided by 75. (a) 1 (b) 12
(a) 0 (b) 15 (c) 25 (d) 50
(c) 7 (d) 5

19. The remainder when 7!"!" is divided by 2400 29. A 50-digit number has all 7s. Find the
is remainder when the number is divided by 74.
(a) 0 (b) 1 (c) 31 (d) 1201 (a) 3 (b) 13
(c) 23 (d) 33
20. Let C be an odd positive integer greater than
99 and T = C C. Then T 2T is certainly 30. Find the remainder when (362 x 363 x 364 x
divisible by . x 454 + 362 x 454) is divided by 93!
(a) 9 (b) 24 (c) 105 (d) 1024 (a) 152753 (b) 164348
(c) 18996 (d) None of these
21. Find the remainder when 6{(p - 4)!} is divided
by p where p is a prime number greater than
2013.
(a) 0 (b) 1
(c) p 1 (d) none of these



83 | C A T C L A S S E S B Y A s h a n k D u b e y

DIGIT SUM

Given a number a, all the digits of a are added to obtain a number b. All the digits of b are added to
obtain a number c, and so on, till we obtain a single digit number p. This single digit number p is called
the digit sum of the original number a.

What is the digit sum of 1234567?
Solution: 1 + 2 + 3 + 4 + 5 + 6 + 7 + 8 + 9 = 45 4 + 5 = 9. Hence, the digit sum of the number is 9.

Note: In finding the digit-Sum of a number we can ignore the digit 9 or the digits that add up to 9. For
example, in finding the digit-sum of the number 345819, we can ignore the digits 3, 5, 1, and 9. Hence,
the digit-sum of 345819 is = 4 + 8 = 12 = 1 + 2 = 3.

Digit-Sum Rule of Multiplication: The digit-sum of the product of two numbers is equal to the digit sum
of the product of the digit sums of the two numbers.

Example: The product of 129 and 35 is 4515.
Digit sum of 129 = 3 and digit sum 35 = 8. Product of the digit sums = 3 8 = 24 Digit-sum = 6.
Digit-sum of 4515 is = 4 + 5 + 1 + 5 = 15 = 1 + 5 = 6. Digit-sum of the product of the digit sums = digit sum
of 24 = 6 Digit sum of the product (4515) = Digit-sum of the product of the digit sums (24) = 6

Finding the sum of the digits of a number

Examples

1. The digits of the number (8)24 are summed up continually till a single digit number is obtained. What is
that number
Solution: 82 = 64. Digit sum of 64 is = 1. 824 = 82 82 82 82 (12 times). Digit sums on both sides will
be the same. digit sum of 824 = digit sum of 1 1 1 1 (12 times) = 1

! If one of the multiplicands is 9, the digit sum is always 9. 235 9 = 2115 digit sum = 9.
! Digit sum of a number = Remainder obtained after dividing that number by 9. (If
remainder is zero then digit sum is 9)

2. Find the sum of the sum of the sum of the digits of 34!
Solution: 34! = 1 2 3 33 34. As one of the multiplicands is 9, the digit sum will be 9.

3. The digits of the number (8888)8888 are summed up continually till a single digit number is obtained.
What is that number?
(a) 7 (b) 2 (c) 5 (d) 1
Solution: Now, we have to divide given number by 9 to get digit sum.
9 and 8888 are co - prime to each other, therefore we can apply Eulers theorem here.
!
9 = 3! (9) = 9 1 = 6.
!

Therefore, Remainder = Remainder = Remainder = 7.



84 | C A T C L A S S E S B Y A s h a n k D u b e y

Determining if a number is a perfect square or not

Number Square Digit-Sum of the square
1 1 1
2 4 4
3 9 9
4 16 7
5 25 7
6 36 9
7 49 4
8 64 1
9 81 9

It can be seen from the table that the digit-sum of the numbers which are perfect squares will always be
1, 4, 9, or 7.

! A number will not be a perfect square if its digit-sum is not 1, 4, 7, or 9, but it may or may not
be a perfect square if its digit-sum is 1, 4, 7, or 9.

4. A 14-digit number N has among its digits two 1s, three 2s, four 3s, and five 4s. Can N be a perfect
square?
Solution: We know that the digit sum of a perfect square is always 1, 4, 7, or 9. As the digit sum of the
number is 6, it cannot be a perfect square.

UNITS DIGIT
To find the units digit of ! we only consider the units digits of the number .
To calculate the units digit of 867!"# we only consider the units digits of the number 867. Hence, we
find the units digit of 7!"# .
To find the units digit of , we only consider the units digits of the numbers and b.
To calculate the units digit of 833 x 274, we only consider the units digits of 833 and 274 i.e. 3 and 4,
respectively. Hence, we find the units digit of 3 x 4, which is 2.
To calculate the unit digit of

Value of b
Unit digit of 4k + 1 4k + 2 4k + 3 4k
1 1 1 1 1
2 2 4 8 6
3 3 8 7 1
4 4 6 4 6
5 5 5 5 5
6 6 6 6 6
7 7 9 3 1
8 8 4 2 6
9 9 1 9 1



85 | C A T C L A S S E S B Y A s h a n k D u b e y

From the above table, we can determine
1. 0, 1, 5, 6 unit place digits will remain same.
2. 2, 3, 7, 8 unit place digits will repeat after 4th power.
3. 4, 9 unit place digits will repeat after 2nd power.

To calculate units digit of numbers in the form .
! b is not a multiple of 4
We find the remainder when b is divided by 4. Let b = 4Q + R where R is the remainder when b
is divided by 4.

! The units digit of is the unit digit of .

! b is multiple of 4
It is equal to the unit digit of .

Examples

1. Find the last digit of .
(a) 6 (b) 4 (c) 2 (d) 6
Solution: 55 can be written as 4k + 3. 455 = 4(4k + 3) = 43 = 4.

2. Find the units digit of 5343 3222.
(a) 4 (b) 7 (c) 3 (d) None of these
Solution: Units digit of 53 is 7 and units digit of 3222 is 4. Hence the units digit of the expression will be
43

7 4 = 3.

3. Find the units digit of .
(a) 3 (b) 7 (c) 1 (d) 9
Solution: We find the remainder when the power is divided by 4. Therefore, we find the remainder
!"
when 15!" is divided by 4. Now 15 = 16 1 Remainder [15Odd] = Remainder [(1)Odd] = 1 = 3.
!"!" 3
Therefore, units digit of 27!" = units digit of 7 = 3.

4. Find the units digit of 13 + 23 + 33 + + 983 + 993.
(a) 0 (b) 1 (c) 3 (d) 9
Solution: Unit digit of 13, 23, 33, 43, 53, 63, 73, 83, 93 are 1, 8, 7, 4, 5, 6, 3, 2, and 9, respectively. The sum of
these units digits gives a unit digit of 5. Now these units digit will repeat 10 times each. Therefore, units
digit of the sum = 5 10 = 0.

LAST TWO DIGITS
We will discuss this in four parts:
(i) Last two digits of numbers which end in 5.
(ii) Last two digits of numbers which end in 1.
(iii) Last two digits of numbers which end in 3, 7 and 9.
(iv) Last two digits of numbers which end with an even number.

Before we start, let us first recall the binomial theorem (discussed earlier in this book), as the last two
digits concept is based upon the theorem.


86 | C A T C L A S S E S B Y A s h a n k D u b e y

!
+
= nCo + nC1 (!) + nC2 (!) + .. where nCr =
! ! !

(i) Last two digits of numbers which end in 5.

Tens digit of base Power Last two digits
Odd Odd 75
Odd Even 25
Even Odd 25
Even Even 25

Examples

1. What are the last two digits of ?
Solution: As we can see here, tens digit of base is odd and power is also odd. So, the last two digits of
the given number = 75.

2. What are the last two digits of ?
(a) 25 (b) 75 (c) 35 (d) None of these
Solution: In the given problem, tens digits of the base is odd but power is even. So, the last two digits
are 25.
(ii) Last two digits of numbers ending in 1

1. What are the last two digits of ?
(a) 01 (b) 41 (c) 21 (d) 81
Solution: 41 = 40 + 1 = C0 x 1 + C1 x 1!" x 40 + 35C2 x 1!! x 40! + , Note that all the terms
!" !" 35 !" 35

after the second term will end in two or more zeroes. The first two terms are 35C0 x 1!" and 35C1 x 1!" x
40. Now, the second term will end with one zero and the tens digit of the second term will be the
product of 35 and 4 i.e. 0. Therefore, the last two digits of the second term will be 00. The last two digits
of the first term are 01. So the last two digits of 41!" are 01.

! Multiply the tens digit of the number (4 here) with the last digit of the exponent (5 here) to
get the tens digit of required number. The unit digit is equal to one.

2. What are the last two digits of ?
(a) 41 (b) 61 (c) 31 (d) 51
Solution: Last two digits will be 41 (2 7 gives 4 and 1 is the units digit)

3. Find the last two digits of 7125747
(a) 81 (b) 71 (c) 91 (d) 31
Solution: Last two digits will be 91 (7 7 gives 9 and 1 is the units digit)

4. Find the last two digits of 71747 x 81564
(a) 91 (b) 01 (c) 11 (d) 61
Solution: Last two digits of 71!"! will be 91 and the last two digits of 81564 will be 21. Therefore, the last
two digits of 71747 x 81564 will be the last two digits of 91 x 21 = 11


87 | C A T C L A S S E S B Y A s h a n k D u b e y

(iii) Last two digits of numbers ending in 3, 7 or 9

Examples

1. What are the last two digits of ?
(a) 41 (b) 21 (c) 61 (d) 81
Solution: 19!"" = 19! !"" . Now, 192 ends in 61 (192 = 361) therefore, we need to find the last two digits
of 61 !"" .
Once the number is ending in 1 we can straight away get the last two digits with the help of the previous
method. The last two digits are 41 (6 4 = 24, so the tens digit will be 4 and last digit will be 1)

2. What are the last two digits of 37296?
(a) 21 (b) 31 (c) 41 (d) 51
296
Solution: 37 = 37 ! !"
. Now 37 ends in 61. (Last two digits of 37! = Last two digits of 13! = 132 x 132
!

= 169 x 169 = xxx61). Therefore, we need to find the last two digits of 61 !" . By the previous method, the
last two digits of 61 !" = 41.

So heres the rule for finding the last two digits of numbers ending in 3, 7 and 9:

! Convert the number till the number gives 1 as the last digit and then find the last two digits
according to the previous method.

3. What are the last two digits of ?
(a) 73 (b) 83 (c) 93 (c) 03
Solution: 53 !"" = 53!"! x 53 = 53! !" x 53 = 09 09 !"
x 53 = 81 !"
x 53 = 41 x 53 = 73.

Last two digits of numbers ending in 2, 4, 6 or 8

There is only one even two-digit number which always ends in itself (last two digits) 76 (try proving
it!); i.e. 76 raised to any power gives the last two digits as 76. Therefore, our purpose is to get 76 as last
two digits for even numbers.
Here if you need to multiply 76 with 2n, then you can straightaway write the last two digits of 2n because
when 76 is multiplied with 2n the last two digits remain the same as the last two digits of 2n.
Therefore, the last two digits of 76 26 will be the last two digits of 26 = 64. Note that this concept works
only for powers of 2 2.


! Last two digits of = 76

! Last two digits of = 24
! The last two digits of , , + , , + . Will always be the

same. Last two digits of , , , , , will always be 44.



Examples

1. What are the last two digits of ?
(a) 12 (b) 24 (c) 28 (d) 64
Solution: 4!"# = 2!"# = 2!" !" x 2! = 76 x 64 = 64.


88 | C A T C L A S S E S B Y A s h a n k D u b e y

2. What are the last two digits of () ?


(a) 26 (b) 46 (c) 76 (d) 36
Solution: (64)!"# = 2! !"# = 2!"!# = 2!" !"! x 2! = 24 x 64 = 36.

3. What are the last two digits of ?
(a) 36 (b) 66 (c) 76 (d) 86
!"" !"" !" !" ! ! !"
Solution: 2 37 = 2 x 2 x 37 = 76 x 56 x 21 = 56 x 21 = 76.

4. What is the tens digit of 56283?
(a) 1 (b) 3 (c) 6 (d) 7
Solution: 56283 = (23 7)283 = 2849 7283 = (210)84 29 (74)70 73 = 76 12 (01)70 43 = 16.
Tens digit of 56!"# = 1

LAST NON ZERO DIGIT IN A FACTORIAL

Find the last non zero digit of n! : Last non zero digit of (ten consecutive integers)

= (10a + 1)(10a + 2)(10a + 3)(10a + 4)(10a + 5)(10a + 6)(10a + 7)(10a +8)(10a +9)(10a +10)
= (10a + 1)(10a + 2)(10a + 3)(10a + 4)5(2a + 1)(10a + 6)(10a + 7)(10a +8)(10a +9)5(2a +2)
= (10a + 1)(10a + 2)(10a + 3)2(5a + 2)5(2a + 1)2(5a + 3)(10a + 7)(10a +8)(10a +9)5(2a +2)
= 100(10a + 1)(10a + 2)(10a + 3)(5a + 2)(2a + 1)(5a + 3)(10a + 7)(10a +8)(10a +9)(2a +2)
= 100(2a + 1) (2a +2)(10a + 1)(10a + 2)(10a + 3)(5a + 2)(5a + 3)(10a + 7)(10a +8)(10a +9)
{Now, we can remove both zeroes}
= Last non zero digit of (2a + 1) (2a +2)(1)(2)(3)(7)(8)(9)(5a + 2)(5a + 3)
= Last non zero digit of 4{25a(a + 1) +6}(2a + 1)(2a + 2)
= 4 x Last non zero digit of (2a + 1)(2a + 2)

Examples

1. What is the last non zero digit of 20!
Solution: Using above property we can write:
! !
1 x 2 x3 x 4 x5 x6 x7 x8 x9 x10 = 4 x remainder
!"
!!
11 x 12 x13 x 14 x15 x16 x17 x18 x19 x20 = 4 x remainder
!"
!
So last non zero digit of 20! = 4 x last non zero digit of 4! = last non zero digit of 6 x 4 = 4

! LNZ (10n!) = LNZ ( ) x LNZ (2n!)

2. What is the last non zero digit of 100!
Solution: Last non zero digit of 100! = LNZ (100!) = LNZ (4!" ) x LNZ (20!)
From the above example, we know last non zero digit of 20!, which is 4. LNZ (100!) = LNZ (6 x 4) = 4

3. Find the last non zero digit of 125!
Solution: Now see that 125 is not a multiple of 10; hence we can apply the above same method up to
120! and then multiply the result with last non zero digit of the product of remaining numbers i.e. 121
to 125.


89 | C A T C L A S S E S B Y A s h a n k D u b e y

So, the last non zero digit of 125! = LNZ(125!) = LNZ(4!" ) x LNZ (24!) x LNZ(121 x 122 x 123 x 124 x 125)
= LNX(6 x 4! ) x LNZ (4!) x LNZ(21 x 22 x 23 x 24) x LNZ (121 x 61 x 123 x 31) = LNZ(6 x 4 x 4 x 3) = 8.

LAST TWO DIGITS IN A FACTORIAL

With the use of above concept, we can directly derive formula for these types of questions.
Last two non zero digits of (10n!) = Last two non zero digits of (44! ) x Last two non zero digits of
(2n!)

! LTNZ (10n!) = LTNZ ( ) x LTNZ (2n!)

Examples

1. What is the last two non-zero digits of 100!
(a) 44 (b) 36 (c) 76 (d) 64
Solution: Last two digits of 100! = 4410 x last two non-zero of 20!
= 220 x 1110 x 442 x last two non-zero digits of 4!
= 76 x 01 x 36 x 24 =76 x 01 x 64 = 64; Option (d)

2. What is the last two non zero digits of 500!
(a) 64 (b) 44 (c) 36 (d) None of these
!"
Solution: Last two non zero digits of 500! = 44 x last two non zero digits of 100!
= Last two non zero digits of 76 x 01 x 64 = 64

Exercise 1.6

1. N = 70! 69! 68! ..... 3! 2! 1! 5. How many digits cannot be the units digit of
Which of the following represents the 147th the product of 3 three-digit numbers whose
digit from the right end of N? sum is 989?
(a) 2 (b) 0 (c) 5 (d) 7
(a) 2 (b) 4 (c) 1 (d) 3

2. If N = 1! 2! + 3! 4! +..+ 47! 48! + 49!, then 6. V is a 56 digit number. All the digits except the
what is the unit digit of ! ? 32nd from the right are the same. If V is divisible
(a) 0 (b) 9 (c) 7 (d) 1 by 13, then which of the following can never be
the units digit of V?
3. What is the digit at the hundreds place of the (a) 4 (b) 7
(c) 1 (d) Both 4 and 7
number N = 4536?

(a) 1 (b) 0 (c) 6 (d) 5 !" !"
7. What is the unit digit of 7!" + 4!" ?

(a) 3 (b) 5 (c) 7 (d) 9
4. How many ordered pairs (P, Q) are there such
that the units digits of PP and QQ are the same? 8. Find the remainder when 375 is divided by 5.
P and Q are natural numbers less than 10 and (a) 3 (b) 4 (c) 1 (d) 2
are not necessarily distinct.
(a) 12 (b) 15 (c) 10 (d) 14 !!
9. Find the unit digit of 7! + 7

(a) 8 (b) 1
(c) 4 (d) None of these


90 | C A T C L A S S E S B Y A s h a n k D u b e y

10. What is the first non-zero integer from the 15. What is the remainder when (1)7+ (71)77 +
right in 83301957 + 83701982? (771)777 + (7771)7777 + (77771)77777 +
(a) 3 (b) 1 (c) 9 (d) none of these (777771)777777 is divided by 50?

(a) 46 (b) 6 (c) 56 (d) 5
11. If N = (13)1! + 2! + 3! + ..+ 13! + (28)1! + 2! + 3!....+ 28! +
(32)1! + 2! + 3! + ....+ 32!+ (67) 1! + 2! + 3! + ....+ 67! then
the unit digit of N is 16. Find the tens digit of (43)2012?
(a) 4 (b) 8 (a) 0 (b) 1 (c) 3 (d) 9
(c) 2 (d) none of these
17. What is the unit digit of 1!! + 2!! + 3!! + ....+
12. The squares of the natural numbers are 49!"! + 50!"! ?
written in a straight line 149162536 to form a (a) 1 (b) 3 (c) 5 (d) 7
200-digits number. What is the 100th digit
from the left? 18. What are the last two non zero digits of 20!
(a) 2 (b) 5 (a) 24 (b) 44
(c) 6 (d) 9 (c) 64 (d) 84

13. What are the last two digits of (3844)47? 19. Find the last two digits of 11!" - 9.
(a) 54 (b) 34 (c) 44 (d) 64 (a) 52 (b) 72
(c) 92 (d) 02


14. What is the remainder when (37)2012 is divided !""#
20. What are the last two digits of 2! ?
by 25? (a)16 (b) 36
(a) 31 (b) 21 (c) 11 (d) 1 (c) 56 (d) 76

Base System
Number of digits or symbols used in a particular system of writing the numbers is known as base. For
example, in decimal system, we use 10 digits (0 9) to write the numbers, and hence the base is 10.
Similarly, we use 2 digits (0 1) to write the numbers, base = 2.

In our decimal system of writing the numbers, we use 10 digits (0 9). So we start writing the numbers
from 0, then 1, 2, 3, 4, 5, 6, 7, 8, 9. For bigger numbers, we are required to take resort to two digit
numbers starting from 10, then 11, 12, 13, .and so on. So, basically, number formation is nothing but
repeating the cycle.

Consider now a system of writing the number that has only 8 digits (0 7). To generate counting
numbers now, we start writing the numbers from 0, then 1, 2, 3, 4, 5, 6, 7. Once we have written 7, we
do not have single digit to take our counting forward. So to form a number larger than 7, we are
required to take resort to two digit numbers here only. So next number will be 10, then 11, 12, 13, 14,
15, 16, 17. We dont have 18 and 19 in the system, so next number will be 20, 21, .and highest two
digit number will be 77.

Similarly, had there been only 6 digits (0 5) to write down the numbers, numbers would have been
formed like this; 0, 1, 2, 3, 4, 5, 10, 11, 15, 20, 21, 23, 24, 25, 39.,54, 55,..100, and so on.


91 | C A T C L A S S E S B Y A s h a n k D u b e y

Lets try to understand the concept of base system through speedometer attached to vehicles. Consider
different cases now that there is a Vehicle A having speedometer with 0 to 9 digits, another vehicle B
having speedometer with only 7 digits from 0 to 6, and vehicle C with only 4 digits from 0 to 3.

Actual distance covered and data shown by both the speedometer is given below.

Actual Distance Speedometer A (0 9) Speedometer B (0 6) Speedometer C (0 3)
0 0 0 0
1 1 1 1
2 2 2 2
3 3 3 3
4 4 4 10
5 5 5 11
6 6 6 12
7 7 10 13
8 8 11 20
9 9 12 21
10 10 13 22

Conversion from any base to base ten
The number ! is converted to base 10 by finding the value of the number. i.e. ! =
! + ! + ! + ! + ! + f

Convert (21346)5 to base 10.
Solution: (21346)5 = 2 625 + 1 125 + 3 25 + 4 5 + 5 = 1475

Conversion from base 10 to any base
A number written in base 10 can be converted to any base 'b' by first dividing the number by 'b', and
then successively dividing the quotients by 'b'. The remainders, written in reverse order, give the
equivalent number in base 'b'.

Write the number 25 in base 3.
Solution:


Writing the remainders in reverse order the number 25 in base 10 is the number 221 in base 3
Conversion from Octal to Binary
The following table helps in quick conversion.

Octal Digit 0 1 2 3 4 5 6 7
Binary 000 001 010 011 100 101 110 111
Equivalent


92 | C A T C L A S S E S B Y A s h a n k D u b e y

Examples

1. Convert to binary.
Solution:


Hence 273 ! = 010111011 !

2. Convert . to binary.
Solution:



Binary to octal conversion
This is the inverse process of the previous conversion. In this process make the triplets starting from the
right. If the number of digits in the binary representation is not in group of 3 then we associate either 1
or 2 zeroes at the left.

3. Convert to octal.
Solution:


Hence 100011010110 ! = 4326 !

Addition, subtraction and multiplication in bases

Add the numbers (4235)7 and (2354)7

The numbers are written as



The addition of 5 and 4 (at the units place) is 9, which being more than 7 would be written as 9 = 7 1
+ 2. The Quotient is 1 and the remainder is 2. The Remainder is placed at the units place of the answer
and the Quotient gets carried over to the ten's place. We obtain



At the tens place: 3 + 5 + 1 (carry) = 9


93 | C A T C L A S S E S B Y A s h a n k D u b e y

Multiply (43)8 (67)8
Solution:
7 3 = 21 = 8 2 + 5 we write 5 and carry 2 base (8)
7 4 + 2 (carry) = 30 = 8 3 + 6 we write 6 and carry 3 base (8)
6 3 = 18 = 8 2 + 2 we write 2 and carry 2 base (8)
6 4 + 2 (carry) = 26 = 8 3 + 2 we write 2 and carry 3 base (8)


For subtraction the procedure is the same as for any ordinary subtraction in base 10 except for the fact
that whenever we need to carry to the right we carry the value equal to the base.

Subtract 45026 from 51231 in base 7.
Solution: In the units column since 1 is smaller than 6, we carry the value equal to the base from the
number on the left. Since the base is 7 we carry 7. Now, 1 + 7 = 8 and 8 6 = 2. Hence we write 2 in the
units column. We proceed the same way in the rest of the columns.



Examples

1. In what base is the equation 53 15 = 732 valid?
Solution: Let the base be n. 53 ! = 3 + 5n, 15 ! = n + 5, 732 ! = 7n2 + 3n + 2 (3 + 5n) (n + 5) = 7n2
+ 3n + 2 n = 13.

2. A positive whole number M less than 100 is represented in base 2 notation, base 3 notation, and base
5 notation. It is found that in all three cases the last digit is 1, while in exactly two out of the three
cases, the leading digit is 1. M equals (CAT 2003)
Solution: Whenever we change a number from base 10 to any other base, the units digit is the first
remainder when the number is divided by that base. Therefore, M when divided by 2, 3 and 5 gives
remainder 1 in each case. LCM of 2, 3 and 5 is 30. Therefore, M = 30k + 1 = 31, 61 and 91. Out of these 3
numbers, only the number 91 satisfies the second criterion of leading digit (last remainder).

3. Solve in base 7, the pair of equations 2x - 4y = 33 and 3x + y = 31, where x, y and the coefficients are in
base 7.
Solution: Working in base 7:
2x - 4y = 33 --- (1)
3x + y = 31 --- (2)
Multiply (2) by 4, noting that 3 4 = 15 in base 7 and 31 4 = 154 in base 7. We obtain
15x + 4y = 154 --- (3)
Adding (1) and (3) we get 20x = 220 x = 11 y = 2.


94 | C A T C L A S S E S B Y A s h a n k D u b e y

Exercise 1.7

1. How many two digit numbers in base 9 get 8. How many three digit numbers in base 11 are
reversed when written in base7? possible such that when they are expressed in
(a) 0 (b) 1 base 9, have their digit reversed?
(c) 2 (d) 3
(a) 1 (b) 2

2. How many numbers have exactly 3 digits (c) 3 (d) more than 3
when expressed in either base 5 or base
11? 9. The inhabitants of planet ZIRCON use a
(a) 4 (b) 5 number system, which is similar to the decimal
(c) 6 (d) more than 6 system used here on EARTH, except that it has
been eleven distinct digits instead of our ten
3. Given below is the sequence of all the
digits. The extra digit is an alien digit theta ()
natural numbers which dont contain the
digit 0. 1, 2, 3, 4, 5, 6, 7, 8, 9, 11, . What which is inserted between the digits 5 and 6.
is the 200th term of this sequence? Now 6 5 = 2 and theta is the digit which is
(a) 238 (b) 236 equidistant from 5 and 6 i.e. - 5 = 6 - = 1. All
(c) 240 (d) 242 the algebraic signs and operations carry the
same meaning as in the usual sense. What is
4. The number 2324X1013 ! is converted to
the decimal equivalent of the two digit number
base 10 and then divided by 6. It leaves no
9?
remainder. What could be the value of X?
(a) 0 (b) 1 (a) 102 (b) 108
(c) 2 (d) 3 (c) 106 (d) 105

5. A number N written in base b is 10. In a writing code dn stands for single digit d
represented as a two-digit number A2, coming n times consecutively. For example,
where A = b - 2. What would N be
356274 would mean the number 33333667777.
represented as when written in base b - 1?
(a) 98 (b) 100 For what ordered triplet (x, y, z) 2! 3! 5! +
(c) 102 (d) 101 3! 5! 2! = 5372835173?
(a) 0 (b) 2
6. My ABN AMRO ATM Pin is a four-digit (c) 4 (d) None of these
number. My HDFC BANK ATM Pin is also a
four-digit number using the same digits, in a 11. The number 6162 in base 10 is written as
different order, as those in my ABN AMRO (222)b. Then, the base b is equal to
Pin. When I subtract the two numbers, I get (a) 60 (b) 55 (c) 45 (d) 42
a four-digit number whose first three digits

are 2, 3 and 9. What is the unit digit of the
difference? 12. If in some base x, 563 ! + 544 ! + 433 !
(a) 2 (b) 4 (c) 6 (d) 3 = 2203 ! then x is equal to
(a) 6 (b) 7
7. abcd is a four-digit number in base 7 such (c) 9 (d) 11
that 2(abcd) = bcda. Find a
(a) 1 (b) 2
(c) 3 (d) 1 or 2


95 | C A T C L A S S E S B Y A s h a n k D u b e y

13. The number 2006! is written in base 22. 17. If the number 3402 is converted from base 10
How many zeroes are there at the end? to base x, it becomes 12630. What is the value
(a) 500 (b) 450 of ?
(c) 200 (d) 199 (a) 6 (b) 7
(c) 8 (d) 9
14. Two natural numbers a and b are given in
base 10. The number a can be written as 18. First 100 natural numbers in the decimal
212 in base b and 128 in base b + 2. The system are converted to base 6 and their
value of a + b in base 10 is product is found. Find the number of
(a) 219 (b) 125 consecutive zeroes at the end of the product
(c) 114 (d) 107 as counted in the base 6 system.
(a) 120 (b) 124
15. For single digit numbers a, b and c, ! = (c) 130 (d) 136
! . what is the value of a + b + c?
(a) 8 (b) 11 19. If 52 ! = 169 !! and a and b differ by two,
(c) 16 (d) cannot be determined then find the value of (a + b).
(a) 6 (b) 8 (c) 10 (d) 12
16. A = 2586 5171. How many digits would be
there in A when it is represented in base 5? 20. When (55)10 is represented in base 25 then the
(a) 172 (b) 171 expression is
(c) 87 (d) 86 (a) (25)25 (b) (35)25
(c) (55)25 (d) none of these

Divisibility Rules depend upon the Base
In base 10, we can find out if the last (or unit) digit of a number is divisible by 5 (then the number is
divisible by 5). We call it divisibility rule of 5 in base 10. But the same divisibility rule of 5 will not
necessarily work in the other bases because the divisibility rules depend on the base.

Lets create the divisibility rules of some numbers in different bases:
Sum of the digits:
In base 10, any two digit number can be written in the form of (10 + ).
(10 + ) = (9 + + b). 9 divided by 9 remainder is 0. So, remainder is based on (a + b), which is sum
of the digits. Similarly
10! Divided by 9 the remainder is 1
10! Divided by 9 the remainder is 1
10! Divided by 9 the remainder is 1
10! Divided by 9 the remainder is 1

10 ! Divided by 9, the remainder is 1, where x is a whole number.
Any 4 digit number !" divided by 9
= {A 10! + B 10! + C 10 + D} divided by 9.
= {A 10! divided by 9 + B 10! divided by 9 + C 10 divided by 9 + D divided by 9}
As we know, 10 ! divided by 9 the remainder is 1.
= A (+ 1) + B (+ 1) + C (+ 1) + D = (A + B + C + D), which is sum of the digits.


96 | C A T C L A S S E S B Y A s h a n k D u b e y

Whether a number is divisible by 9 in base 10 depends upon the sum of the digits of the number. Hence,
in base 10, if the sum of the digits of a number is divisible by 9 then so is the number. Thats the
divisibility rule of 9 in base 10.

Example
What is the remainder if 7632 is divided by 9?
(a) 0 (b) 1 (c) 8 (d) 2
Solution: 7632 divided by 9
= {7 x 10 ! + 6 x 10 ! + 3 x (10) + 2} divided by 9
= {7 10! divided by 9 + 6 10! divided by 9 + 3 10 divided by 9 + 2 divided by 9}
= {7 (+ 1) + 6 (+ 1) + 3 (+ 1) + 2 (+ 1)} = {7 + 6 + 3 + 2} = 18 divided by 9 remainder is 0.

In base 10, the divisibility rule of the form sum of digits applies to (10 1) = 9 and the same rule also to
each factor of (10 1) = 9 because each factor of 9 except 1 generates remainder 1 with 10 ! , where is
a whole number.
In general, for any base , ! divided by ( 1) leaves the remainder 1, where is a whole number.
Therefore, we can apply the divisibility rule of the form Sum of the digits for ( 1) in base .
Furthermore, for any base , ! divided by remainder = 1, where is factor of ( 1) other than 1.
Therefore, we can apply the divisibility rule of the form Sum of the digits for each factor of ( 1) in
base .

! In base , a number is divided by ( 1) leaves the same remainder as the sum of the digits of
the number is divided by ( 1).
! In base , a number divided by a factor of ( 1) leaves the same remainder as the sum of the
digits of the number is divided by that factor of ( 1).

In base 19, a number is divisible by (19 1) = 18 if the sum the digits of the number is divisible by 18.

In base 13, the divisibility rule Sum of the digits applies to all the factors of (13 1) = 12. The factors
of 12 are 1, 2, 3, 4, 6 and 12. Therefore, if the sum of the digits of a number is divisible by then so is
the number where is a factor of 12.

Examples
1. Is divisible by 13?
Solution: As (14 1) = 13, we can apply the divisibility rule Sum of the digits
8495 !" divided by 13 = (8 + 4 + 9 + 5) divided by 13 = 26 divided by 13 remainder = 0
Hence, 8495 !" is divisible by 13.

2. Is divisible by 5?
Solution: 5 is a factor of (16 1) = 15. Therefore we can apply the divisibility rule Sum of the digits
here. 3732 !" divided by 5 = (3 + 7 + 3 + 2) divided by 5 = 15 divided by 5 remainder = 0.
Hence, 3732 !" is divisible by 5.

3. The positive integer n divides . Find the total number of possible values of if is factor of
30.
(a) 4 (b) 6 (c) 5 (d) None of these
Solution: Here is factor of (31 1) = 30. So, we can apply the divisibility rule Sum of the digits to test
the divisibility of 3030 !" by . The sum of the digits = 3 + 0 + 3 + 0 = 6. The factors of 30 are 1, 2, 3, 5,


97 | C A T C L A S S E S B Y A s h a n k D u b e y

6, 10, 15 and 30. Out of these factors only 1, 2, 3 and 6 divide the sum 6. Hence, the total number of
possible values of n = 4.

4. What is the remainder when is divided by 12?
(a) 0 (b) 1 (c) 2 (d) None of these
Solution: The remainder when 7373 !" is divided by 12 can be obtained by dividing the sum of digits of
7373 !" by 12. (7 + 3 + 7 + 3) divided by 12 = 20 divided by 12 remainder = 8.

Alternate digits:
Lets try to invent the divisibility rule of 11 in base 10.
10! Divided by 11 the remainder is 1
10! Divided by 11 the remainder is 1 or 10
10! Divided by 11 the remainder is 1
10! Divided by 11 the remainder is 1 or 10

10 ! Divided by 11 the remainder is 1. Where, x is an odd number.
10 ! Divided by 11 the remainder is 1 or 10. Where, x is an even number.

Any 4 digit number !" divided by 11
= {A 10! + B 10! + C 10 + D} divided by 9.
= {A 10! divided by 11 + B 10! divided by 11 + C 10 divided by 11 + D divided by 11}
= A ( 1) + B (+ 1) + C ( 1) + D = {(B + D) (A + C)}

As, you can see whether a number is divisible by 11 in base 10 depends upon (sum of the digits in odd
places sum of the digits in even places). Thats the divisibility rule of 11 in base 10.
In general, for any base , ! divided by ( + 1) the remainders are + 1, 1, + 1, 1, .Therefore, we can
apply the divisibility rule of the form Alternate Digits for (n + 1) in base n. Furthermore, for any base ,
the ! divided by m remainders are + 1, 1, + 1, 1, where is a factor of ( + 1) other than 1.
Therefore, we can apply the divisibility rule of the form Alternate Digits for each factor of ( + 1) in
base .

! In base , a number divided by ( + 1) leaves the same remainder as the (sum of the digits in odd
places sum of the digits in even places) is divided by ( + 1).
! In base , a number divided by a factor of ( + 1) leaves the same remainder as the (sum of the
digits in odd places sum of the digits in even places) is divided by that factor of ( + 1).

In base 16, a number is divisible by (16 + 1) = 17 if the (sum of the digits in odd places sum of
the digits in even places) divisible by 17.

In base 29, the divisibility rule Alternate Digits applies to all the factors of (29 + 1) = 30. The
factors of 30 are 1, 2, 3, 5, 6, 10, 15 and 30. Therefore, if the (sum of the digits in odd places
sum of the digits in even places) is divisible by a factor of 30 then the number is also divisible by
that factor of 30.

Examples
1. What is the remainder when is divided by 3?
(a) 0 (b) 2 (c) 1 (d) None of these


98 | C A T C L A S S E S B Y A s h a n k D u b e y

Solution: 3 is a factor of 8 + 1 = 9, therefore we can apply the divisibility rule Alternate Digits here. {(1 +
3) (2 + 5)} = (4 7) = 3 is divided by 3 remainder is 0.

2. What is the remainder when is divided by 13?
(a) 1 (b) 12 (c) 5 (d) 8
Solution: The remainder when 8484 !" is divided by 13 can be obtained by dividing the difference
between the sum of the digits of 8484 !" in odd places and that in even places by 13. [(4 + 4) (8 + 8)]
= 8 is divided by 13. Remainder = 5.

Lets now arrive at the divisibility rule of 11 in base 10.
10! Divided by 11 the remainder is 1
10! Divided by 11 the remainder is 1 or 10
10! Divided by 11 the remainder is 1
10! Divided by 11 the remainder is 1 or 10

10! Divided by 11 the remainder is 1
10! Divided by 11 the remainder is 1 or 10
10!" Divided by 11 the remainder is 1

10 ! Divided by 7 the remainder is 1. Where, x is an even multiple of 3.
10 ! Divided by 7 the remainder is 1 or 10. Where, x is an odd multiple of 3.
Any 12 digit number !" divided by 11
= {ABC 10! + 10! + PQR 10! + XYZ} divided by 11.
= {ABC 10! divided by 11 + 10! divided by 11 + PQR 10! divided by 11 + XYZ divided by 11}
= ABC ( 1) + MNO (+ 1) + PQR ( 1) + XYZ
= {(MNO + XYZ) (ABC + PQR)}
As, you can see whether a number is divisible by 7 in base 10 depends upon (sum of the triplets in odd
places sum of the triplets in even places).

! In base 10, a number is divided by 11 leaves the same remainder as the sum of the triplets in
odd places sum of the triplets in even places is divided by 11.

Examples
1. What is the remainder when 12478375 is divided by 11?
(a) 0 (b) 3 (c) 8 (d) 9
Solution: Here, we can apply the rule of 11. Given number can be written in the form of 012478357 =
{375 + 012 478} = 91. Now, 91 is divided by 11. Remainder = 3 or 8.

2. N= 123123.300 digits. Find the remainder when N is divided by 11?
(a) 10 (b) 0 (c) 1 (d) none of these
Solution: In the given number, there are 100 triplets. Out of 100 triplets 50 will come at odd places and
50 will come at even places. {123 x 50 123 x 50} = 0. Remainder = 0.

Rule for 7 and 13 in base 10
10! Divided by 7 the remainder is 3
10! Divided by 7 the remainder is 2
10! Divided by 7 the remainder is 1 or 6


99 | C A T C L A S S E S B Y A s h a n k D u b e y


10! Divided by 7 the remainder is 1
10! Divided by 7 the remainder is 1 or 6
10!" Divided by 7 the remainder is 1
10 ! Divided by 7 the remainder is 1. Where, x is an even multiple of 3.
10 ! Divided by 7 the remainder is 1 or 6. Where, x is an odd multiple of 3.
Any 12 digit number !" divided by 7
= {AB C 10! + 10! + PQR 10! + XYZ} divided by 7.
= {ABC 10! divided by 7 + 10! divided by 7 + PQR 10! divided by 7 + XYZ divided by 7}
= ABC ( 1) + MNO (+ 1) + PQR ( 1) + XYZ
= {(MNO + XYZ) (ABC + PQR)}

Clearly, whether a number is divisible by 7 in base 10 depends upon (sum of the triplets in odd places
sum of the triplets in even places). Thats the divisibility rule of 7 in base 10.

Rule for 13 in base 10
10! Divided by 13 the remainder is 10
10! Divided by 13 the remainder is 9
10! Divided by 13 the remainder is 1 or 12

10! Divided by 13 the remainder is 1
10! Divided by 13 the remainder is 1 or 12
10!" Divided by 13 the remainder is 1

10 ! Divided by 13 the remainder is 1. Where, x is an even multiple of 3.
10 ! Divided by 13 the remainder is 1 or 6. Where, x is an odd multiple of 3.
Any 12 digit number !" divided by 13
= {AB C 10! + 10! + PQR 10! + XYZ} divided by 13.
= {ABC 10! divided by 13 + 10! divided by 13 + PQR 10! divided by 13 + XYZ divided by 13}
= ABC ( 1) + MNO (+ 1) + PQR ( 1) + XYZ
= {(MNO + XYZ) (ABC + PQR)}
As, you can see whether a number is divisible by 13 in base 10 depends upon (sum of the triplets in odd
places sum of the triplets in even places). Thats the divisibility rule of 13 in base 10.

! In base 10, a number is divided by 7 or 13 leaves the same remainder as (the sum of the
triplets in odd places sum of the triplets in even places) is divided by 7 or 13.

Examples
1. What is the remainder when 12478375 is divided by 13?
(a) 0 (b) 3 (c) 8 (d) 9
Solution: Here, we can apply the rule of 11. Given number can be written in the form of 012478357 =
{375 + 012 478} = 91. Now, 91 is divisible by 13. Remainder = 0.

2. N= 123123.300 digits. Find the remainder when N is divided by 7?
(a) 10 (b) 0 (c) 1 (d) none of these
Solution: In the given number, there are 100 triplets. Out of 100 triplets 50 will come at odd places and
50 will come at even places. {123 x 50 123 x 50} = 0. Remainder = 0.


100 | C A T C L A S S E S B Y A s h a n k D u b e y

Rule for 37 in base 10
10! Divided by 37 the remainder is 10
10! Divided by 37 the remainder is 26
10! Divided by 37 the remainder is 1

10! Divided by 37 the remainder is 1
10! Divided by 37 the remainder is 1
10!" Divided by 37 the remainder is 1
10 ! Divided by 37 the remainder is 1. Where, x is a multiple of 3.
Any 12 digit number !" divided by 37
= {AB C 10! + 10! + PQR 10! + XYZ} divided by 13.
= {ABC 10! divided by 13 + 10! divided by 13 + PQR 10! divided by 13 + XYZ divided by 13}
= ABC (+1) + MNO (+ 1) + PQR (+ 1) + XYZ
= {ABC + MNO + PQR + XYZ}

Clearly, whether a number is divisible by 37 in base 10 depends upon the sum of the triplets. Thats the
divisibility rule of 37 in base 10.

! In base 10, a number is divided by 37 leaves the same remainder as the sum of the triplets is
divided by 37.

Examples

1. N= 123123.300 digits. Find the remainder when N is divided by 7?
(a) 10 (b) 0 (c) 16 (d) none of these
Solution: In the given number, there are 100 triplets. Sum of the triplets = {123 x 100} =12300. Again,
sum of the triplets of (12300) = 300 + 12 = 312. 312 is divided by 37. Remainder = 16.

2. What is the remainder when 12478375 is divided by 11?
(a) 0 (b) 3 (c) 8 (d) 9
Solution: Here, we can apply the rule of 11. Given number can be written in the form of 012478357 =
{375 + 012 478} = 91. Now, 91 is divided by 11. Remainder = 3 or 8.

Rule for 2 in base 10
Lets invent the divisibility rule of 2 in base 10.
10! Divided by 2 the remainder is 1
10! Divided by 2 the remainder is 0
10! Divided by 2 the remainder is 0
10! Divided by 2 the remainder is 0

10 ! Divided by 2 the remainder is 0. Where, x is a positive integer.
!"
Any 3 digit number !" divided by 2
= {A 10! + B 10 + C 10 } divided by 2.
= {A 10! divided by 2 + B 10 divided by 2 + C divided by 9}
= A (0) + B (0) + C (0) + D
= (0 + 0 + 0 + D) = D, which is last digit of the given number.


101 | C A T C L A S S E S B Y A s h a n k D u b e y

As, you can see whether a number is divisible by 2 in base 10 depends upon the last digit of the number.
Hence, in base 10, if the last digit of a number is divisible by 2 then so is the number. Thats the
divisibility rule of 2 in base 10.
2 is a factor of 10! therefore it is also a factor of 10(!!!) where n is a positive integer; so divide only the
last digit of a number to test the divisibility of the number by 2 in base 10; if the last digit is divisible by 2
then so is the number.
Similarly, 5 is a factor of 10! therefore it is also a factor of 10(!!!) where n is a positive integer; so
divide only the last digit of a number to test the divisibility of the number by 5 in base 10; if the last digit
is divisible by 5 then so is the number.
In short, if any number is a factor of 10! then divide the last one digit of a number by x to test the
divisibility of the number by in base 10; if last digit is divisible by x then so is the number.
In general, in base, if is a factor of then divide the last one digit of a number by to test the
divisibility of the number by in base ; if the last digit is divisible by then so is the number.
4 is not a factor of 2 but it is factor of 10! therefore it is also factor of 10!! ! where is a positive
integer.

Is 1564 divisible by 4?
1564 = (1500 + 64) divided by 4
= [{1500 divided by 4 + 64 divided by 4}]
= [{0 + 0}] = 0.

As we can see 4 is a factor of 10! therefore it is also a factor of 1500; so divide only the last two digits of
a number to test the divisibility of the number by 4 in base 10.
Similarly, 20 is a factor of 10! therefore it is also a factor of 10!!! where n is a positive integer; so
divide only the last two digits of a number to test the divisibility of the number by 20 in base 10.
To summarize, in base 10, if a number is factor of 10! then it is also a factor of 10!! ; so divide the last
digits of a number to test the divisibility of the number by in base 10.

! In base , a number divided by a factor of leaves the same remainder as the last digit of the
number is divided by that factor of .
! In base , a number divided by a factor of leaves the same remainder as the last digits of
the number are divided by that factor of .

# In base 10, a number when divided by 5 leaves the same remainder as the last digit of the
number is divided by 5.

# In base 12, a number when divided by 36 leaves the same remainder as the last two digits of the
number are divided by 36.

Examples

1. What is the remainder when is divided by 16?
(a) 4 (b) 2 (c) 12 (d) 0
!
Solution: 16 is a factor of 12 . The number 8484 !" when divided by 16 will leave the same remainder
as the last two digits of 8484 !" are divided by 16. {8 x 12! + 4 x 12! } = 100. 100 divided by 16
remainder = 4


102 | C A T C L A S S E S B Y A s h a n k D u b e y

2. What is the remainder when divided by 5?
(a) 1 (b) 3 (c) 0 (d) 4
Solution:
12! Divided by 5 remainder is 2
12! Divided by 5 remainder is 4
12! Divided by 5 remainder is 3. (As, the total number of digits of 2748 is 4, we will stop this process
here).

2748 !" = [(2 x 12! ) + (7 x 12! ) + (4 x 12! ) + (8)] divided by 5
= [(2 x 3) + (7 x 4) + (4 x 2) + (3)] divided by 5
= [6 + 28 + 8 + 3] divided by 5
= [1 + 3 + 3 + 3] divided by 5
= 10 divided by 5. Remainder = 0

! In even number base, if the last digit of a number is even then so is the number.
! In odd number base, if the sum of the digits of a number is even then so the number.
! If a palindrome number has even number of digits in base then the number is divisible by (n + 1).


Examples
1. Is even?
Solution: Yes. Last digit of the number is even and number is given on the even base.

2. Is even?
Solution: Nope. Sum of the digits = 2 + 7 + 2 + 4 = 15, which is odd.

Questions based on Alphabets
Lets start with very simple alphabetic problems and we will soon understand the magical method of
solving them.

Examples
1. The sum of first n natural numbers is a three digit number, all of whose digits are the same. What is
the value of n?
!(!!!)
Solution: We arrive at the equation = ddd (111, 222, ..etc.). Here goes the simple logic:
!
!(!!!)
= ddd = d x 111 = d x 3 x 37 ( + 1) = 6d x 37
!
Look at L.H.S of the equation n(n + 1) is a product of two consecutive natural numbers. Therefore, R.H.S.
should be a product of two consecutive natural numbers. One of the numbers is 37. Therefore, what
could be the other number, 6d, consecutive to 37 be can only be 36, giving d = 6 and n = 36. Therefore,
36 numbers have been summed up and their sum is equal to 666.
2. What is the value of A + B + C + D such that AB x CB = DDD, where AB and CB are two digit numbers
and DDD is a three digit number.
Solution: AB and CB are two twodigit numbers with the same unit digit. Therefore, R.H.S. should also
be a multiplication of two twodigit numbers with the same unit digit.
R.H.S. = DDD = D x 111 = D x 3 x 37 = 3D x 37. Now, 37 is a two digit number with 7 as the unit digit.
Therefore, 3D should also be a two digit number with 7 as the unit digit D = 9 and 3D = 27.
Therefore, 27 x 37 = 999. A + B + C + D = 2 + 7 + 3 + 9 = 21.


103 | C A T C L A S S E S B Y A s h a n k D u b e y

3. Find the four digit number ABCD such that ABCD x 4 = DCBA
Solution:
Any number multiplied by 4 will give us an even number. Hence, the digit D when multiplied by
4 will give us an even number. Since A is the unit digit of the product it is even. Hence, A = 2, 4, 6
or 8 (It cannot be 0.)
A is also the first digit of the multiplicand and if A = 4, 6 or 8 the product ABCD x 4 will become a
5 digit number. Hence A = 2. Writing the value of A we get 2BCD x 4 = DCB2.
Look at the first and last digits of the multiplicand - we can see that 4 x D gives the unit digit of 2
and 4 x 2 gives the first digit of D. Yes, D =8. Writing the multiplication again with the value of D
we get 2BC8 x 4 = 8CB2.
As we know, a number is divisible by 4 if the number formed by the last two digits is divisible by
4. Since the number 8CB2 is a multiple of 4, the number B2 should be divisible by 4. Or, the
number B2 = 12, 32, 52, 72 or 92. Hence the original number ABCD is 21C8, 23C8, 25C8 or 29C8.
But the last 4 numbers when multiplied by 4 will not give you the first digit of 8 is the product.
Therefore B = 1and original number is 21C8. We write the multiplication again 21C8 x 4 = 8C12.
Notice that when you multiple 8, the unit digit of 21C8, by 4 you write 2 in the unit digit of the
product and carry 3. The tenths digit of the product is 1. Therefore, 4 x C + 3 (carryover) gives a
unit digit of 1. Hence, C is 2 or 7. You can easily check by hundreds digit in the product (which is
C again) that C = 7.
Therefore, required answer is 2178 x 4 = 8712.

4. S is a six digit number beginning with 1. If the digit 1 is moved from the leftmost place to the
rightmost place the number obtained is three times of S. What is the sum of the digits of S?
Solution: Lets put the given problem in variable, it is very simple:


What can be the value of E? E x 3 is giving a unit digit of 1. Therefore, E = 7. We get a carryover
of 2. Writing the value of E, we get


Since a carryover of 2 is resulting in 7, we should get a unit digit of 5 when we multiply D by 3.
Therefore, D = 5. Proceeding in the same manner, we slowly reach the complete solution in this
way:


Therefore, the sum of digits of S = 27.








104 | C A T C L A S S E S B Y A s h a n k D u b e y

Exercise 1.8

1. The number 35A246772 is in base 9. This new sequence is formed. The same process is
number is divisible by 8. Find the value of digit repeated until only a single number remains.
A. What is the final number left if n= 127?
(a) 3 (b) 4
(a) 64 (b) 32
(c) 5 (d) 6
(c) 122 (d) none of these
2. A palindromic number reads the same forward
and backward. A 10-digit palindromic number in 9. A set P consists of all odd numbers from 1 to
base 16 will always be divisible by 150. What is the highest power of 3 in the
(a) 8 (b) 15 product of all the elements of the set P?
(c) 16 (d) 17
(a) 37 (b) 48

3. A four-digit number N1 is written in base 13. A (c) 9 (d) 24
new four-digit number N2 is formed by
rearranging the digits of N1 in any order. Then 10. n > 47 & n is a natural number. Then how
the difference N1 N2 is divisible by many of the following will exactly divide n(n2-
(a) 9 (b) 10 1)(n2-4)
(c) 12 (d) 13 (i) 120 (ii) 24 (iii) 48 (iv) 49

(a) 3 (b) 1
4. How many zeroes will be there in 15! (defined
(c) 2 (d) 4
in base 10 ) in base 12?

(a) 5 (b) 6
(c) 7 (d) none of these. 11. How many natural numbers are there with 24
factors where 2, 3 and 5 are the only prime
5. In a number system, the product of 122 and 41 factors?
is 5442. The number 4434 of this system when (a) 36 (b) 27
converted to decimal system becomes: (c) 12 (d) 9
(a) 1030 (b) 1020

(c) 1040 (d) None of these
12. How many 3 digit numbers are there which can

6. How many 3 digit numbers in base 10 are there be expressed as a perfect square, perfect cube
which can be expressed using 3 digits in base 9 and a perfect fourth power?
as well as in base 11? (a) 2 (b) 3
(a) 609 (b) 720 (c) 1 (d) none of these
(c) 580 (d) none of these

13. If ABC x CBA = 65125, where A, B and C are
7. If n= 10000! is divisible by ! where p is a
single digits, Then find the value of A + B + C
prime number, what is the maximum value of
(a) 8 (b) 10
p?
(c) 6 (d) 12
(a) 29 (b) 101

(c) 97 (d) none of these
14. If TWO x TWO = THREE, where each alphabet

represents a distinct digit. What is the value of
8. Numbers 1,2,3,n are written in sequence.
T + H + R + E + E?
Numbers at odd places are struck off and a
(a) 8 (b) 17


105 | C A T C L A S S E S B Y A s h a n k D u b e y

(c) 18 (d) None of these 20. P, Q, R, S and T are five prime numbers, where
P < Q < R < S < T. It is also given that P + Q + R +
Directions for the questions 15 and 16: Answer
S + T = 482. What is the value of ! ?
the questions based on the following information.
(a) 243 (b) 16807
A defence code is defined by assigning the (c) 32 (d) More than one value
numbers 1 to 9 to the letters in the grid below
such that by adding horizontally, or diagonally Directions for questions 21 and 22: (a, b) is an
the sum of the numbers is the same i.e. 15, and ordered pair such that a and b are two digit
also Y : S is 1 : 4 and S : W is 2 : 1 and W : Q is 2: numbers a is the number obtained by
3. interchanging the digits of b.

S Z Q 21. How many ordered pairs are there such that (a
X V T + b) is a perfect square?
W R Y (a) 3 (b) 4
(c) 8 (d) None of these
15. Which letter has the highest numerical value?
(a) X (b) T 22. How many ordered pairs are there such that (a
(c) V (d) None of these
b) is a perfect square greater than zero?
(a) 3 (b) 10
16. If the code is 1159, then the message will be (c) 13 (d) None of these
(a) ZZVR (b) RRZV

(c) XXVT (d) b or c 23. A three digit number ABC is a perfect square
and the number of factors of this number is
17.
also a perfect square. If (A + B + C) is also a
AA A>0
perfect square, then what is the number of
+BB B>0
factors of the 6 digit number ABCABC?
CDC
All A, B, C are integers. Find the value of D. (a) 32 (b) 52
(a) 1 (b) 2 (c) 3 (c) 72 (d) Cannot be determined
(d) 4 (e) 5
Directions for questions 24 and 25: ABCDEF is a
Directions for questions 18 and 19: The 6 digit number with distinct digits. Further,
multiplication of two numbers is shown below: the number is divisible by 11 and the sum of its
digits is 24. Further A>C>E and B>D>F.

AD4

x E
24. The sum A + C + E is equal to
A206
Where A, D and E are all distinct digits. (a) 6 (b) 8
(c) 12 (d) cannot be determined
18. The value of D is
(a) 3 (b) 9 (c) 2 (d) 7 25. A + B is always
(a) 6 (b) 9
19. What is the value of A + E is (c) 10 (d) cannot be determined
(a) 6 (b) 9 (c) 10 (d) 11


106 | C A T C L A S S E S B Y A s h a n k D u b e y

26. a, b and c are positive integers such that, a + b + 28. The integer which is positive is
c = 2003. Let E = 1 ! + 1 ! + 1 ! . Find (a) s (b) q (c) r
the number of possible values of E. (d) p (e) t
(a) 2 (b) 3
(c) 1003 (d) 2004 29. A positive integer is the nth multiple of p
(where n > 1) and its reverse is (! 1)th
Directions for Questions 27 and 28: Answer the multiple of (p + 1). Which of the following is
questions based on the following information: the number?
Three out of the five integers p, q, r, s and t are (a) 1008 (b) 7201
negative. One of the integers is positive and one (c) 4012 (d) 2004
is zero. Some additional information is given
below. 30. When ! + {where n and a are some
I. pr > qs particular natural number} is divided by 7, the
II. p = 0.3 q remainder are 3, 4 or 5 for all natural numbers
III. t is greater than q but less than p x. Find the least possible values of n and a
respectively.
27. The integer which is zero is (a) 2 and 3 (b) 2 and 4
(a) q (b) r (c) s (c) 3 and 3 (d) 3 and 4
(d) t (e) p


Practice Exercise 1

1. What is the unit digit of the sum 1! + 2! + 3! +... + 20!
(a) 0 (b) 2 (c) 3 (d) 5

2. What is the first non - zero integer from the right in 83301858 + 83701887?
(a) 3 (b) 1 (c) 9 (d) none of these

3. The remainder when 599 is divided 13 is:
(a) 5 (b) 8 (c) 0 (d) 1

4. Palindrome is the numbers which read same forward and backward. How many palindrome numbers
are there less than 1 million?
(a) 999 (b) 90 (c) 99 (d) 1998

5. How many times you write digit 1 while writing all natural numbers from 1 to 1000?
(a) 299 (b) 300 (c) 301 (d)None of these

6. How many times you write digit 0 while writing all natural numbers from 1001 to 2000?
(a) 299 (b) 300 (c) 301 (d) None of these


107 | C A T C L A S S E S B Y A s h a n k D u b e y


7. To write all the page numbers of a book, exactly 136 times digit 1 has been used. Find the number of
pages in the book.
(a) 199 (b) 200 (c) 195 (d) 194

8. All the page numbers of a book has been added and sum was found to be 1000. But teacher told that
one page number has been mistakenly added 3 times. Can you identify the mistakenly added page
number?
(a) 5 (b) 10 (c) 27 (d) cannot be determined

9. Sum of first 'n' consecutive natural numbers is a three digit number, whose each digit is same. Find 'n'.
(a) 36 (b) 37 (c) 6 (d) None of these

10. How many numbers of the set S {1, 11, 111, ......} are perfect squares?
(a) 1 (b) 2 (c) 3 (d) more than 3

11. Find the number of positive integral pairs (a, b) such that a2 + b2 = 1000.
(a) 2 (b) 4 (c) 6 (d) 8

12. Nimai and Nitai are two brothers who have some mangoes with them to sell. They fix the price of each
mango to be equal to the number of total mangoes with both of them together initially. Together they
sell all the mangoes and after that they start distributing the money collected in this particular fashion.
First Nimai takes a 10 rupee note, and then Nitai takes a 10 rupee note and so on. In end it's turn of Nitai
who don't get any more 10 rupees. Can you tell me how much rupees he get in his last turn?
(a) 3 (b) 5 (c) 6 (d) Cant say

13. How many numbers less than 1200 are not divisible by 15 but divisible by 12?
(a) 40 (b) 80 (c) 120 (d) None of these

14. How many of first 100 numbers are multiple of 2 and 5 but not of 3?
(a) 6 (b) 5 (c) 8 (d) 7

15. Find the sum of all the numbers less than 101 which are co-prime to 100.
(a) 1000 (b) 999 (c) 900 (d) None of these

16. Find the sum of the numbers which are less than 2100 and are multiples of 3 but not multiple of 2, 5 and
7?
(a) 252000 (b) 151000 (c) 95000 (d) 222000

17. If 37N is divided by 41 remainder is 5, Find the remainder when N is divided by 41.
(a) 1 (b) 40 (c) 9 (d) 5


108 | C A T C L A S S E S B Y A s h a n k D u b e y

18. Find the smallest natural number N such that 13N is divided by 19 remainder is 17.
(a) 13 (b) 18 (c) 17 (d) None of these

19. Find the smallest natural number N such that N(N+1) is divided by 20 remainder is 2.
(a) 6 (b) 13 (c) 18 (d) None of these

20. Find the remainder when 277 is divided by 77.
(a) 76 (b) 18 (c) 0 (d) None of these

21. How many four digit numbers exist such that when divided by 7, 9 and 11 one gets the remainders of 2,
3 and 4 respectively.
(a) 12 (b) 13 (c) 14 (d) None of these

22. How many natural numbers less than 65 have odd number of divisors (excluding 1 and those numbers
themselves)?
(a) 8 (b) 7 (c) 9 (d) 12

23. Find the highest power of 567 that can divide 285! exactly?
(a) 35 (b) 40 (c) 45 (d) 140

24. Find the number of zeroes in the end of the expression 5! !?
(a) 3906 (b) 50 (c) 100 (d) 781

25. Consider the set A = {1,2,3,..1000}How many arithmetic progressions can be formed from the
elements of A that start with 1 and end with 1000 and have at least 3 terms?
(a) 4 (b) 6 (c) 7 (d) 8

26. In how many ways can the number 105 be written as a sum of two or more consecutive positive
integers?
(a) 7 (b) 4 (c) 5 (d) 6

27. How many numbers below 100 can be expressed as a difference of two perfect squares in only one way?
(a) 25 (b) 26 (c) 34 (d) 35
28. How many 2 digit odd numbers are there with exactly 8 factors?
(a) 1 (b) 2 (c) 3 (d) 0

29. How many natural numbers less than 1000 will have exactly 3 factors?
(a) 36 (b) 27 (c) 31 (d) 11

30. P is a set of all natural numbers with four factors such that sum of the factors, excluding the number
itself is 31. Find the sum of all the elements in P?
(a) 125 (b) 495 (c) 716 (d) 898


109 | C A T C L A S S E S B Y A s h a n k D u b e y

31. The number of ways in which 7! can be written as sum of some consecutive natural numbers is
(a) 11 (b) 7 (c) 13 (d) 9

32. N is a set of all natural numbers less than 500 which can be written as sum of two or more consecutive
natural numbers. Find the maximum number of elements possible in N?
(a) 250 (b) 492 (c) 493 (d) none of these

33. How many integers between 1 and 1000, both inclusive, can be expressed as the difference of the squares of
two non - negative integers?
(a) 750 (b) 748 (c) 300 (d) 250

34. Which of the following numbers can be written as the sum of the squares of three odd natural numbers?
(a) 5021 (b) 4445 (c) 3339 (d) 1233

35. When 10 ! is divided by 13, remainder is 1. If x is a natural number less than 100, how many values can x
take?
(a) 8 (b) 16 (c) 1 (d) 12

36. AB= 9C +1, where A, B, C are natural numbers and 100 A 200. How many different values can A take?
(a) 101 (b) 50 (c) 11 (d) none of these

37. After the division of a number successively by 3, 4 and 7, the remainder obtained are 2, 1 and 4
respectively. What will be the remainder if 16 divide the same number?
(a) 53 (b) 75 (c) 41 (d) cannot be determined

38. N is a positive integer, which when divided by 16, 17 and 18 leaves remainders of 6, 7 and 8 respectively.
Find the remainder when N2 + 5N + 6 is divided by 12.
(a) 2 (b) 6 (c) 8 (d) 4

39. If a= 12, b= 23, c= 34, z= 2627. In the product of all the alphabets, how many zeroes exist in the end?
(a) 100 (b) 104 (c) 80 (d) 106

40. Students in a college decide to form a philatelic club. The requirements for being a member of this club
is that the number of stamps a member owns must be a perfect square. Also, each member must have a
number of stamps which differs by 192 from only one other member. If this club eventually grows to
have its maximum possible number of members, determine the difference between the greatest
number of stamps owned by a member and the least number of stamps owned by a member.
(a) 2873 (b) 3614 (c) 3122 (d) 2397

41. LCM and HCF of two numbers are 609 and 21 respectively. Find the number of maximum possible
ordered pairs of such numbers?
(a) 2 (b) 3 (c) 4 (d) 5


110 | C A T C L A S S E S B Y A s h a n k D u b e y


42. How many non - zero integral values of X, Y and Z are there, such that ! = ! + ! and ! 100?
(a) 8 (b) 16 (c) 2 (d) 32

43. How many natural numbers are there with 24 factors where 2,3 and 5 are the only prime factors?
(a) 36 (b) 27 (c) 12 (d) 9

44. N is a 3 digit number which when divided by 3, 4, 5 successively remainders obtained are 2,1,3. How
many values can N take?
(a) 21 (b) 15 (c) 14 (d) 16

45. Find the last three digits of: 7 x 31 x 55 x 79 x .. x 3103.
(a) 895 (b) 545 (c) 635 (d) none of these

46. A number is formed by multiplying the 1st 100 natural number multiples of 5. What will be the highest
power of 25 in that number?
(a) 124 (b) 24 (c) 62 (d) None of these

47. A maxi power number is defined as a number which is perfect square, cube, 4th power, 5th power, 6th
power, 7th power, 8th power, 9th power and 10th power of distinct natural nos. The last digit of the
difference between the smallest two maxi-power numbers is:
(a) 1 (b) 0 (c) 5 (d) None of these

48. How many 3 digit numbers are there which can be expressed as a perfect square, perfect cube and a
perfect fourth power?
(a) 2 (b) 3 (c) 1 (d) None of these

! !
49. If 3125! = 5!"#$ , then what is the value of ?
(a) 1 (b) 1 (c) 1/4 (d)

!
50. For how many integers A is square of an integer?
!" !!
(a) 2 (b) 3 (c) 4 (d) 5

51. Let be the smallest positive number such that the number N = 8! 5!"" has 304 digits. Then the
sum of the digits of N is
(a) 10 (b) 11 (c) 18 (d) 19

52. A man was born in a year that was a square number, lived a square number of years and died in a year
that was also a square number. Then the year he could have been born in was:
(a) 1444 (b) 1600 (c) 1764 (d) 1936


111 | C A T C L A S S E S B Y A s h a n k D u b e y

53. The difference between the cubes of two consecutive positive integers is 1027. Then the product of
these integers is
(a) 132 (b) 306 (c) 342 (d) 552

54. In Mahabalipuram temple there are some magical bells which tolls 18 items in a day, simultaneously.
But every bell tolls at a different interval of time, but not in fraction of minutes. The maximum number
of bells in the temple can be:
(a) 18 (b) 10 (c) 24 (d) 6

55. ! ! = ( + )! q , where 1 < q < r < p < 10 then the value of p + q + r is :
(a) 31 (b) 21 (c) 15 (d) 12

56. The value of n in the expression n2 2 (n!)+ n =0 for every n N is:
(a) 6 (b) 1 (c) 3 (d) both b and c are true

57. A set P consists of all odd numbers from 1 to 150. What is the highest power of 3 in the product of all
the elements of the set P?
(a) 37 (b) 48 (c) 9 (d) 24

58. A smallest positive integer N has exact 12 factors but only 3 prime factors. If the sum of these prime
factors is 20, find the sum of digits of N.
(a) 11 (b) 6 (c) 8 (d) None of these

59. N is a 3 digit number which when divided by 3, 4, 5 successively remainders obtained are 2, 1, 3. How
many values can N take?
(a) 21 (b) 15 (c) 14 (d) 16

60. The smallest possible number that can be expressed as the sum of cube of two natural numbers in two
different combinations.
(a) 1000 (b) 1728 (c) 129 (d) None of these

61. Total number of factors of a number is 24 and the sum of the prime factors out of four, is 25. The
product of all 4 prime factors of this number is 1365. Then such a greatest possible number can be:
(a) 17745 (b) 28561 (c) 4095 (d) cant be determined

62. When any two natural numbers N! and N! , such that N! = N! + 2, are multiplied with each other, then
which digit appears least time as a unit digit if N! 10000?
(a) 0 (b) 9 (c) 4 (d) both (a) and (c)

63. In the above problem, if all such unit digits will be added the maximum sum can be:
(a) 4491 (b) 4500 (c) 3609 (d) 5400


112 | C A T C L A S S E S B Y A s h a n k D u b e y

64. A diamond expert cuts a huge cubical diamond into 960 identical diamond pieces in minimum number
of n cuts. If he wants to maximize the number of identical diamond pieces making same number of n
cuts to it. So the maximum numbers of such diamond pieces are:
(a) 1000 (b) 1331 (c) 1200 (d) none of these

65. The sum of all the factors of 45000 which are exactly the multiples of 10 is:
(a) 152295 (b) 141960 (c) 600 (d) none of these

66. A number is divided strictly into two unequal parts such that the difference of the squares of the two
parts equals 50 times the difference between the two parts. The number is:
(a) 100 (b) 250 (c) 50 (d) cant be determined

67. A positive number p is such that (p + 4) is divisible by 7. N being a smallest possible number larger than
first prime number, which can make p + N ! divisible by 7. The value of N is:
(a) 3 (b) 9 (c) 5 (d) 7

68. Anjali and Bhagwat fired 45 shots each. Total 66 bullets hit the target and the remaining bullets missed
it. How many times does the Anjali hit the target if it is known that the number of hits per one miss
shown by the Anjali is twice that of Bhagwat?
(a) 30 (b) 36 (c) 40 (d) 35

69. The number of digits in the product of 5!" x 8!" is:
(a) 77 (b) 75 (c) 78 (d) None of these

70. A student of 5th standard started writing down the counting numbers as 1, 2, 3, 4, and then he added
all those numbers and got the result 500. But when I checked the result I have found that he had missed
a number. What is the missing number?
(a) 25 (b) 32 (c) 30 (d) 28

71. Paltry and Sundry, the two bird hunters went to woods. Paltry fires 5 shots when Sundry fires 7 shots.
But Paltry kills 2 out of 5 while Sundry kills 3 out of 7. When Sundry has missed 32 shots, then how many
birds has Paltry killed?
(a) 25 (b) 24 (c) 16 (d) 12

72. In the morning batch at Testcracker we have observed that when five-five students took seat on a
bench, 4 students remained unseated. But when eleven students took seat per bench, 4 benches
remained vacant. The number of students in our morning batch was?
(a) 55 (b) 48 (c) 26 (d) None of these

73. A man sells chocolates which are in the boxes. Only either full box or half a box of chocolates can be
purchased from him. A customer comes and buys half the number of boxes which the seller had plus


113 | C A T C L A S S E S B Y A s h a n k D u b e y

half a box more. After this the seller is left with no chocolate boxes. How many chocolate boxes the
seller had initially?
(a) 2 (b) 3 (c) 4 (d) 3.5

74. 1! 2! + 3! 4! + 198! + 199! :


(a) 19900 (b) 12321 (c) 19998 (d) none of these

75. In how many ways can 729 be expressed as a difference of the square of whole numbers?
(a) 4 (b) 6 (c) 8 (d) none of these

76. In how many ways can 2310 be expressed as a product of 3 factors?
(a) 41 (b) 23 (c) 56 (d) 46

77. Each family in Gyanpur village has atmost two adults and the total number of boys in this village is less
than the number of girls. Similarly the number of girls is less than the number of adults in the village.
Raghubir Singh, the chief of this village is the only adult in his family. The minimum number of families in
his village is:
(a) 2 (b) 3 (c) 4 (d) none of these

78. A company offers total 150 pens to its customers. As per the scheme one pen will be offered on the
purchase of a Quantitative Aptitude book. Out of 150 pens the cost of some pens is Rs. 3 and the cost
of the rest pens is Rs. 5. Maximum how many customers can avail a pen of Rs. 5 as an offer from the
company if the total cost of the pens cannot exceed Rs. 745.
(a) 45 (b) 120 (c) 147 (d) none of these

79. Earlier when I have created my e-mail-ID, the password was consisting of first 4 prime numbers.
Recently when I tried to check my emails I got dumbfounded since I could be remember my password
exactly. So when I have written 2735, my computer indicated me that no digit is correctly placed. Again I
tried 5273, I got the same response. So once again I have written only 3 as the left most digit for my
password it again indicated me that it was wrong. Finally I have taken one more attempt and got the
account open. The code of my password is:
(a) 2537 (b) 7352 (c) 7325 (d) none of these

80. We publish a monthly magazine of 84 pages. Once I found that in a magazine 4 pages were missing. One
out of them was page number 29 it is known that the page number of the last page of the magazine is
84, (including the cover page). The numbers printed on the missing pages were:
(a) 29, 52, 53 (b) 30, 55, 56 (c) 28, 52, 53 (d) cant be determined

81. There are six locks exactly with one key for each lock. All the keys are mixed to each other. The
maximum number of attempts needed to get the correct combination is:
(a) 21 (b) 15 (c) 6 (d) cant be determined

82. Recently, a small village, in Tamilnadu where only male shepherd reside with four sheep each, was
devastated by Tsunami waves. Therefore 8 persons and 47 sheep were found to be dead and the person


114 | C A T C L A S S E S B Y A s h a n k D u b e y

who luckily survived, left the village with one sheep each since 21 sheep were too injured to move so
have been left on their own luck, in the village. The number of sheep which were earlier in the village is:
(a) 84 (b) 120 (c) cant be determined (d) none of these

83. The sum of: (22 + 42 + 62 + + 1002) (12 + 32 + 52 + + 992) is:
(a) 5555 (b) 5050 (c) 888 (d) 222

84. The sum of the n terms of a series is n! + n2 then the 6th term is, if n N:
(a) 756 (b) 611 (c) data insufficient (d) none of these

85. Anjuli bought some chocolates from Nestles exclusive shop, she gave to Amit one less than half of what
she had initially. Then she had given 3 chocolates to Bablu and then half of the chocolates which she had
then given to Charles. Thus finally she gave one chocolate to Deepak and the remaining one she ate
herself. The number of chocolates she had purchased.
(a) 9 (b) 12 (c) 10 (d) 15

86. If a! b! c! !"" + a! b! c! !"" + a! b! c! !"! +. . . . . a!"" b!"" c!"" !"" , where a! b! c! is a three digit
positive number and in the expression all the 100 numbers are any consecutive 3 digit numbers. The last
digit is:
(a) 0 (b) 1 (c) 3 (d) none of these

87. Captain Manoj Pandey once decided to distribute 180 bullets among his 36 soldiers. But he gave n
bullets to a soldier of nth row and there were same number of soldiers in each row. Thus he distributed
all his 180 bullets among his soldiers. The number of soldiers in (n 1)th row was?
(a) 3 (b) 8 (c) 9 (d) none of these

88. A certain number n can exactly divide 3!" 1 then the number can also divide the number:
(a) 3!" + 1 (b) 3! 1 (c) 3!" 1 (d) 3!" 1

89. If a number n can exactly divide 5!" 1 then n can divide necessarily:
(a) 5!" 1 (b) 5!" 1 (c) 5!" + 1 (d) both (a) and (b)

90. If = 555! and = 278 !!! then which one of the following relation is appropriate?
(a) > (b) = (c) < (d) cant say

91. A number when divided by it leaves the remainder 18 and if another number is divided by the
same divisor it leaves the remained 11. Further if we divide + by then we obtain the remainder
4. Then the common divisor is:
(a) 1 (b) 3 (c) 25 (d) cant be determined

92. Total number of natural numbers being the perfect square whose square root is equal to the sum of the
digits of the perfect square is:
(a) 0 (b) 1 (c) 2 (d) 12

93. The total number of 3 digit numbers which have two or more consecutive digits identical is:
(a) 171 (b) 170 (c) 90 (d) 180


115 | C A T C L A S S E S B Y A s h a n k D u b e y

94. Mrs. Dubey appeared in CAT for four consecutive years, but coincidently each time her net score was 75.
!
She told me that there was rd negative marking for every wrong answer and 1 mark was allotted for
!
every correct answer. She has attempted all the questions every year, but certainly some answers have
been wrong due to stress and conceptual problems. Which is not the total number of questions asked
for CAT in any year, in that period?
(a) 231 (b) 163 (c) 150 (d) 123

95. The number log2 7 is:
(a) an integer (b) a rational number (c) an irrational number (d) a prime number

96. Let > 1, be a positive integer. Then the largest integer , such that ! + 1 divides 1 + + ! +
! + + !"# is :
(a) 127 (b) 63 (c) 64 (d) 32

97. If the integers m and n are chosen at random between 1 and 100, then at most distinct numbers of the
form 7! + 7! is divisible by 5 equals to:
(a) 1250 (b) 10000 (c) 2500 (d) none of these

98. A typist while typing the numbers from 600 to 799 mistakenly he typed 8 every time in place of 6. So the
total number of times he has typed 8 is:
(a) 300 (b) 230 (c) 180 (d) none of these

99. A thief somehow managed to steal some golden coins from a banks cash but while coming out of it at
the first door he was caught by the watch man and he successfully dealt him by paying 1 coin plus half of
the rest coins. Further he had to pay 2 coins, then half of the rest to the second watchman. Once again
at the third gate (outermost) he gave 3 coins and then half of the rest. After it he was left with only one
coin. How many coins had he stolen?
(a) 32 (b) 36 (c) 25 (d) none of these

100. The distance between the houses of Sarvesh and Ravi is 900 km and the house of former is at 100th
milestone where as the house of Ravis is at 1000th milestone. There are total 901 milestones at a regular
interval of 1 km each. When you go to Ravis house from the house of Sarvesh which are on the same
highway, you will find that if the last digit (i.e., unit digit) of the 3 digit number on every milestone is same
as the first (i.e., hundreds digit) of the number on the next mile stone is same, then these milestones must
be red colour and rest will be of black. Total number of red colour milestones is:
(a) 179 (b) 90 (c) cant be determined (d) none of these

For Answer Keys and Solution Contact Ashank Dubey @ ashankdubeyclasses@gmail.com

! Ashank is unarguably India's best Quant trainer and 100%iler of CAT 16 & CAT - 15 (Qunat). He has
trained numerous CAT toppers ever since the CAT went online, including 99.9%tilers of CAT 2015
Shrikanth, Rushil, Sandeep, Abhilasha, Prakhar, Pratik, Shivani to name a few.


116 | C A T C L A S S E S B Y A s h a n k D u b e y

Practice Exercise 2

!""!! !!
1. If S = , then, the sum of digits of S is
!
(a) 195 (b) 203 (c) 201 (d) 196

2. How many numbers N are there such that sum of N and its digits is 1000003?
(a) 0 (b) 1 (c) 2 (d) More than 2

3. How many three digit numbers are 34 times of sum of their digits?
(a) 3 (b) 4 (c) 5 (d) 6

4. The sum of four two digit numbers is 256. None of the four numbers have a digit zero. Also all the
eight digits are different. Which digit has not been used in the four numbers?
(a) 4 (b) 5 (c) 6 (d) none of these

5. Find the 1000th term of the number sequence: 1, 3, 4, 7, 8, 9, 10, 11, 13, 14,. In which there is no
number which contain the digit 2, 5 or 6.
(a) 3939 (b) 3938 (c) 4000 (d) 4001

6. How many positive divisors of 100!" end in exactly two zeroes?
(a) 361 (b) 36 (c) 324 (d) 37

7. I am thrice as old as you were when I was twice as you are, yelled Rahul at his only son Sachin. Sum of
their ages is 65 years. How old is Rahul?
(a) 45 years (b) 38 years (c) 48 years (d) None of these

8. Find the number of positive integral pairs (, ) that satisfy ! + ! = 2325.
(a) 0 (b) 1 (c) 2 (d) 3

9. What is the highest natural number that divided ! (! 1)( ! 2) for all natural numbers n?
(a) 24 (b) 48 (c) 72 (d) 60

10. How many 3 digit positive integers cannot be written as sum of four distinct 2 digit positive integers?
(a) 609 (b) 600 (c) 610 (d) None of these

!
11. For how many positive integers N, product of its digits is (7N) 205?
!
(a) 1 (b) 2 (c) 3 (d) None of these

12. How many numbers are there that appear in both the arithmetic sequence 4, 11, 18, 25, . 2020 and
the arithmetic sequence 4, 13, 22, 31, , 2020?
(a) 32 (b) 33 (c) 34 (d) 30


117 | C A T C L A S S E S B Y A s h a n k D u b e y

13. 3! and 5! are converted in base 15 and written one after other to form a single number of base 15.
How many digits are there in this newly formed number?
(For example when How many 3 digit 3! and 5! are converted in base 15, they become 9 and 1A and
after writing together they form a 3 digit number 91A of base 15.)
(a) 2 (b) 7 (c) 6 (d) None of these

14. How many 11 digit numbers have at least two equal digits?
(a) 9 10!" (b) 9 10! (c) 11 10!" (d) None of these

15. Ashank measured and noted the distance (in cm) between four trees in his garden in the following table.
Find the distance between Guava and Mango trees.
Apple Guava Mango
Banana 60 80 150
Apple 0 100 210

(a) 17 (b) 15 (c) 20 (d) None of these


16. How many two digit positive integers are there which are one and a half times larger than the product of
their digits?
(a) 0 (b) 1 (c) 2 (d) 3

17. For how many ordered pairs (x, y), where x and y are non negative integers, is the equation + =
1332 satisfied?
(a) 3 (b) 5 (c) 7 (d) 9

18. Let N = 2!" x 3!" . How many factors of ! are less than N but do not divide N completely?
(a) 180 (b) 208 (c) 310 (d) 387

19. The number 5450 has the property that the number formed by first two digits is four more than the
number formed by last two digits. N is a four-digit number with this property, which is a perfect square.
Find the sum of the digits of N.
(a) 18 (b) 17 (c) 12 (d) None of these

20. For prime numbers A and B both greater than 3, what is the highest integer which divides (A - B)
completely?
(a) 3 (b) 4 (c) 12 (d) 24

21. A new sequence of positive integers is formed by removing all perfect cubes as {2, 3, 4, }. What is the
2012th number in the sequence?
(a) 2023 (b) 2024 (c) 2025 (d) 2026


118 | C A T C L A S S E S B Y A s h a n k D u b e y

22. How many positive integers among first 2100 are there which are multiple of at least one of 2, 3, 5 and
7?
(a) 1600 (b) 1620 (c) 1620 (d) None of these

(!!"" ! !)
23. What would be digit sum of last three digits of ?
!
(a) 1 (b) 3 (c) 5 (d) 9

24. is a number formed by appending y to x, where x is an n digit natural number and y a single digit
natural number. Find the remainder when xy 9y + 6y + 3 is divided by 69, if xy is a multiple of 69.
(a) 0 (b) 1 (c) 2 (d) 3 (e) 4

25. When ! + (where n and a are some particular natural numbers) is divided by 7, the remainders are
3, 4, or 5 for all natural numbers . Find the least possible values of n and a respectively.
(a) 2 and 3 (b) 2 and 4 (c) 3 and 3 (d) 3 and 4

26. A number divided by a certain divisor leaves a remainder of 11, whereas the square of the number when
divided by the same divisor, leaves a remainder of 1. Find how many such divisors are possible.
(a) 2 (b) 4 (c) 6 (d) 8

27. Find in how many ways can 9216 be written as a product of two numbers, such that both the numbers
are perfect squares.
(a) 1 (b) 6 (c) 5 (d) 8

28. Find the remainder when 3!"#$ is divided by 549.
(a) 1 (b) 9 (c) 81 (d) 540

29. A number ! has 11 factors less than N. Find in how many ways can ! be written as the product of 2
distinct numbers.
(a) 8 (b) 16 (c) 15 (d) 33

30. The sum of factorials of the first n natural numbers is equal to the sum of the cubes of the digits of the
sum. Find the value of n.
(a) 3 (b) 4 (c) 5 (d) 6

31. For any natural numbers, let K be the index of the highest power of 2 which divides the number. Among
the first 100 natural numbers all those numbers for which k is even, are excluded. Find the index of the
highest power of 2 which divides the product of the remaining numbers.
(a) 43 (b) 53 (c) 67 (d) 97 (e) 33

32. One day, there were n students in a class. Their teacher distributed a certain number of chocolates
equally among them and was left with 5 chocolates. The next day the number of students quadrupled.


119 | C A T C L A S S E S B Y A s h a n k D u b e y

Again the teacher distributed the same number of chocolates equally among them and was left with 24
chocolates. What was the value of n?
(a) 4 (b) 20 (c) 43 (d) 91 (e) None of these

33. What will be the remainder when 79!" is divided by 100.
(a) 21 (b) 41 (c) 81 (d) 39 (e) 19

34. The difference between a four digit number and its reverse is 4995. The difference between the digit in
thousand place and the digit in units place is 5. Find the difference between the digits in hundred and
tens place.
(a) 0 (b) 1 (c) 3 (d) 5 (e) 6

35. Find the five digit palindrome, which when divided by 633, 1055 and 1477 leaves 600, 1022 and 1444
respectively as remainders.
(a) 21212 (b) 22122 (c) 24542 (d) 23132

36. In a proper fraction, written in its simplest form, the numerator is 1 more than one ninth of the
denominator. When written as a decimal number, the decimal point is followed by one digit and then by
a pair of recurring digits. Which of the following could be a fraction?
(a) 0. 113 (b) 0. 112 (c) 0. 123 (d) 0. 132

37. A chain smoker had spent all the money he had. He had no money to buy his cigarettes. Hence, he
resorted to join the stubs and to smoke them. He needed 4 stubs to make a single cigarette. If he got a
pack of 10 cigarettes as a gift, then how many cigarettes could he smoke in all?
(a) 10 (b) 11 (c) 12 (d) 13

38. abc is a three digit natural number so that abc = a! + b! + c!. What is the value of + ! ?
(a) 9 (b) 1296 (c) 3125 (d) 19683
! ! !
39. abc is a three digit whole number so that abc = + + . [300 < abc < 400]
What is the value of (a + b + c)?
(a) 9 (b) 10 (c) 11 (d) Cant say

40. In a basket, there are some apples. Sanjeev takes half of them but return one of them. Rashi takes one
third of the remaining and return two of them. Rohit takes one fourth of the remaining and return
three of them. No apple is cut. The minimum number of apples left in the basket at the end is
(a) 9 (b) 12 (c) 15 (d) 28

41. It is given that 2!" + 1 is exactly divisible by a certain number. Which one of the following is also divisible
by the same number?
(a) 2!" + 1 (b) 2!" + 1 (c) 2!" 1 (d) 2!" + 1


120 | C A T C L A S S E S B Y A s h a n k D u b e y

42. The number 500! Is successively divided by 35, P times, and then by 15, Q times and finally by 21, R
times where P, Q, R are positive integers. These divisions have no remainders at any stage. If the final
quotient (M) thus obtained is divisible by 3!"# but not by 3!"" , and also not divisible by 7, which of the
following numbers could be the index of the highest power of 5 that divides M?
(a) 5 (b) 77 (c) 79 (d) 82 (e) 89

43. Find the remainder when (16! + 91) is divided by 323.
(a) 0 (b) 1 (c) 5 (d) 91 (e) 286

44. A positive integer is the nth multiple of p (where n > 1) and its reverse is (! 1)th multiple of (p + 1).
Which of the following is the number?
(a) 1008 (b) 7201 (c) 4012 (d) 2004 (e) 1089

45. If P is a prime number greater than 2, find the remainder when (P 2)! + Q is divided by p, where Q = P
1.
(a) 0 (b) 1 (c) 2 (d) Q (e) 3

46. In the above question, what is the remainder when [(p 1)!] [(P 2)!] is divided by P?
(a) 0 (b) P 1 (c) P 2 (d) 1 (e) P 3

47. Find the least number N which when divided by P successively 3 times, leaves remainders of 2, 3, 7
respectively, where P is the least number such that when it is divided by 7, 6 and 5 it leaves remainders
of 2, 3, 4 respectively.
(a) 590 (b) 594 (c) 595 (d) 596 (e) 597

48. When 200 and 120 are divided by a number P, we get remainders of 5 and 3 respectively. Find the sum
of all the possible values of P.
(a) 52 (b) 53 (c) 55 (d) 56 (e) 57

49. Rakesh has a set of 3 digit numbers. He divided all those numbers by 6, 3, 2 successively and found
that all numbers left the same remainders of 4, 2, 1. Find the sum of all such possible 3 digit numbers
which Rakesh can have.
(a) 13,556 (b) 13,520 (c) 13,486 (d) 13,350 (e) 13,450

50. Rahul expresses 10! as a product of four factors, all greater than 1, such that at least 4 of all the 6
possible pairs of factors are co prime. In how many ways can he form the factors?
(a) 4 (b) 6 (c) 9 (d) 16 (e) 36

51. In the product of all co primes of 59 that are less than 59, find the largest power of 10.
(a) 11 (b) 13 (c) 143 (d) 362 (e) 384


121 | C A T C L A S S E S B Y A s h a n k D u b e y

52. Find the sum of all those numbers less than or equal to 108, which are neither factors not multiples of
36.
(a) 5579 (b) 5570 (c) 5850 (d) 5615 (e) 5886

53. Soumya had divided p by 9, where p = 1! 2! + 3! 4! +.+ 1 !!! ! and she found that the
remainder for all odd n greater than 5 is the same. Find the remainder that Soumya got.
(a) 0 (b) 1 (c) 2 (d) 3 (e) 4

54. Find the product of odd factors of 3024
(a) 3674 (b) 31274 (c) 3672 (d) 31272 (e) 31874

55. Let a, b, c, d be four real numbers such that
a + b + c + d = 8,
ab + ac + ad + bc + bd + cd = 12.
Find the greatest possible value of d.
(a) 2 + 3 2 (b) 3 + 3 2 (c) 2 + 3 3 (d) None of these

56. What are the dimensions of the greatest n n square chessboard for which it is possible to arrange 121
coins on its cells so that the numbers of coins on any two adjacent cells (i.e. that share a side) differ by
1?
(a) 12 x 12 (b) 11 x 11 (c) 15 x 15 (d) None of these

! ! !
!! ! ! ! ! ! !!
! !
57. Find the minimum value of ! ! !
for x > 0.
!! ! !!! !
! !
(a) 4 (b) 6 (c) 8 (d) Cant say

58. Three men - Arthur, Bernard and Charles with their wives Ann, Barbara and Cynthia, not necessarily
in order make some purchases. When their shopping is finished each finds that the average cost in
dollars of the articles he or she has purchased is equal to the number of his or her purchases. Arthur has
bought 23 more articles than Barbara and Bernard has bought 11 more than Ann. Each husband has
spent $63 more than his wife. What is the total amount spent by Charles and Cynthia?
(a) 1024 (b) 512 (c) 961 (d) 64

59. Find the sum of all remainders when n5 5n3 + 4n is divided by 120 for all positive integers n
2010.
(a) 0 (b) 4 (c) 3 (d) None of these

60. There are 12 balls of equal size and shape, but one is either lighter or heavier than the other eleven. For
how many minimum number of times weighing required with ordinary beam balance to determine the
faulty ball?
(a) 1 (b) 2 (c) 3 (d) 4

61. Find the largest prime number p such that p3 divided 2009! + 2010! + 2011!
(a) 661 (b) 667 (c) 2011 (d) 659


122 | C A T C L A S S E S B Y A s h a n k D u b e y

62. Three boys Ali, Bashar and Chirag are sitting around a round table in that order. Ali has a ball in his hand.
Starting from Ali the boy having the ball passes it to either of the two boys. After 6 passes the ball goes
back to Ali. How many different ways can the ball be passed?
(a) 16 (b) 6 (c) 20 (d) 22

63. A closed bag contains 3 green hats and 2 red hats. Amar, Akbar, Anthony all close their eyes, take a hat,
put it on, and close the bag. When they open their eyes, Amar looks at Akbar and Anthony, but can't
deduce the color of his own hat. Akbar now tries to deduce his own hat's color but can't be certain.
What color is Anthony's hat?
(a) Red (b) Green (c) Cannot be determined (d) None of these

64. The product of three numbers is 1620. If HCF of any two out of the three numbers is 3, what is their
LCM?
(a) 90 (b) 180 (c) 270 (d) 135

65. If p is a prime number and w, x, y, z are four natural numbers whose sum is less than p, then
+ + + ! ! + ! + ! + ! is always divisible by
(a) p 1 (b) p (c) p + 1 (d) !

66. How many divisors of 25200 can be expressed in the form 4n + 3, where n is a whole number?
(a) 6 (b) 8 (c) 9 (d) None of these

!!"
67. If m and n are positive integers such that ! = , then how many pairs (m, n) are possible?
!!!!!
(a) 4 (b) 10 (c) 16 (d) Infinite


68. How many 4-digit multiples of 3 can be formed using the digits 2 and 3 only?
(a) 4 (b) 6 (c) 5 (d) 7

69. N is a five-digit perfect square whose unit digit is same as the tens digit. How many such N are there?
(a) 31 (b) 30 (c) 33 (d) 32

70. The sequence 1, 2, 4, 5, 7, 9, 10, 12, 14, 16, 17,. has one odd number followed by the next two even
numbers, then the next three odd numbers followed by the next four even numbers and so on. What is
the 2003rd term of the sequence?
(a) 3953 (b) 3943 (c) 3940 (d) 3950

!!!!"
71. How many positive integer values of a are possible such that is an integer?
!!!
(a) 12 (b) 13 (c) 15 (d) 16

72. In a test consisting of 15 questions, 3 marks are awarded for a correct answer, 1 mark is deducted for an
incorrect answer and no mark is awarded for an unattempted question. If a student attempts at least
one question in the paper, what is the number of distinct scores that he can get?
(a) 57 (b) 58 (c) 59 (d) None of these


123 | C A T C L A S S E S B Y A s h a n k D u b e y

! !"
73. Find the number of positive integer solutions of the equation + = 5
! !
(a) 0 (b) 1 (c) 2 (d) 3

74. N = 77777777, where the digit 7 repeats itself 429 times. What is the remainder left when N is
divided by 1144?
(a) 913 (b) 1129 (c) 777 (d) None of these

75. Ashank wrote down the squares of first n natural numbers in his notebook. He then erased one of the
numbers written by him and found that the sum of the numbers remaining on the notebook was 1432.
The square of which of the following numbers was erased by Ashank?
(a) 1 (b) 4 (c) 6 (d) 8

76. How many positive integer pairs (x, y) satisfy + = 2003
(a) 0 (b) 1 (c) 2 (d) 3

77. The product of three positive integers is 6 times their sum. One of these integers is the sum of the other
two integers. Find the sum of all distinct possible products of these three integers.
(a) 336 (b) 324 (c) 252 (d) 144

78. If N is a natural number less than 100, then for how many values of N are the numbers 6N + 1 and 15N +
2 relatively prime?
(a) 16 (b) 10 (c) 33 (d) None of these

79. A two digit number is divided by the sum of its digits. What is the maximum possible remainder?
(a) 13 (b) 14 (c) 15 (d) 16

80. If n is a natural number, then what is the sum of all the possible distinct remainders when
8! + 7! + 3! + 2! is divided by 10?
(a) 6 (b) 10 (c) 2 (d) 0

81. How many negative integer solutions exist for ! ! = 400?
(a) 4 (b) 6 (c) 8 (d) 3

82. A is the set of all the integers from 200 to 200. How many subsets of A having exactly 399 elements
can be formed such that the sum of all the elements in each subset is either 1 or 1?
(a) 401 (b) 200 (c) 399 (d) 400

83. Both the H.C.F and the difference of two numbers is 4. If the L.C.M of the two numbers is a three digit
number, then what is the maximum possible value of the smaller number?
(a) 44 (b) 198 (c) 60 (d) 64

84. N = 5!!!!!!!!!!"!!!"!
The digits of the number N are added to get another number. Then the digits of the number obtained
are added to get yet another number. The process is repeated till a single digit number is obtained.
What is that single digit number?
(a) 7 (b) 6 (c) 5 (d) 8


124 | C A T C L A S S E S B Y A s h a n k D u b e y

85. Find the number of possible solutions of m! = a! + b! + c!, where m, a, b and c are positive integers.
(a) 2 (b) 3 (c) 7 (d) 1

86. A natural number N has k distinct prime factors. If the total number of factors of N is 72, then what is
the product of all the possible values of k?
(a) 180 (b) 60 (c) 24 (d) 120

87. A leap year X has exactly the same calendar as another leap year Y. What can be the absolute difference
between the values of X and Y?
(a) 12 (b) 28 (c) 40 (d) All of these

88. If S1 = {1, 2, 3, 4, ... , 23} and S2 = {207, 208, 209, 210, 211, ... , 691}, how many elements of the set S2 are
divisible by at least four distinct prime numbers that are elements of the set S1?
(a) 9 (b) 8 (c) 11 (d) 12

89. A teacher asks one of her students to divide a 30-digit number by 11. The number consists of six
consecutive 1s, then six consecutive 2s, and likewise six 3s, six 4s and six 7s in that order from left to
right. The student inserts a three-digit number between the last 4 and the first 7 by mistake and finds
the resulting number to be divisible by 11. Find the number of possible values of the three-digit number.
(a) 82 (b) 92 (c) 81 (d) 91

90. A bag contains 35 tokens numbered 1 to 35. Five of them are picked randomly and are found to have
five consecutive numbers. If the product of the five numbers is P, how many distinct values that P can
assume are multiples of 16?
(a) 23 (b) 15 (c) 21 (d) 31

91. The product of three natural numbers N1, N2 and N3 is twelve times their H.C.F. How many ordered
triplets (N1, N2, N3) are possible?
(a) 12 (b) 15 (c) 18 (d) 21

92. If f(n) represents the sum of the digit(s) of n for n = 1, 2, 3, 4, , find the remainder when f(1) + f(2) + f(3)
+ f(4) + + f(100) is divided by 90.
(a) 1 (b) 11 (c) 46 (d) 0

93. The following addition is incorrect if considered in base-10. Find the base in which this addition is
correct.
66 + 87 + 85 + 48 = 132
(a) 11 (b) 17 (c) 26 (d) 24

94. I cut a piece of paper in four equal parts. Now, I cut exactly one out of these four parts into four equal
parts. Now, again if I keep on repeating the same process for infinite number of times, then which of the
following can be the total number of parts of paper at any instant of time?
(a) 2048 (b) 2050 (c) 2049 (d) 2051

!
95. If A and B are positive integers such that = 113.3125, then which of the following can be the
!
remainder when A is divided by B?
(a) 7 (b) 15 (c) 9 (d) 2


125 | C A T C L A S S E S B Y A s h a n k D u b e y

96. Which of the following cannot be expressed as the sum of three distinct composite numbers?
(a) 21 (b) 23 (c) 16 (d) 18

97. How many integers exist such that not only are they multiples of 2008!""# but also are factors of
2008!"!" ?
(a) 12 (b) 481 (c) 587 (d) 2008!"

98. The sum of four natural numbers a1, a2, a3 and a4 is 210. What can be the minimum possible L.C.M. of
these numbers?
(a) 60 (b) 84 (c) 105 (d) 42

99. 'a' and 'b' are natural numbers, such that 9 < a + b < 20 and 10b divides a! completely. How many
possible sets of (a, b) exists?
(a! is the product of the first 'a' natural numbers.)
(a) 8 (b) 20 (c) 19 (d) 21

100. A is a positive integer such that A is a multiple of 180 and A has 40 factors. If A is less than 3000, then
!
the value of is
!"
(a) 54 (b) 60 (c) 240 (d) 270

For Answer Keys and Solution Contact Ashank Dubey @ ashankdubeyclasses@gmail.com

! Ashank is unarguably India's best Quant trainer and 100%iler of CAT 16 & CAT - 15 (Qunat). He has
trained numerous CAT toppers ever since the CAT went online, including 99.9%tilers of CAT 2015
Shrikanth, Rushil, Sandeep, Abhilasha, Prakhar, Pratik, Shivani to name a few.


126 | C A T C L A S S E S B Y A s h a n k D u b e y

Practice Exercise 3

1. If ! ! is completely divisible by 2000, where a and b are positive integers, then the minimum value
that the product of a and b can assume is
(a) 20 (b) 15 (c) 12 (d) 10

2. The number of ordered pairs (x, y) that satisfy the equation 2!! 3!! = 55, where x and y are natural
numbers, is
(a) 0 (b) 1 (c) 2 (d) 3

3. What is the sum of all the possible values of a natural number n that satisfy
!
! 2 ! !!" = ! 2 !"!!!"
(a) 17 (b) 18 (c) 19 (d) 20

4. How many 10-digit natural numbers in binary are there in which at least two 1s come together?
(For example: 11102 is a valid 4-digit such number but 10012 is not.)
(a) 512 (b) 55 (c) 457 (d) None of these

5. A basket of fruit is being arranged out of apples, bananas, and oranges. What is the smallest number of
pieces of fruit that should be put in the basket in order to guarantee that either there are at least 8
apples or at least 6 bananas or at least 9 oranges?
(a) 20 (b) 21 (c) 22 (d) None of these

6. When the age of the plane is the same as the pilot's age now, the pilot will be exactly 32 years older
than the plane was when the pilot was half as old as the plane is now. How old can the pilot be?
(a) 74 (b) (24, 64) (c) [24, 64] (d) [24, 64)

7. The number of persons who booked ticket for the New Year's concert is a perfect square. If 100 more
persons booked ticket then the number of spectators would be a perfect square plus 1. If still 100 more
persons booked ticket then the number of spectators would be again a perfect square. How many
persons booked ticket for the concert?
(a) 2401 (b) 529 (c) 25 (d) Cannot be determine

8. If the last digits of the products 1.2, 2.3, 3.4, ... , n(n+1) are added, the result is 2010. What is the
minimum possible value of n?
(a) 1004 (b) 1005 (c) 1006 (d) None of these

9. What is the remainder obtained when 2!" is divided by 641?
(a) 0 (b) 1 (c) 640 (d) None of these

10. Several sets of prime numbers, such as {7, 83, 421, 659}, use each of the nine nonzero digits exactly
once. What is the smallest possible sum such a set of primes could have?


127 | C A T C L A S S E S B Y A s h a n k D u b e y

(a) 215 (b) 207 (c) 565 (d) 567

11. For what smallest positive integral n, factorial of n is divisible by 414?
(a) 414 (b) 9 (c) 23 (d) 207

12. 2010 inhabitants of Dream Land are divided into two groups: the Truth tellers who always tell the
truth and the Liars who always tell a lie. Each person is exactly one of the following a cricketer, a
guitarist or a swimmer. Each inhabitant was asked the three questions: 1) Are you a cricketer? 2) Are
you a guitarist? 3) Are you a swimmer? 1221 persons answered yes to the first question. 729 persons
answered yes to second question and 660 persons answered yes to third question. How many
Liars are present on the Dream Land?
(a) 400 (b) 500 (c) 550 (d) 600

13. How many positive integers are equal to 12 times the sum of their digits?
(a) 1 (b) 2 (c) 33 (d) None of these

14. If X = 1 2 3 4 5 6 7 8 9 10. How many possible values can X take?
(a) 55 (b) 56 (c) 1024 (d) 512

15. N is the smallest number that has 5 factors. How many factors does (N - 1) have?
(a) 2 (b) 3 (c) 4 (d) 5

16. What is the LCM of (2111 1) and (215 1)?
(!!!! !) (!!" !) (!!!! !) (!!" !)
(a) (26 1) (b) (c) (d) (23 1)
(!! !) (!! !)

17. HCF of two numbers is 17 and their sum is 7616. How many such pairs are possible?
(a) 201 (b) 96 (c) 88 (d) 192

18. Let S(n) = n in case n is a single digit number. If n greater than equal to 10 is an integer, S(n) is the sum of
the digits for n. Let N denotes the smallest positive integer such that n + S(n) + S(S(n)) = 99. What is S(n)?
(a) 9 (b) 10 (c) 15 (d) 18

19. What is the maximum value of n for which 2! completely divides 2010!?
(a) 9995 (b) 2002 (c) 1507 (d) None of these

20. If N = 0.abcdabcdabcdabcd and if a + c = b + d and a + b + c + d = 9, what will be the minimum value
of the natural no. which on multiplying with N gives an integer?
(a) 9999 (b) 1989 (c) 1234 (d) 101

21. How many ordered triplets (a, b, c) of non zero real numbers have the property that each number is
the product of the other two?
(a) 8 (b) 6 (c) 4 (d) None of these

22. While adding all the page numbers of a book, I found the sum to be 1000. But then I realized that two
page numbers (not necessarily consecutive) have not been counted. How many different pairs of two
page numbers can be there?


128 | C A T C L A S S E S B Y A s h a n k D u b e y

(a) 20 (b) 21 (c) 22 (d) None of these

23. For how many prime numbers (p), 5p + 1 is a perfect square?
(a) 1 (b) 2 (c) 3 (d) More than 3

24. A programmer carelessly increased the tens digit by 1 for each multi-digit Fermat number in a lengthy
!
list produced by a computer program. Fermat numbers are integers of the form: = 2! + 1 for
integer n > 1. How many numbers on this new list are prime?
(a) 0 (b) 1 (c) 2 (d) more than 2

25. What is the greatest common divisor of the 2010 digit and 2005 digit numbers below?
(a) 11111 (b) 33333 (c) 77777 (d) None of these

26. Let a1, a2,..a2011 represents the arbitrary arrangement of the numbers 1, 2,..2011. Then what is the
remainder when (a1 1)(a2 2)..(a2011 2011) is divided by 2?
(a) 0 (b) 1 (c) a2011 (d) Cannot be determined

27. Consider the numbers 3, 8, 13 103, 108. What is the smallest value of n such that every collection of n
of these numbers will always contain a pair which sums to 121?
(a) 12 (b) 13 (c) 14 (d) 16

28. The ordered pair of four-digit numbers (2025; 3136) has the property that each number in the pair is a
perfect square and each digit of the second number is 1 more than the corresponding digit of the first
number. How many ordered pairs of five-digit numbers exist with the same property?
(a) 1 (b) 2 (c) 3 (d) more than 3

29. Given that a and b are digits from 1 to 9, what is the number of fractions of the form a/b, expressed in
lowest terms, which are less than 1?
(a) 12 (b) 26 (c) 27 (d) None of these

30. If N be the number of consecutive zeros at the end of the decimal representation of the expression 1!
2! 3! 4! ............ 99! 100! Find the remainder when N is divided by 1000?
(a) 970 (b) 124 (c) 524 (d) 624

31. The road from village P to village Q is divided into three parts. If the first section was 1.5 times as long
and the second one was 2/3 as long as they are now, then the three parts would be all equal in length.
What fraction of the total length of the road is the third section?
(a) 5/18 (b) 5/14 (c) 14/5 (d) 18/5

32. Four different digits are chosen and all possible positive four-digit numbers of distinct digits are
constructed out of them. The sum of the four-digit numbers is 186648. How many different sets of such
four digits can be chosen?
(a) 1 (b) 2 (c) 3 (d) more than 3

33. How many 4 digit numbers exist in which, when two digits are removed, 35 remains (e.g. 2315 and 3215
will be there in the list)?
(a) 495 (b) 6 (c) 540 (d) None of these


129 | C A T C L A S S E S B Y A s h a n k D u b e y

34. What is the sum of the series: 22 + 42 + 62 + 102 + 162 +... + 7542 + 12202?
(a) 2408278 (b) 2408280 (c) 2408276 (d) None of these

35. Lohit: I am thinking of a two digit number. Bet you cant guess it.
Ashank: Bet I can.
Lohit: Well, Ill only tell you the remainders of my number with anything from 1 to 10. How many
questions do you think that you will have to ask?
Ashank: Hmmm! That depends on how lucky I am. But Im not going to take chances. I am sure that I can
guess your number with exactly ______ questions.
How many questions does Ashank tell Lohit he will ask?
(a) Two (b) Three (c) Four (d) Cannot be determine

36. Ashank and Lohith are playing the following game. They take turns writing down the digits of a six-digit
number from left to right; Ashank writes the first digit, which must be nonzero, and repetition of digits is
not permitted. Ashank wins the game if resulting six-digit number is divisible by 2, 3 or 5, and Lohith
wins otherwise. Who has a winning strategy?
(a) Ashank (b) Lohith (c) no one (d) cannot be determined

37. What is the least number of links you can cut in a chain of 21 links to be able to give someone all
possible number of links up to 21?
(a) 2 (b) 5 (c) 11 (d) None of these

38. Several weights are given, each of which is not heavier than 1 kg. It is known that they cannot be divided
into two groups such that the weight of each group is greater than 1 kg. Find the maximum possible
total weight of these weights.
(a) 2 (b) 3 (c) 4 (d) 8

39. How many integers less than 500 can be written as the sum of 2 positive integer cubes?
(a) 28 (b) 26 (c) 200 (d) None of these

40. In a group of people, there are 19 who like apples, 13 who like bananas, 17 who like cherries, and 4 who
like dates. (A person can like more than 1 kind of fruit.) Each person who likes bananas also likes exactly
one of apples and cherries. Each person who likes cherries also likes exactly one of bananas and dates.
Find the minimum possible number of people in the group.
(a) 32 (b) 33 (c) 34 (d) 35

41. How many positive integers N are there such that 3 N is a three digit number and 4 N is a four digit
number?
(a) 84 (b) 333 (c) 249 (d) None of these

42. What is the smallest possible difference between a square number and a prime number, if prime is
greater than 3 and the square number is greater than prime?
(a) 2 (b) 3 (c) 1 (d) 4

43. If N is a natural number less than 120, then for how many values of N do the numbers 12N + 1 and 32N +
6 have a common divisor, other than 1?
(a) 20 (b) 24 (c) 30 (d) None of these


130 | C A T C L A S S E S B Y A s h a n k D u b e y

44. A two digit number is divided by absolute value of difference of their digits. What is the maximum
possible remainder?
(a) 3 (b) 5 (c) 6 (d) 4

45. In 1936, my age was equal to the last two digits of my birth year. My grandfather said that it was true for
him also. Then, the sum of my age and my grandfathers age in 1936 was
(a) 84 (b) 86 (c) 90 (d) 94

46. N is a number such that 200 < N < 300 and it has exactly 6 positive divisors. How many different values
of N are possible?
(a) 12 (b) 13 (c) 14 (d) 15

47. N is the sum of the squares of three consecutive odd numbers such that all the digits of N are the same.
If N is a four-digit number, then the value of N is
(a) 3333 (b) 5555 (c) 7777 (d) 9999

48. In the nineteenth century a person was X years old in the year X2. How old was he in 1884?
(a) 43 (b) 58 (c) 68 (d) 78

49. In a mathematical game, one hundred people are standing in a line and they are required to count off in
fives as one, two, three, four, five, one, two, three, four, five, and so on from the first person in the
line. The person who says 'five' is taken out of the line. Those remaining repeat this procedure until only
four people remain in the line.
What was the original position in the line of the last person to leave?
(a) 93 (b) 96 (c) 97 (d) 98

50. The product P of three positive integers is 9 times their sum, and one of the integers is the sum of the
other two. The sum of all possible values of P is
(a) 621 (b) 702 (c) 540 (d) 336

51. In a bag, some slips of paper are kept with the numbers thirteen or fourteen written on them. The slips
with number thirteen written on them are five more than the slips with number fourteen written on
them.
Which of the following can be the sum of the numbers in the bag?
(a) 254 (b) 300 (c) 327 (d) 353

52. The value of x + y such that ! ! = 343, is
(a) 343 (b) 49 (c) 7 (d) A or B

53. If 11 sweets are distributed among four boys, then, which of the following is true?
(a) Two boys each received more than 1 sweet (b) One of the boys received more than 3 sweets
(c) One of the boys received fewer than 3 sweets (d) One of the boys received exactly 2 sweets


131 | C A T C L A S S E S B Y A s h a n k D u b e y

54. How many natural numbers between 1 and 100 have exactly four factors?
(a) 30 (b) 32 (c) 300 (d) 33

55. Privileged Number is a natural number which has two prime numbers as its neighbors on the number
line. For example, 4 and 12 are privileged numbers. What is the mean of all privileged numbers less than
100?
(a) 30.5 (b) 24 (c) 28.5 (d) 26

56. In how many ways can 7!" be written as product of 3 natural numbers?
(a) 21 (b) 20 (c) 19 (d) 18

57. N is a number such that the ratio of sum of its digit to product of its digits is 3: 40. If N is divisible by 37
and N is the smallest such number, how many factors does N have?
(a) 8 (b) 16 (c) 20 (d) 30

58. In a village of 2029 inhabitants, at least x villagers have the same English initials for their first name and
their surname. The least possible value of x is
(a) 4 (b) 3 (c) 6 (d) 5 (e) 2

59. My grandfather said he was 84 years old but he was not counting the Sundays. How old my grandfather
really was?
(a) 64 (b) 97 (c) 96 (d) 98

!"#$
60. Let S = . If you swap two digits of the numerator of S with two digits of the denominator, the
!"!#
fraction is equal to 1/3. What is the sum of the digits of the denominator of the new fraction?
(a) 8 (b) 9 (c) 12 (d) 15

61. Given that x and y are integers and 5 ! + 2 ! = 5922, what can be the unit digit of y?
(a) 3 (b) 5 (c) 7 (d) 9

62. The remainder when x200 2x199 + x50 2x49 + x2 + x + 1 is divided by (x 1)(x 2) is
(a) 7 (b) 2x 3 (c) 6x 5 (d) 0

63. If the remainder when x100 is divided by x2 3x + 2 is ax + b, then the values of a and b are
(a) 2100 and 1 (b) 2100 and 1 2100
100 100
(c) 2 and 2 2 (d) 2100 1 and 2 2100

!
64. What are the last two digits of ?
!!"#
(a) 08 (b) 48 (c) 36 (d) 18


132 | C A T C L A S S E S B Y A s h a n k D u b e y

65. Let x and y be two four-digit palindromes (numbers that read the same forwards and backwards) and z
be a five-digit palindrome. If x + y = z, how many values of z are possible?
(a) 2 (b) 3 (c) 4 (d) 5

66. Suppose you write the numbers 0, 1, 2, 3,, 20 on the board. You perform following operation 20 times
- You pick up two numbers at random (call them a and b), erase them, and write down |a - b|. Then, the
final number left on the board is
(a) odd (b) even (c) more than 10 (d) less than 10

67. Find the last two digits of 1120 - 9.
(a) 52 (b) 72 (c) 92 (d) 02

68. If S = 1 2 + 2 22 + 3 23 + 4 24 + ... + 20 220, find the remainder when S is divided by
19.
(a) 0 (b) 2 (c) 9 (d) 18

69. Find the remainder when 1 2 + 2 3 + 3 4 + ... + 98 99 + 99 100 is divided by 101.
(a) 0 (b) 1 (c) 99 (d) 100

70. How many numbers between 1 and 1000 are there such that n2 + 3n + 5 is divisible by 121?
(a) 0 (b) 5 (c) 7 (d) 10

71. Which of the following natural numbers can be expressed as the sum of the squares of six odd integers?
(a) 1996 (b) 1997 (c) 1998 (d) 1999

72. The number of integer solutions of the equation x2 + 12 = y4 is
(a) 2 (b) 4 (c) 6 (d) 8

73. For how many positive integers does n! end with exactly 100 zeroes?
(a) 0 (b) 3 (c) 4 (d) 5

74. How many ordered pairs (a, b) satisfy ! = ! + 1, where a and b are integers?
(a) 2 (b) 3 (c) 4 (d) 5

75. Let N 111...111.. (73 times). When N is divided by 259, the remainder is R1 and when N is divided by
32, the remainder is R2. Then R1 + R2 is equal to
(a) 6 (b) 8 (c) 253 (d) none of these

76. What are the last two non zero digits of 36! 24!?
(a) 08 (b) 36 (c) 64 (d) 88


133 | C A T C L A S S E S B Y A s h a n k D u b e y

77. For how many integers m is m3 - 8m2 + 20m - 13 a prime number?
(a) 1 (b) 2 (c) 3 (d) more than 3

78. P and Q are two distinct whole numbers and P + 1, P + 2, P + 3, ..., P + 7 are integral multiples of
Q + 1, Q + 2, Q + 3,..., Q + 7, respectively. What is the minimum value of P?
(a) 0 (b) 240 (c) 420 (d) 1080

79. We define a number 44...48...89 with the following procedure. First, we insert the number 48 between
the two digits of 49. Next, 48 is inserted between the digits 4 and 8 of the resulting number. Finally, we
repeat this second step a few times. Every number obtained this way is
(a) a prime number (b) a perfect square (c) a perfect cube (d) of the form (3n + 1)

80. Given a + 1 = b + 2 = c + 3 = d + 4 = a + b + c + d + 5, then the value of a + b + c + d is
(a) 2 (b) 6 (c) 1/2 (d) 10/3

81. N is a natural number. How many values of N exist, such that N2 + 24N + 21 has exactly three factors?
(a) 0 (b) 1 (c) 2 (d) 4

82. Let p be a prime number such that p 23. Let n = p! + 1. The no. of primes in the list (n + 1), (n + 2), (n +
3), (n + p - 1)
(a) 0 (b) 1 (c) 2 (d) 3

83. A natural number N is 100 times the number of its factors. Find the sum of the digits of N, given that N
has only 2 prime factors.
(a) 1 (b) 2 (c) 3 (d) 4 (e) 5

84. The digits a, b and c of the three-digit natural number abc satisfy the condition 49a + 7b + c =
286. Find the sum of the digits a, b and c.
(a) 16 (b) 56 (c) 21 (d) None of these

85. 123456789123456789.. up to 180 digits when divided by 11 will leave a remainder of
(a) 4 (b) 5 (c) 6 (d) 0

86. When a 4 digit number is multiplied by N, the 4 digit no. repeats itself to give an 8 digit no. if the 4 digit
no. has all distinct digits then N is a multiple of
(a) 11 (b) 27 (c) 7 (d) 73

87. If N = 539 x 2!" and M = 9 x 2!" , then the remainder when N is divided by M is
(a) 8 (b) 2!" (c) 8 x 2!" (d) None of these

88. If N = 2!""" x 5!""! , then the sum of the digits of N is equal to
(a) 7 (b) 8 (c) 10 (d) 13


134 | C A T C L A S S E S B Y A s h a n k D u b e y

89. If n is a positive integer such that 2n has 28 positive divisors and 3n has 30 positive divisors, then how
many positive divisors does 6n have?
(a) 32 (b) 34 (c) 35 (d) 36

90. Find the highest natural number N, less than 400, such that N can be written as sum of consecutive
natural numbers in 11 ways but cannot be written as sum of 11 consecutive natural numbers.
(a) 315 (b) 387 (c) 234 (d) None of these

!!""
91. If {x} denotes fractional part of x then is
!
(a) 1/8 (b) (c) 3/8 (d) 5/8

92. If [3(150 + )]! = 22752S then the value of single-digit number S is
(a) 0 (b) 3 (c) 5 (d) 9

93. The highest power of 12 that can divide 5!" 1 is
(a) 1 (b) 2 (c) 3 (d) 4

94. For how many positive integers x between 1 and 1000, both inclusive, is 4x6 + x3 + 5 is divisible by 7?
(a) 0 (b) 4 (c) 11 (d) 36

95. How many ones will you write in binary representation of (2!" 1)?
(a) 88 (b) 89 (c) 90 (d) 91

96. How many three-digit natural numbers have less than 5 prime factors?
(a) 900 (b) 890 (c) 871 (d) 843

97. For how many values of natural number A, where A < 500, is the HCF of A and 2500 equal to 1?
(a) 400 (b) 200 (c) 100 (d) 50

98. For any natural number n, then which of the following could represent the exact number of zeroes that
n! could end for some values of n.
(a) 17 (b) 29 (c) 30 (d) 32

99. What is the largest factor of 11! that is one bigger than a multiple of 6 ?
(a) 385 (b) 11! (c) 2816 (d) None of these

100. Find the largest positive integer n such that ! +100 is divisible by (n + 10)
(a) 890 (b) 900 (c) 760 (d) None of these

For Answer Keys and Solution Contact Ashank Dubey @ ashankdubeyclasses@gmail.com


135 | C A T C L A S S E S B Y A s h a n k D u b e y

Practice Exercise 4

1. How many natural numbers n are there, such that n! ends with exactly 30 zeroes?
(a) 0 (b) 1 (c) 3 (d) 4

2. Find the highest power of 3 in N = 1! 2! 3! .. 99!
(a) 2260 (b) 2280 (c) 2240 (d) 2220 (e) None of these

3. What is the highest power of 3 in 11972 1?
(a) 486 (b) 0 (c) 1 (d) 6 (e) 243

4. 1388, 3309 and 7151 were divided by a certain three-digit number. The remainders in all the cases were
found to be the same. What was the remainder?
(a) 17 (b) 32 (c) 113 (d) 11

5. What is the least value of n such that difference of (16!"#! 7) and (2012!" + n) is divisible by 3?
(a) 0 (b) 1 (c) 2 (d) 3

6. What is the remainder when 7 + 77 + 777 + 7777 + . (till 100 terms) is divided by 8?
(a) 0 (b) 2 (c) 4 (d) 6

7. Let N = 22! 23! 24! When N is divided by 260 the quotient is X. What is the remainder when X is
divided by 10?
(a) 1 (b) 3 (c) 5 (d) 7

8. The number 2!" is a 9 digit number with distinct digits. Which digit is missing?
(a) 4 (b) 1 (c) 9 (d) 5

9. What is the sum of digits or smallest positive integer N such that N! is a multiple of 102009?
(a) 13 (b) 14 (c) 15 (d) 16 (e) 17

10. If the factorial of the number X contain Y trailing zeroes, where Y is the least four digit number. Then
what is the digital sum of the minimum value of X?
(a) 9 (b) 5 (c) 2 (d) None of these

11. Let S(n) = n when n is a single natural number and S(n) = sum of the digits of n, when n > 10. Let N
denote the smallest positive integer such that N + S(N) + S(S(N)) = 99. What is S(N)?
(a) 12 (b) 18 (c) 9 (d) 15


136 | C A T C L A S S E S B Y A s h a n k D u b e y

12. S is a set of all natural numbers having 24 factors and 3 prime factors. Find the digital sum of the
smallest element in set S? (Digital sum is defined as the sum of all the digits of number until we get a
single digit)
(a) 9 (b) 6 (c) 1 (d) 3

13. For given pair (x, y) of positive integer, such that 3x11y = 1 and given that x can take maximum value of
1000, how many such pairs are possible?
(a) 46 (b) 71 (c) 88 (d) None of these

14. How many integral solutions do the equation 11x + 7y = 4 has, if x < 400 and y < 400?
(a) 91 (b) 93 (c) 90 (d) None of these

15. How many N not greater than 2100 are there such that 2N N2 is divisible by 7?
(a) 599 (b) 600 (c) 602 (d) None of these

16. For how many natural numbers (n) less than 17, n! + (n + 1)! + (n + 2)! is divisible by 49?
(a) 4 (b) 3 (c) 2 (d) 5
.!
. ..
! !.
17. What is the remainder when 7! is divided by 29?
(a) 5 (b) 6 (c) 7 (d) None of these

18. 20!""# + 16!""# 3!""# 1 is divisible by:
(a) 253 (b) 91 (c) 317 (d) 323

19. Find the number of positive integers N for which ! + 2014 is square of an integer.
(a) 0 (b) 1 (c) 4 (d) None of these

20. Let n! = 1 x 2 x 3 x . x n for integer n 1. If P = 1! + (2 x 2!) + (3 x 3!) + + (10 x 10!), then (P + 2)
when divided by 11! leaves a remainder of
(a) 10 (b) 7 (c) 1 (d) 0

21. A = 1111.1 (46 times) and M = 2222..2 (64 times). What is the remainder when A
M is divided by 18?
(a) 1 (b) 2 (c) 4 (d) none of these

22. N = 1010 + 10100 + .. + 1010000000. What is the remainder when N is divided by 7?
(a) 6 (b) 5 (c) 4 (d) 0

23. If 8 is written 88 times side by side, we will get a large number of 88 digits. What is the remainder when
this large number is divided by 7?
(a) 1 (b) 4 (c) 3 (d) 5


137 | C A T C L A S S E S B Y A s h a n k D u b e y

24. 4 is written 3400 times side by side to give you a big number i.e. 444..44(3400 times). What is the
remainder when 37 divide this number?
(a) 7 (b) 26 (c) 33 (d) 4

.! !"#$% .! !"#$% .! !"#$%


!! !! !! !! !! !!
25. Find the remainder when 1! + 2! + 3! + . +
!""#! !""#! .! !"#$%
2009! is divided by 10. Where k is a natural number greater than 2009.
(a) 5 (b) 0 (c) 1 (d) 7 (e) None of these

26. What is the remainder when 105! Is divided by 107?
(a) 1 (b) 0 ` (c) 105 (d) None of these

27. What is the remainder when 17! Is divided by 23?
(a) 15 (b) 14 (c) 3 (d) None of these

!"!!"!
28. Find the remainder when 18!!"! is divided by 19 where N! is product of first N positive integers.
(a) 1 (b) 0 (c) 18 (d) None of these

!"!!""""
29. Find the reminder when 38 is divided by 17
(a) 1 (b) 16 (c) 8 (d) 13 (e) None of these

!"!!"!
30. Find the remainder when 18!!"! is divided by 19 where N! is product of first N positive integers.
(a) 0 (b) 1 (c) 18 (d) 17

31. What is the remainder when (3401)17 is divided by 306?
(a) 305 (b) 35 (c) 1 (d) none of these

32. For prime numbers A and B both greater than 3, what is the highest integer which divides A - B
completely?
(a) 12 (b) 24 (c) 48 (d) None of these

33. Find the smallest number by which 9408 must be divided so that the quotient is a perfect cube.
(a) 42 (b) 24 (c) 56 (d) None of these

34. In a three digit number with only odd digits, exactly two of the digits are equal. The number is divisible
by 11. How many values can the number assume?
(a) 3 (b) 4 (c) 1 (d) 2 (e) 5

35. n is selected from the set {1, 2, 3, .,100} and the no. 2n + 3n + 5n is formed. How many values n can
take so that the number formed is divisible by 4.
(a) 49 (b) 51 (c) 50 (d) None of these


138 | C A T C L A S S E S B Y A s h a n k D u b e y

36. A man has 1! + 2! + 3! + .. 1000! Chocolates which he has to divide equally among n Children. What
is the only possible value of n from the options below?
(a) 5 (b) 7 (c) 16 (d) 9

37. x = 7287, y = 30x. Find reminder when y is divided by 11
(a) 5 (b) 9 (c) 6 (d) 3 (e) None of these

38. Find the remainder when (10n)! is divided by 10n where n is a natural number greater than
2012. {n! is product of first n positive integers}
(a) 2011 (b) 1 (c) 0 (d) None of these

39. Find the remainder when 1 + 2!" + 3!" + 4!" + 5!" + + 38!" is divided by 57.
(a) 1 (b) 0 (c) 56 (d) None of these

40. What is the remainder when 81 !" + 27 !" + 9 !" + 3 !" + 1 is divided by (3!" + 1)
(a) 0 (b) 1 (c) 61 (d) 121

41. The number of digits in the smallest number consisting of only ones and zeroes and divisible by 225 is:
(a) 11 (b) 9 (c) 10 (d) 8

42. Find the largest natural number n such that n3 + 555 is completely divisible by (n + 5)
(a) 225 (b) 425 (c) 555 (d) None of these

43. Two natural numbers x and y when divided by another natural number k leave positive remainders a
and b, respectively. When x + y is divided by k, the remainder is c. Which of the following statements is
necessarily true?
(a) c = a + b (b) k can be uniquely identified, if a, b and c are known
(c) c = a + b or c = (a + b) k (d) all the above statements are false

44. If N is a natural number greater than 1 and D(N) is the number of divisors of N, then the last term of the
infinite series D(N), D(D(N)), D(D(D(N))),..
(a) 3 (b) 0 (c) 2 (d) 1

45. The sum of eight three-digit consecutive even numbers is S. When S is divided by 5, it results in a perfect
cube. How many sets of such eight numbers are possible?
(a) 1 (b) 2 (c) 3 (d) 4

46. For what smallest positive integral n, factorial of n is divisible by 414?
(a) 23 (b) 25 (c) 14 (d) None of these

47. If P is product of 15 consecutive positive integers, then P is certainly multiple of
(a) 69 (b) 65 (c) 57 (d) 58


139 | C A T C L A S S E S B Y A s h a n k D u b e y

48. Let S (n) be the sum of the squares of the first n positive odd integers. What is the unit digit of S
(12345)?
(a) 5 (b) 6 (c) 9 (d) 0

!"!""""
49. What is the unit digit of where [x] is the greatest integer less than or equal to x?
!"!"" ! !
(a) 3 (b) 4 (c) 2 (d) 5

!
50. Find the 50th digit after decimal in the expansion of .
!"
(a) 1 (b) 3 (c) 7 (d) 6

51. What is the units digit of the following product: (8 5)(82 52)(83 53) . (82005 52005)
(a) 1 (b) 3 (c) 7 (d) 9

52. In the value of the number 30!, all the zeroes at the end are erased. Then, the unit digit of the number
that is left is
(a) 2 (b) 4 (c) 6 (d) 8

53. Let (m, n) be a pair of integers such that 5m2 + 2n2 = 2002. Which of the following digits could be the
units digit of n?
(a) 2 (b) 3 (c) 5 (d) 9

54. Let S = (3 + 3! + 3! + + 3!"" ) (7 + 7! + 7! + + 7!"# ). The last two digits of S are
(a) 00 (b) 07 (c) 43 (d) 93

55. What is the remainder when 1202!" is divided by 1000?
(a) 126 b) 006 (c) 001 (d) none of these

56. N = 3333333..333 upto 51 digits 6666666.666 upto 51 digits. What is the 52nd digit of N
from right?
(a) 8 (b) 1 (c) 7 (d) 2 (e) 3

57. It is very big number and the 6 digits of this number are 117649(7 ! = ..117649) and 7! = 117649.
For this to be possible what is the minimum value of X?
(a) 20 (b) 5004 (c) 5006 (d) None of these

58. If N be the number of consecutive zeroes at the end of the decimal representation of the expression 1!
2! 3! 4! ............ 99! 100! Find the remainder when N is divided by 1000?
(a) 970 (b) 154 (c) 124 (d) None of these


140 | C A T C L A S S E S B Y A s h a n k D u b e y

59. Let f(n) be the sum of the distinct positive prime divisors less than 50 for all positive integers n. For
example: f(15) = 3 + 5 = 8 and f(61) = 0. Find the remainder when f(1) + f(2) + + f(99) is divided by
1000.
(a) 368 (b) 998 (c) 268 (d) None of these

60. After the division of a number successively by 3, 4 and 7, the remainder obtained are 2, 1 and 4
respectively. What will be the remainder if 16 divide the same number?
(a) 53 (b) 75 (c) 41 (d) cannot be determined

61. For what smallest positive integer N, will the number 3! + 5! + 7! + ..+ (2n + 1)! be a perfect square?
(a) 6 (b) 8 (c) 98 (d) None of these

! ! !
62. Find the sum of all possible values of such that + = , where x and y are positive integers.
! ! !
(a) 50 (b) 45 (c) 30 (d) 35

63. A is the product of first 100 multiples of 8, i.e. A = 8 16 24 .. 800. How many zeroes would be
there at the end of A?
(a) 10 (b) 12 (c) 11 (d) none of these

64. A = 626! 625!. How many consecutive zeroes would be there at the end of A?
(a) 156 (b) 160 (c) 1 (d) none of these

65. N! is having 137 zeroes at its end. How many values of N are possible?
(a) 4 (b) 5 (c) 6 (d) None of these

66. A positive integer N has exactly 31 positive divisors. How many distinct remainders can be obtained
when N is divided by 9?
(a) 2 (b) 3 (c) 6 (d) none of these

67. A is the product of ten consecutive two-digit numbers. Y is the highest power of 5 in A. what can be the
maximum value of Y?
(a) 4 (b) 3 (c) 3 (d) 1

68. Sheru had 1000 rupees in his bank account and he wished to withdraw all his saving because he had
finished all cash with him. But bank allowed only two types of transactions i.e. a withdrawal of 350 & its
multiples and deposit of 231 & its multiples. Find the largest amount that Sheru can withdraw from his
bank account.
(a) 994 (b) 1034 (c) 867 (d) None of these

69. Out of the first 200 even natural numbers. How many even numbers exist having even number of
factors?


141 | C A T C L A S S E S B Y A s h a n k D u b e y

(a) 90 (b) 10 (c) 190 (d) none of these

70. How many two digits odd numbers are there with 10 factors?
(a) 1 (b) 2 (c) 3 (d) 0

71. The number of divisors of every natural number from 1 to 1000 is calculated. Which natural number has
the highest number of divisors?
(a) 840 (b) 630 (c) 960 (d) None of these

72. How many four-digit numbers are there with less than 6 different prime factors?
(a) 1224 (b) 8476 (c) 9000 (d) 7613

73. Let x be the smallest value of x for which 198x 1 is divisible by 139. Then the number of factors of x is
(a) 4 (b) 5 (c) 8 (d) 16

74. M is a positive integer less than 1000 having exactly 10 factors. How many different values of M are
there?
(a) 20 (b) 21 (c) 22 (d) none of these

! ! ! !!! !
75. Find number of integers, N such that is an integer.
! ! ! ! ! !"
(a) 1 (b) 2 (c) 3 (d) 4

76. How many factors of 105 end with zero?
(a) 5 (b) 16 (c) 25 (d) none of these

77. N = 215 37 510. How many factors of N are multiples of 360 but not multiple of 10800?
(a) 402 (b) 240 (c) 204 (d) 420

78. N = 28 310 58 72. How many factors of N are multiples of 360 but not a multiple of 540?
(a) 210 (b) 162 (c) 144 (d) 124

79. How many natural numbers are factors of 540 but not a factor of 720?
(a) 6 (b) 9 (c) 12 (d) 15

80. How many integers exist such that not only are they multiples of 20092009 but also are factors of
20092020?
(a) 11 (b) 242 (c) 276 (d) 200911 (e) 637

81. Let D(a, b, c) denote the number of multiples of a that are less than c and greater than b, For example,
D(2, 3, 8)= 2 because there are two multiples of 2 between 3 and 8. What is D(93, 94, 96)?
(a) 71 (b) 719 (c) 720 (d) 7200


142 | C A T C L A S S E S B Y A s h a n k D u b e y

82. A number has exactly 32 factors out of which 4 are not composite. Product of these 4 factors (which are
not composite) is 30. How many such numbers are possible?
(a) 4 (b) 6 (c) 8 (d) None of these

83. Find the sum of the factors of 8! Which are odd and of the form 3m + 2, where m is a natural no.
(a) 35 (b) 40 (c) 5 (d) None of these

84. Find the no. of divisors of N = 27 35 53 which are for the form 4t + 1, where t is a natural number
(a) 6 (b) 12 (c) 11 (d) None of these

85. N = ! ! ! where a, b, c are squares of distinct primes. How many factors of N have 36 factors?
(a) 18 (b) 36 (c) 9 (d) None of these

86. The sum of 20 distinct numbers is 801. What is their minimum LCM possible?
(a) 480 (b) 360 (c) 840 (d) 420

87. What is the minimum number of positive factors of a 6-digit number of the form abbabb, where a and b
represent distinct natural numbers
(a) 10 (b) 6 (c) 16 (d) 2

88. The product of four numbers is 9261000 and each of these four numbers is the product of three distinct
prime numbers? What is the difference between the smallest and the largest of these four numbers?
(a) 55 (b) 65 (c) 75 (d) None of these

89. 10000! = 100! ! x P, where P and K are integers. What can be the maximum value of K?
(a) 102 (b) 103 (c) 104 (d) 105

90. Let d1, d2. dk be all the factors of a positive integer n in ascending order including 1 and n. Suppose d1 +
d2 + .. dk = 72. Then the value of 1/d1 + 1/d2 + .. + 1/dk is
(a) n2/72 (b) n/72 (c) 72/n (d) cannot be determined

91. How many 7-digit numbers divisible by both 5 and 7 can be formed by using only the digits 5 and 7?
(a) 8 (b) 10 (c) 9 (d) 11

92. A number have exactly 1024 factors. What can be the maximum number of prime factors of this
number.
(a) 1023 (b) 24 (c) 10 (d) none of these

93. Let N = 24 32 72 136 and P is the product of all the factors of N. What is the total number of factors
of P?
(a) 630 315! 945 (b) 630! 315 945 (c) 630 315 945! (d) 631 316! 946


143 | C A T C L A S S E S B Y A s h a n k D u b e y

94. T is a four digit number. A new four digit number C is obtained by subtracting 19 times sum of digits of T
from T i.e. C = T 19 (sum of digits of T)
Rashi erased one digit of C and remaining three in increasing order are 0, 3, 4. What is the erased digit?
(a) 2 (b) 5 (c) 7 (d) None of these

95. Find sum of all numbers which give same remainder as the quotient when divided by 21
(a) 4620 (b) 4500 (c) 4600 (d) None of these

96. LCM of two numbers is 24948 and their HCF is 42. How many such pairs of two numbers are possible?
(a) 4 (b) 6 (c) 8 (d) 5

97. The LCM of 45, 65 and a natural number k is 6435. The HCF of these three numbers is 5. What is the
least possible value of k?
(a) 65 (b) 55 (c) 225 (d) 15

98. The HCF of (n + 3) and (7n + 48) is k, where n is a natural number. How many values of k are
possible?
(a) 4 (b) 5 (c) 1 (d) 2 (e) 3

99. A is a natural number (3 < A < 500). How many As are there so that HCF of A and 1250 is 1?
(a) 200 (b) 198 (c) 150 (d) 197

100. Find the sum of all rational numbers x/90 which are less than 1 and such that HCF (x, 90) = 1.
(a) 10 (b) 12 (c) 15 (d) 24

101. K and (K + 200) both are perfect squares. While (K + 100) is four more than a perfect square. Find the
sum of all possible values of K
(a) 604 (b) 500 (c) 598 (d) 610

102. When the numbers 5, 7, 11 divide a multiple of 17, the remainders left are respectively 4, 6 and 10.
Which multiple of 17 gives the least number that satisfies the given condition?
(a) 384th (b) 317th (c) 385th (d) none of these

103. The smallest number divisible by 3, 5 and 7 and also the sum of the digits divisible by 3, 5 and 7 is made
up of how many different digits?
(a) 3 (b) 4 (c) 5 (d) None of these

104. Let N be a number such that N is dividable by every natural number less than the cube root of N. What is
highest possible value of N?
(a) 420 (b) 500 (c) 221 (d) None of these

105. How many positive integers A are there such that 2A is square, 3A is cube and 5A is fifty power?


144 | C A T C L A S S E S B Y A s h a n k D u b e y

(a) 1 (b) 5 (c) 10 (d) None of these

106. A is a three-digit natural number. If you strike out extreme left digit of A, remaining number is a perfect
square. If you strike out extreme right digit of A, remaining number is still a perfect square. How many
different values of A are possible?
(a) 2 (b) 3 (c) 4 (d) 5

107. In a big hostel, there are 1000 rooms. In that hostel only even numbers are used for room numbers i.e.
the room numbers are 2, 4, 6., 1998, 2000. All the room has one resident each. One fine morning, the
warden calls the entire resident and tells them to go back to their rooms as well as multiples of their
room numbers. When a guy visits a room and finds the door open, he closes it, and if the door is closed,
he opens it. All 1000 guys do this operation. All the doors were opened initially.
Total number of doors that are closed is
(a) 31 (b) 969 (c) 22 (d) none of these

108. How many 3 digit numbers are there which can be expressed as a perfect square, perfect cube and a
perfect fourth power?
(a) 2 (b) 3 (c) 1 (d) none of these

109. 8! + ! = 10!" , where a is an odd number and b is a natural number. How many values of a are
possible?
(a) 0 (b) 1 (c) 2 (d) 3

110. In how many ways can 145! be expressed as a sum of squares of two natural numbers?
(a) 1 (b) 2 (c) 3 (d) 4

111. Find the number of positive integral solutions to ! + ! = 7225, where a and b are distinct integers.
(a) 6 (b) 8 (c) 2 (d) 4

112. How many numbers below 100 are co-prime to 100 but not co-prime to 1500?
(a) 12 (b) 14 (c) 16 (d) 18

113. What is sum of the digits of the least number that can be expressed as product of 2 co-primes in 8 ways?
(a) 3 (b) 6 (c) 1 (d) 7 (e) None of these

114. What is the sum of all natural numbers which are less than 10!"" and are co-prime to it?
(a) 2 x 10!"" (b) 10!"" (c) 2 x 10!"" (d) 10!""

115. Let S be the set of all 3-digit numbers. What is the least number of terms that you will have to
remove from S so that the sum of no two terms is a multiple of 3?
(a) 599 (b) 500 (c) 600 (d) 699


145 | C A T C L A S S E S B Y A s h a n k D u b e y

116. Find the sum of all naturals numbers less than 2012 which are multiples of both 3 and 5 but not divisible
by 2.
(a) 67332 (b) 67339 (c) 67335 (d) None of these

117. Find the sum of the digit of the number which is cube of 99999..9999 ..(2014 digits).
(a) 9999 (b) 36252 (c) 45342 (d) None of these

118. Three friends, returning from a movie, stopped to eat at a restaurant. After dinner they paid their bill
and noticed a bowl of mints at the front corner. Sita took one-third of the mints but returned four
because she had a momentary pang of guilt. Fatima then took one-fourth of what was left but returned
three for similar reasons. Eswari then took half of the remainder but threw two back in the bowl. The
bowl had only 17 mints left when the raid was over. How many mints were originally in the bowl?
(a) 38 (b) 31 (c) 41 (d) None of these

119. A new sequence of positive integers is formed by removing all perfect cubes as {2, 3, 4, }. What is
the 2012th number in the sequence?
(a) 2024 (b) 2197 (c) 2025 (d) 2023

120. A student wrote all the natural no. from 2 to 10000 on a blackboard, one after the other. Another
student came and erased all perfect cubes. If students come in this way and erase all the higher powers,
find the no. of students who erase at least one number.
(a) 6 (b) 7 (c) 12 (d) None of these

121. The number 5450 has the property that the number formed by first two digits is four more than the
number formed by last two digits. N is a four-digit number with this property, which is a perfect square.
Find the sum of the digits of N.
(a) 18 (b) 22 (c) 17 (d) none of these

122. Two elements of set S = {1, 2, 3, 4, 5, 6, 7, 8, 9, 10} are chosen such that a/b is in its simplest form. How
many ordered pairs (a, b) are there?
(a) 74 (b) 66 (c) 63 (d) 62

123. Consider the following sequence:- 1, 2, 1, 2, 2, 1, 2, 2, 2, 1, 2, 2, 2, 2, 1, ...... S(n) is defined as sum of first
n terms of the above sequence. Find the value of n such that S(n) = 2010
(a) 254 (b) 256 (c) 257 (d) None of these

124. Find the sum of all odd numbers between 901 and 999
(a) 45600 (b) 46599 (c) 46501 (d) 47500 (e) None of these

125. You are selecting 10 numbers randomly out of the first 100 odd numbers. Sum of these 10 odd numbers
is A. How many different values of A are possible?
(a) 100C10 (b) 1801 (c) 1800 (d) 901


146 | C A T C L A S S E S B Y A s h a n k D u b e y

126. Given that Q = 1! + 2! + 3! + 4! + . + (n 1)! + n!. For how many values of n, Q is a perfect square?
(a) 1 (b) 2 (c) 3 (d) 4 (e) More than 4

127. How many four digit perfect squares abcd are possible such that dcba is also a four digit perfect
square and is also a factor of abcd? (Given a is not equal to zero)
(a) 0 (b) 1 (c) 2 (d) 3 (e) More than 3

128. If 3a = 5, 5b =9, 9c = 12, 12d = 25 and 25e = 27 for some real numbers a, b, c, d and e. Find the value of
abcde.
(a) 1 (b) 2 (c) 3 (d) 4

! ! ! ! ! !!
129. + + + + can be written as . Find the value of a + b.
!! !! !! !! !! !! !! !! !! !! !!
(a) 19 (b) 27 (c) 10 (d) None of these

130. The series of numbers 1, 1/2 , 1/3 , . . . . 1/1972 is taken. Two numbers are taken from the series, say x
and y. The operation x + y + xy is performed to get a consolidated number. The process is repeated. The
number obtained in the end will be
(a) 1970 (b) 1971 (c) 1972 (d) 1973 (e) None of these

131. If f(x) = sum of all the digits of x, where x is a natural number, then what is the value of f(101) + f(102)
+ f(103) + + f(200)?
(a) 1000 (b) 901 (c) 999 (d) 1001 (e) 1111

132. g(P) represents the product of all the digits of P, e.g. g(45) = 4 5. What is the value of g(67) + g(68) +
g(69) + .. + g(122) + g(123)?
(a) 1381 (b) 1281 (c) 1481 (d) 1181 (e) None of these

133. How many three digit even numbers have all distinct digits?
(a) 232 (b) 254 (c) 256 (d) None of these

134. Take any two-digit number and multiply the digits together. If this process is continued, all 2-digit
number will become a single digit number. For example: 73: 7 3 = 21; 2 1 = 2
How many two digit numbers will finish on zero?
(a) 12 (b) 24 (c) 36 (d) 48

135. If distinct letters represent distinct single digit non-negative integers such that
A + BC + DEF = GHIJ,
Find G + H + I + J. [Here BC, DEF and GHIJ are 2-digit, 3-digit and 4-digit numbers respectively.]
(a) 11 (b) 18 (c) 9 (d) None of these


147 | C A T C L A S S E S B Y A s h a n k D u b e y

136. Given that


.. where every letter represents a different single digit positive integer. Find the minimum value of the sum A
+ B + C.
(a) 6 (b) 7 (c) 9 (d) 11

137. The letters in the alphanumeric addition are all different. All the letters are digits from 0 to 9.What is the
value of (S + I + X + T + Y)?


(a) 22 (b) 13 (c) 32 (d) None of these

138. Substitute different digits (0, 1, ., 9) for different letters in the problem below, so that the
corresponding addition is correct and it results in the maximum possible value of MONEY.


The resulting value of money is
(a) 10364 (b) 10563 (c) 10978 (d) none of these

139. Pav Bhaji Buns are sold in packages of 6 and 11 only at a certain store. Find the largest number of Buns
that cannot be purchased from the store.
(a) 49 (b) 43 (c) 62 (d) 59

140. Lets see the property of two square numbers:
9801 = (98 + 01)2
3025 = (30 + 25)2
Let k be the third number which has the same property. Then
(a) 1000 < k < 2000 (b) 2000 < k < 3000 (c) 3000 < k < 4000 (d) 4000 < k

141. K is a set of five consecutive prime numbers such that the sum of all elements in K is greater than 200
and less than 300. Which of the following cannot be the sum of the elements in the set K?
(a) 221 (b) 263 (c) 243 (d) 271 (e) 287

142. For how many prime numbers p, is p4 + 15p2 1 also a prime number?
(a) 0 (b) 1 (c) 2 (d) 3 (e) None of these


148 | C A T C L A S S E S B Y A s h a n k D u b e y


143. The prime numbers p & q and natural number n satisfy the following equality: 1/p + 1/q + 1/pq = 1/n. If
q > p, then the value of q is:
(a) 11 (b) 7 (c) 5 (d) 3

144. What is the smallest prime number which can be written a sum of two prime numbers as well as sum of
three prime numbers not necessarily distinct?
(a) 13 (b) 17 (c) 19 (d) 23

Directions for questions 145 and 146: If you wrote down the number from 1 to 20 in order, youd have 1 2 3
4 5 6 7 8 9 10 11 12 13 14 15 16 17 18 19 20. You would have used 31 digits, 2 zeroes, and the twentieth
digit would be 1. If you were to write down the numbers from 1 to 1000000 in order, then

145. How many digits would you use?


(a) 5888896 (b) 548886 (c) 5388876 (d) 5288886

146. How many 0s would you use?
(a) 458897 (b) 478896 (c) 447897 (d) 488895

147. A positive integer which reads same forward as backwards, such as 121 and 2002 is termed as
palindromic. How many 10 digit palindromic numbers are prime?
(a) 9 (b) 9000 (c) 9 104 (d) None of these

148. 6 lies exactly midway of 3 and 3! . Which of the following numbers do not lie exactly midway of a
positive integer and its square?
(a) 3 (b) 10 (c) 15 (d) 30

149. A candidate takes a test and attempts all the 120 questions therein. If 1/8 of the questions carry 1/8
negative marks, of the questions carry negative marks and the rest of the question carry negative
mark, then what is the difference between maximum and minimum marks that he can score gien that he
gets 1 mark for every correct answer?
! ! ! !
(a) 73 (b) 83 (c) 147 (d) 166
! ! ! !

150. 31 students in a row were numbered 1, 2,..,31 in order. The teacher wrote down a number on the
blackboard
Student 1 said the number is divisible by 1,
Student 2 said the number is divisible by 2,
and so forth . Until
Student 31 said the number is divisible by 31.
The teacher remarked: Very well pupils, but two of you gave a wrong statement, and those two sit
beside each other. Determine those two.
(a) 23, 24 (b) 17, 18 (c) 29, 30 (d) 16, 17 (e) 30, 31

Você também pode gostar